Vous êtes sur la page 1sur 208

PLUS MEDIC A

Manual de INFECTOLOGÍA PLUS MEDIC A

POR AMOR A LA MEDICINA

INFECTOLOGÍA

ramón Flores

Página 1
www.plus-medica.com administrador@plus-medica.com
Manual de INFECTOLOGÍA PLUS MEDIC A

Por amor a la Medicina

INFECTOLOGÍA

6 ª Edición 2018

Autor y editor
Ramón Flores Valdeiglesias1

Coautores
Yuri Garcia Córtez2
Daniel Flores Valdeiglesias3

1
Médico internista
Asistente del Dpto de Medicina del Hospital Nacional Dos de Mayo
2
Médico infectólogo
Asistente del Dpto de Medicina especializada del HNDM
3
Médico especialista en Medicina Física y Rehabilitación
Asistente del Hospital Rebagliati Martins

Colaboradores
Equipo de creativos de PLUS MEDIC A

Derechos Reservados 2018


Prohibida su venta

Página 2
www.plus-medica.com administrador@plus-medica.com
Manual de INFECTOLOGÍA PLUS MEDIC A

Página 3
www.plus-medica.com administrador@plus-medica.com
Manual de INFECTOLOGÍA PLUS MEDIC A

PRESENTACIÓN
PLUS MEDIC A se complace en presentar la 7a. Edición
del Manual PLUS de Infectología , se han realizado
actualizaciones en base a las guías médicas más
recientes.

En esta edición seguimos con nuestro estilo superdidáctico


y confiable.

Página 4
www.plus-medica.com administrador@plus-medica.com
Manual de INFECTOLOGÍA PLUS MEDIC A

INDICE
1.Exámenes de Infectología RM por temas………………………………………… 5
2.Microbiología …………………………………………………………………………21
3.Antibióticos …………………………………………………………………………39
4.Antimicóticos …………………………………………………………………………61
5.Dengue ……………………………………………………………………………… 66
6.Mononucleosis infecciosa………………………………………………………… 81
7.Fiebre amarilla…………………………………………………………………………88
8.Enfermedad de Chagas……………………………………………………………… 94
9.Bartonelosis ……………………………………………………………………… 105
10.Leishmaniosis…………………………………………………………………… 114
11.Infección por VIH ………………………………………………………………123
12. Malaria …………………………………………………………………………… 154
13. Fasciolasis ………………………………………………………………………166
14. Rabia……………………………………………………………………………… 172
15. Brucelosis ………………………………………………………………………182
16. Fiebre tifoidea …………………………………………………………………191
17. Leptospirosis ………………………………………………………………… 199
18. Anexo ….. …..……..…………….………………………………………………… 207

Página 5
www.plus-medica.com administrador@plus-medica.com
Manual de INFECTOLOGÍA PLUS MEDIC A

INFECTOLOGÍA RM 2012-17

2017
( 13 preguntas )
CON CLAVE Y POR TEMAS

RM 2017-A (20): ¿Cuál de las siguientes A. Inhibición competitiva de la dihidropteroato


poblaciones son consideradas de alto riesgo para sintasa
infección por tuberculosis y deben recibir B. Inhibición de la DNA girasa
tratamiento preventivo? C. Inhibición de la transpeptidasa
A. Contactos VIH (+) con PPD mayor de 5mm D. Unión a la subunidad 30S ribosomal
B. Fumadores crónicos con fibrosis pulmonar E. Inhibición de las enzimas CYP hepáticas
C. Usuarios de drogas por vía oral
D. Convertidores recientes de tuberculina mayor RM 2017-A (77): En una gestante con
de 5mm diagnóstico de VIH en TARGA, el parto por
E. Personal de salud con reacción de PPD mayor cesárea se realizará cuando…
de 5mm A. la gestación es un óbito fetal.
B. la dilatación cervical es mayor de 4 cm.
RM 2017-A (38): Gestante con VIH positivo C. la ruptura prematura de membranas es mayor
diagnosticada durante el control prenatal. La de cuatro horas.
terapia antiretroviral triple se debe iniciar desde D. la gestación es a término y sin control prenatal.
las ……. semanas. E. llega en periodo expulsivo y con control
A. 10 prenatal.
B. 12
C. 14 RM 2017-A (82): Varón de 35 años, presenta
D. 11 hace 5 días en forma súbita malestar general,
E. 13 fiebre, escalofríos intermitentes cada 2 días,
concomitantemente mialgias y artralgias. Al
RM 2017-A (51): Mujer de 28 años presenta cuarto día de enfermedad exantema pruriginoso
hace cuatro días, dolor lumbar bilateral, aumento con adenopatía generalizada sin
de temperatura, disuria, polaquiuria y vómitos. hepatoesplenomegalia. Antecedente: en su
Examen: PPL (+). ¿Cuál es el diagnóstico más domicilio su hermano con dengue confirmado.
probable? ¿Cuál es la conducta contraindicada?
A. Pielonefritis aguda A. Uso de metamizol para controlar la fiebre
B. Cistitis aguda B. Reposición cuidadosa de líquidos y electrolitos
C. Uretritis aguda C. Urgente solicitud de hemograma y recuento de
D. Lumbalgia mecánica plaquetas
E. Salpingitis aguda D. Uso de paracetamol para control de la fiebre
E. Solicitar Ig M para dengue
RM 2017-A (70): ¿Cuál es el mecanismo de
acción de las sulfonamidas?

Página 6
www.plus-medica.com administrador@plus-medica.com
Manual de INFECTOLOGÍA PLUS MEDIC A
RM 2017-B (8): ¿En cuál de las siguientes
condiciones aumenta la transmisión vertical de
VIH?
A. Niveles elevados de CD4
B. RPM más de 4 horas
C. Paciente en tratamiento TARGA
D. Parto por cesárea
E. Carga viral disminuida
dolor abdominal intenso y continuo; con
RM 2017-A antecedente de haber estado en zona endémica
20. A ; 38. C ; 51. A ; 70. A ; 77. D; 82. A de dengue hace 10 días; según esto la definición
de CASO es:
RM 2017-B
A. Probable con señales de alarma
8. B B. Sospechoso
C. Probable sin señales de alarma
RM 2017-B (17): Ama de casa que al limpiar D. Grave
detrás de cuadro siente un lancetazo en mano; a E. Confirmado
las 18 horas presenta orina oscura y presencia de
una lesión livedoide en dicha zona. ¿Cuál es el RM 2017-B (86): Mujer de 35 años, presenta
diagnóstico? fiebre, escalofríos y sudoración hace 1 semana
A. Loxocelismo con periodos de remisión cada dos días. Además
B. Picadura de escorpión cefalea global, vómitos, mialgias y artralgias.
C. Bothropismo Viaje a Piura hace dos semanas. Examen: palidez
D. Latrodectismo e ictericia, FR: 20 X´. FC: 90 X´. Abdomen:
E. Picadura de abeja hepatoesplenomegalia. Laboratorio: Hb: 10 g/dl;
Plaquetas: 90,000. TGO: 250 mg/dl; Bil. Totales:
RM 2017-B (28): ¿Cuál es la articulación más 3.5 g/dl. ¿Cuál es el diagnóstico más probable?
afectada en una artritis infecciosa? A. Fiebre tifoidea
A. Cadera B. Brucelosis subaguda
B. Muñeca C. Dengue grave
C. Tobillo D. Malaria no complicada
D. Hombro E. Hepatitis granulomatosa
E. Rodilla
RM 2017-B (92): Varón de 35 años, presenta tos
RM 2017-B (42): ¿Cuál es el virus causal de esporádica con expectoración hemoptoica.
sarcoma de Kaposi en el paciente con VIH/SIDA? Antecedente de TB pulmonar hace 5 años, recibió
A. Herpes VHH tipo 6 tratamiento completo. Examen: IMC: 28; FR: 22
B. Herpes genital X’, FC: 90 X’. Tórax: vibraciones vocales
C. Herpes VHH tipo 8 aumentadas en tercio superior de HTD. BK en
D. Herpes VHH tipo 2 esputo: (-), TAC de tórax: cavidad en región
E. Citomegalovirus infraclavicular derecha conteniendo una imagen
hipodensa. ¿Cuál es el diagnóstico más probable?
RM 2017-B (60): Paciente varón de 45 años, A. Aspergiloma
que acude al establecimiento de salud por fiebre B. Adenocarcinoma
de 4 días de evolución, cefalea, artralgias, rash y C. Nódulo de Kaplan
D. Absceso piógeno
E. Hidatidosis pulmonar
Página 7
www.plus-medica.com administrador@plus-medica.com
Manual de INFECTOLOGÍA PLUS MEDIC A

RM 2017-B
17. A ; 28. E ; 42. C ; 60. A ; 86. D
RM 2017-B
92. A

2016 E. hipermotilidad uretral.

Complicaciones
( 11 preguntas )
CON CLAVE Y POR TEMAS RM 2016- I A (86): El agente que con más
frecuencia produce shock séptico, de punto de
Infecciones bacterianas partida urinario, relacionado con la gestación, es:
Artritis séptica

Etiología A. Escherichia coli


B. Pseudomonas sp
RM 2016- I A (44): ¿Cuál es el microorganismo C. Chlamydia trachomatis
más frecuente causante de la artritis séptica? D. Klebsiella sp
A. Staphylococcus aureus E. Clostridium perfringes
B. Escherichia coli
C. Hemophilus influenzae Shock séptico
D. Neisseria gonorrhoeae
E. Pseudomonas aeruginosa Manejo
RM 2016- I B (73): En paciente con diagnóstico
ITU de shock séptico. ¿Cuál es la primera medida
terapéutica a indicar?
Diagnóstico A. Administración de cloruro de sodio al 9%0
B. Infusión de dopamina
RM 2016- I A (52): Mujer de 28 años, vida C. Transfusión de sangre total
sexual activa, presenta hiporexia, fiebre, D. Administración de bicarbonato de sodio
escalofríos, disuria, polaquiuria y dolor lumbar. E. Infusión de adrenalina
Examen: febril, puño percusión lumbar bilateral
positivo. ¿Cuál es el diagnóstico probable?
A. Anexitis Metaxénicas
B. Cistitis Malaria
C. EPI
Tratamiento
D. Pielonefritis
E. Endometritis
RM 2016- I A (45):¿Cuál es el medicamento a
utilizar en la prevención de las recaídas de la
Bacteriuria asintomática en gestantes
infección por Plasmodium vivax u ovale?
A. Doxiciclina
RM 2016- I A (66): La bacteriuria
B. Cloroquina
asintomática en gestantes, predispone a
C. Quinina

D. Quinidina
A. insuficiencia renal.
E. Primaquina
B. hidronefrosis.
C. pielonefritis aguda.
D. litiasis renal.

Página 8
www.plus-medica.com administrador@plus-medica.com
Manual de INFECTOLOGÍA PLUS MEDIC A

Infección por virus Zika

Tratamiento

RM 2016- I B (23): ¿Cuál de las siguientes


patologías está relacionada a la infección por virus
del Zika durante la gestación?
A. Microcefalia
B. Agenesia renal Examen: signos meníngeos presentes. Cultivo de
C. Hidrocefalia líquido cefalorraquídeo positivo a Cryptococcus
D. Gastrosquisis neoformans. ¿Cuál es el tratamiento de inducción
E. Agenesia pulmonar para la meningitis?
A. Anfotericin B + Flucitosina
Dengue B. Trimetroprim + Sulfametoxazol
C. Ganciclovir + Corticoides
Diagnóstico D. Fluconazol + Corticoides
E. Sulfadiazina + Pirimetamina
RM 2016- I B (36): Varón de 25 años con fiebre
reciente de cinco días de evolución y antecedente
RM 2016-A
epidemiológico de exposición a dengue, refiere
artralgias, mialgias, cefalea, dolor retro-ocular, 44. A ; 52. D ; 66. C ; 86. A ; 86. D; 45. E
RM 2017-B
dolor lumbar y se evidencia erupción cutánea. ¿A
73. B ; 23. A ; 36. A ; 39. D ; 45. A
qué tipo de caso de la clasificación de dengue
corresponde?
A. Probable sin signos de alarma
B. Sospechoso con signos de alarma
C. Confirmado por laboratorio
D. Descartado
E. Grave
Parasitosis intestinal
Ascariasis

Sd de Loeffler- Diagnóstico

RM 2016- I B (39): La neumonía eosinofílica


caracterizada por infiltrados alveolares
migratorios y mínimas manifestaciones clínicas
corresponde a:
A. Neumonía eosinófila aguda
B. Granulomatosis alérgica de Churg-Strauss
C. Micosis pulmonar alérgica
D. Síndrome de Loeffler
E. Síndrome de Goodpasture

Micosis
MEC criptococósica

Tratamiento

RM 2016- I B (45): Mujer de 30 años, VIH


positivo, hace 15 días presenta fiebre y cefalea.
Página 9
www.plus-medica.com administrador@plus-medica.com
Manual de INFECTOLOGÍA PLUS MEDIC A

2015-II
( 10 preguntas )
CON CLAVE Y POR TEMAS

Parasitosis intestinal
Mecanismo de infección A.Escherichia coli
B.Estafilococo saprohyticcus
RM 2015_II-A (73): Se consideran parasitosis C.Enterococos faecalis
trasmitida desde el suelo a... D. Pseudomona aeuriginosa
A. anquilostomiasis, oxiuriasis. E. Proteus rnirabilis
B. paragonimiasis, trichuriasis.
C. áscaridiasis, trichuriasis Cólera
D. fasciolasis ,anquilostomiasis. Tratamiento
E. amebiasis, fasciolasis.
RM 2015-II B (26) : Paciente de 45 años.,
MEC 'presenta abruptamente deposiciones líquidas
TBC-Alteraciones del LCR blanquecinas frecuentes, tenesmo y dolor
abdominal precedido de vómitos. Examen:
RM 2015_II-A (83): En el líquido signos de deshidratación moderada. Es
cefalorraquídeo de pacientes con meningitis TBC. diagnosticado como infección por cólera. ¿Cuál
¿Cuál es el hallazgo más considerable para el es el tratamiento inmediato?
diagnóstico? A.Controlar el proceso infeccioso
A. Pleocitosis polimorfonucelar B.Mejorar la dieta proteica
B. Células normales por campo C.Restaurar pérdida de líquidos
C. Hiperglucorraquia D. Localizar el foco epidémico
D. Hipoproteinorraquia E. Lavado frecuente de manos
E. Pleocitosis mononuclear
Infección intrahospitalaria
Infección por VIH Mecanismo
NAC por P. jirovecci- Tto
RM 2015-II B (85) : ¿Cuál de los siguientes
RM 2015-II B (1) : En cuanto a las factores o condiciones que intervienen en las
enfermedades oportunistas en los pacientes con infecciones intrahospitalarias, está más asociado
SIDA. ¿Cuál es el tratamiento de elección en con una infección de sitio quirúrgico por
neumonía por Pneumocystis carinii? estafilococo?
A.Trimetroprim-sulfametoxazol A. Contaminación de superficies
B.Ceftazidima B. Contaminación aérea
C.Vancomicina C. Agua contaminada
D. Penicilina D. Vehículo inanimado
E. Ciprofloxacino E. Inadecuado lavado de manos

ITU RM 2015 II-A


Asociada a Urolitiasis-Etiología
73. B ; 83. E ; 66. C ; 86. A ; 86. D; 45. E
RM 2015-II B (18) : En pacientes con RM 2015 II-B
antecedente de cólicos renales frecuentes 1. A ; 18. E ; 26. C ; 85. E
asociados a litiasis renal. ¿Cuál es la bacteria que
se asocia con mayor frecuencia?

Página 10
www.plus-medica.com administrador@plus-medica.com
Manual de INFECTOLOGÍA PLUS MEDIC A

2015-I
(12 preguntas )
CON CLAVE Y POR TEMAS

MEC
Tratamiento

RM 2015 -IA (18): Varón de 35 años que presenta


bruscamente fiebre, cefalea, náuseas y vómitos. Al D.- Hepatocarcinoma complicado
examen físico: T:38.5°C, rigidez de nuca, letargo y E.- Hematoma subcapsular hepático
confusión. La tinción de gram de LCR revela
microorganismos intracelulares Gram negativos. Infecciones de la piel
¿Cuál es el tratamiento antibiótico de elección? Etiología
A.- Clindamicina 600 mg IV cada 8 horas
B.- Ceftriaxona 2 gr. IV cada 12 horas RM 2015 -IA (41): ¿Cuál de los siguientes
C.- Meropenem 1 gr. IV cada 8 horas post cultivo gérmenes es el causante de forunculosis?
D.- Ciprofloxacina 200 mg IV cada 8 horas A.- Bacillus anthracis
E.- Oxifloxacina 400 mg V cada 6 horas
B.- Bartonella bacilliformis
C.- Estafilococcus aureus
Antibióticos
D.- Clostridium perfringes
AMG-Efectos adversos
E.- Estreptococcus pyogenes
RM 2015 -IA (2): La disfunción que se
ITU
desarrolla por efecto tóxico de los Etiología
aminoglucósidos en el sistema nervioso central
se presenta a nivel de: RM 2015 -IA (79): ¿Cuál de los siguientes
A.- Coclear y olivar gérmenes es responsable del 80% de las
B.- Cerebeloso y vestibular infecciones urinarias en adultos?
C.- Vestibular y coclear A.- Enterobacter cloacae
D.- Talámico y cortical B.- Proteus mirabilis
E.- Hipotalámico y talámico C.- Pseudomona aeroginosa
D.- Escherichia coli
Absceso piógenos
E.- Klebsiella pneumoneae
Diagnóstico
Colitis pseudomembranosa
RM 2015 -IA (12): Varón de 65 años que acude Tratameinto
a Emergencia, porque hace 6 horas presenta alza
térmica, escalofríos, ictericia y dolor espontáneo RM 2015 -IA (85): ¿Cuál de los antibióticos es
en cuadrante superior derecho del abdomen. más eficaz para el tratamiento de infecciones
Posteriormente tos seca. Antecedente de intestinales causadas por Clostridium difficile?:
Diabetes y colelitiasis. RX Abdomen: A.- Vancomicina
hemidiafragma derecho elevado. Examen físico: B.- Bacitracina
dolor a la palpación en el hipocondrio derecho, C.- Acido Fusídico
no se palpan masas. ¿Cuál es el diagnóstico más D.- Eritromicina
probable? E.- Nitazoxanida
A.- Absceso piógeno
B.- Quiste hidatídico complicado
C.- Absceso amebiano ¡Talento a tu servicio!
www.plus-medica.com
administrador@plus-medica.com

Página 11
www.plus-medica.com administrador@plus-medica.com
Manual de INFECTOLOGÍA PLUS MEDIC A

EDA
Complicaciones

RM 2015 -IA (86): Varón de 65 años, con T: 38


°C , náuseas, vómitos y deposiciones líquidas
con moco, sangre y tenesmo. Estando
hospitalizado presenta una convulsión por
primera vez. ¿Cuál es el agente causal? Infección por Streptococo
A.- Entamoeba histolítica Complicaciones
B.- Salmonella
C.- Escherichia colie RM 2015 –I B (17): La complicación más severa
D.- Clostridiun perfingens de la infección por estreptococo beta hemolítico
E.- Shiguella es:
A.- Artritis séptica
2015-I A B.- Síndrome urémico hemolítico
18: B ; 2: C ; 12: A ; 41: C ; 79: D C.- Miocarditis
85: A ; 86: E D.- Osteomelitis
E.- Fiebre reumática
Sd mononucleçosico ITU
Mononucleosis infecciosa Factores de riesgo
RM 2015 -IA (91): Mujer de 26 años, con fiebre RM 2015 –I B (51): La Pielonefritis
de 39°C, de 4 semanas de evolución que calma Enfisematosa se presenta con mayor frecuencia
con antipiréticos; fatiga intensa y malestar en pacientes con:
general. Al Examen físico: Linfoade- nomopatías A.- Litiasis renal
cervicales, submaxilares y supraclaviculares y B.- Desnutriciòn crònica
hepatoesplenomegalia. Hemograma con 15% de C.- Diabetes mellitus II
linfocitos atípicos y leve trombocitopenia. ¿Cuál D.- Insuficiencia renal crònica
es el diagnóstico más probable? E.- Sindrome metabolico
A.- Dengue
B.- Fiebre tifoidea RM 2015 –I B (69): ¿Cuál de los siguientes
C.- Fiebre Chikungunya antibióticos no requiere ajuste de dosis en
D.- Mononucleosis infecciosa pacientes con insuficiencia renal severa?
E.- Tuberculosis pulmonar A.- Ticarcilina
B.- Azitromicina
Chancroide
C.- Amikacina
Diagnóstico
D.- Ceftazidima
RM 2015 –I B (4): Varón de 20 años, que 7 dias E.- Levofloxacino
después de una relación sexual presenta en el
pene múltiples úlceras, de diámetro variable, 2015-I B
bordes irregulares, sangran fácilmente, blandas 91: D ; 4: E ; 17: E ; 51: C ; 69: B
y dolorosas a la palpación. ¿Cuál el diagnóstico
más probable?
A.- Sífilis
B.- Donovanosis
C.- Herpes
D.- Linfogranuloma venéreo
E.- Chancroide

Página 12
www.plus-medica.com administrador@plus-medica.com
Manual de INFECTOLOGÍA PLUS MEDIC A

2014-II
7 preguntas
CON CLAVE Y POR TEMAS
Cólera
MEC-VIH
Cólera
Factores de riesgo
RM 2014 II –B (24): Varón de 45 años, llega a
RM 2014 II –A (25): ¿Cuál es la causa
Emergencia por presentar abruptamente
frecuente de infecciones secundarias del
deposiciones líquidas blanquecinas frecuentes,
sistema nervioso central, en pacientes con
tenesmo y dolor abdominal precedido de
SIDA?
vómitos. Examen fisico: afebril, signos de
A. Candidiasis cerebral
deshidratación moderada. El diagnóstico co-
B. Linfoma primario
al Cólera. ¿Cuál es el tratamiento inmediato a
C. Meningitis criptocócica seguir?
D. Toxoplasmosis cerebral A. Restaurar las pérdidas de líquidos
E. Encefalopatía viral B. Utilizar racecadotrilo
C. Controlar el proceso infeccioso
Absceso hepáticor D. Corregir la acidosis metabólica
Características E. Utilizar loperamida
RM 2014 II –A (39): ¿Cuál de las siguientes MEC
características macroscópicas tiene el Tratamiento
contenido de un absceso hepático amebiano?
A. Verdoso RM 2014 II –B (37): Mujer de 25 años con
B. Cremoso diagnóstico de meningoencefalitis hospitalizada
C. Seroso en emergencia. Se inicia tratamiento con
D. Pardo rojizo Penicilina G sódica 2 millones c/4 horas EV
E. Cristal de roca durante 48 horas sin presentar mejoría. El cultivo
revela Neisseria meningitidis ¿Cuál es el
ITS antibiótico más adecuado?
Chancroide A. Cefuroxima
B. Ampicilina/ sulbactam
RM 2014 II –A (63): ¿Cuál es el agente C. Cefoxitima
etiológico del Chancroide (chancro blando)? D. Ceftriaxona
A. Haemophilus ducreyi E. Aztreonam
B. Corynebacterium urealyticum
C. Treponema pallidum Uretritis
D. Chlamydia trachomatis Etiología
E. Haemophilus vaginalis
RM 2014 II –B (42): ¿Cuál es el agente causal
de uretritis no gonocócica transmitida
¡Talento a tu servicio! sexualmente?
A. Bacilos gram negativos
www.plus-medica.com B. Adenovirus
administrador@plus-medica.com C. Chlamydia trachomatis
D. Neisseria meningitidis
E. Microsporidios

Página 13
www.plus-medica.com administrador@plus-medica.com
Manual de INFECTOLOGÍA PLUS MEDIC A

Infección por VIH

Infecciones oportunistas con CD$ > 300 /ul

RM 2014 II-B (19): ¿ Cuál es la Infección


oportunista en SIDA que cursa con CD4 mayor
de 300 /uL al momento del diagnóstico?

A.Neumonía por pneumocystls


B.Toxoplasmosis
C.Criptococosis
D.Retlnitls por cltomegalovirus
E.Tuberculosis

2014 II-A
25: D ; 39: D ; 24: A ; 42: C ; 19: D
85: A ; 86: E
2014 II-B
24: A ; 42: C ; 19: E

Página 14
www.plus-medica.com administrador@plus-medica.com
Manual de INFECTOLOGÍA PLUS MEDIC A

2014-I
13 preguntas
CON CLAVE Y POR TEMAS

Sarcoma de Kaposi

Características clínicas
RM 2014 I-A (80) : Paciente con diagnóstico
RM 2014 I-A (23): ¿Cuál es la caracteristlca que de Pielonefritis Aguda ¿Cuáles son los hallazgos
corresponde al sarcomá de Kaposi?: en el examen de orina?
A. Leucocituria y hematuria
A.Las lesiones son dolorosas y pruriginosas B. Proteinuria y hematuria
B.Es una neoplasia maligna C. Cilindros granulosos y hematuria
C. No invaden la cavidad oral D. Cilindros leucocitarios y hemáticos
E. Lipiduria y bacteriuria
D.Es infrecuente en el tracto digestivo
E.La etiología es el VIH Patogenicidad
Reservorio
RM 2014 I-A (81) : La capacidad de un agente
RM 2014 I-A (49): . El hábitat en el cual un infeccioso de producir enfermedad en una
agente infeccioso vive, crece y se multiplica se persona infectada, se denomina:
denomina: A. Infectividad
A. Reservorio B. Inmunogenicidad
B. Vehículo C. Patogenicidad
C. Huésped D. Virulencia
D. Fuente E. Antigenicidad
E. Vector
Botulismo
Loxocelismo Fisiopatología
Diagnóstico
RM 2014 I-A (92) :¿Cuál de las siguientes
RM 2014 I-A (72): Ama de casa que al entidades se presenta por déficit de acetilcolina
limpiar detrás de cuadros siente un lancetazo en la unión neuromuscular de las terminaciones
en mano. A las 36 horas presenta hematuria y la neurales?
presencia de una placa azulado violáceo (placa A. Polimiositis
livedoide). ¿Cuál es el diagnóstico más probable? B. Miastenia Gravis
A. Latrodectismo C. Botulismo
B. Picadura de escorpión D. Parálisis periódica hiperkalémica
C. Bohotropismo E. Tétanos
D. Loxocelismo
E. Picadura de abeja Infección

PNA RM 2014 I-A (94) : ¿Cuál es la vía de


Diagnóstico diseminación infecciosa más importante y difícil
de controlar?
¡Talento a tu servicio! A. El tracto respiratorio
www.plus-medica.com B. El tracto digestivo
administrador@plus-medica.com C. El tracto urinario

Página 15
www.plus-medica.com administrador@plus-medica.com
Manual de INFECTOLOGÍA PLUS MEDIC A

2014 I-A
23: B ; 49: A ; 71: D ; 80: A ; 81: C
92: C ; 94 : A

TBCi
Tratamiento-Corticoides

RM 2014 I-B (5) : ¿En cuál de las siguientes


D. Staphylococcus epidermidis
patologías se agrega corticoides al tratamiento
específico? E. Streptococcus pyogenes
A. Meningitis tuberculosa
Malaria
B. Tuberculosis miliar
Indice parasitario anual
C. Tuberculosis primaria
D. Pleuritis tuberculosa RM 2014 I-B (53) : En casos de malaria, el
E. Tuberculosis peritoneal índice parasitario anual (IPA) relaciona el número
de casos ……….. sobre la población …………
Giardiasis A. confirmados / en riesgo.
Efectos secundarios-Sd pseudoulceroso B. probables / en riesgo.
C. confirmados / de casos probables.
RM 2014 I-B (11) : ¿Cuál de las siguientes
D. probables / de casos confirmados.
patologías produce síndrome pseudo ulceroso?
E. confirmados / del total de casos.
A. Fiebre tifoidea
B. Amebiasis Giardiasis
C. Giardiasis Efectos
D. Shigellosis
E. Cólera RM 2014 I-B (74) : Varón de 30 años,
procedente de zona endémica de malaria que
Infección por anaerobios presenta fiebre cuartana. ¿Cuál es la especie de
ITU Plasmodium más probable?
A. Difícile
RM 2014 I-B (15) : ¿En cuál de las infecciones, B. Malarie
los gérmenes anaerobios se encuentra con C. Ovale
menos frecuencia? D. Falcíparum
A. Infecciones urinarias E. Vívax
B. Maxilofaciales
C. Tracto genital femenino 2014 I-B
D. Abscesos uretrales 9: A ; 11: C ; 15: A ; 28: B ; 53: A
E. Partes blandas profundas 74: B

Enfermedad por arañazo de gatoi


Etiología

RM 2014 I-B (28) : ¿Cuál es el agente


etiológico de la enfermedad por arañazo de gato?
A. Bordetella pertusis
B. Bartonella henselae
C. Chlamydia trachomatis

Página 16
www.plus-medica.com administrador@plus-medica.com
Manual de INFECTOLOGÍA PLUS MEDIC A

2013
12 preguntas
CON CLAVE Y POR TEMAS

Fasceitis necrotizante
Tratamiento
D. Ancytostoma duodenale
RM 2013-A (42): ¿Cuál es el tratamiento E. Ascarís lumbrícoides
esencial de la fasceitis necrotizante?
A. Desbridamiento quirúrgico amplio Loxocelismo
B. Antibioticoterapia Veneno.composición
C. Soporte hemodinámico
D. Curaciones diarias RM 2013-A (92): ¿Cuál es la enzima
E. Antibioticoterapia más curaciones dermonecrótica más importante el veneno de la
Loxoceles laeta?
Malaria A. Esfingomieiinasa B
Etiología B. Hidrolasa
C. Hiaiuronidasa
RM 2013-A (46): Varón de 24 años, que D. Lipasa –A
procede de la selva con 10 días de enfermedad E. Colagenasa
caracterizada por alza térmica cada dos días,
cefalea, hipersomnia, palidez e ictericia. Si la ITU
sospecha es paludismo el agente causal es el Urolitiasis-Etiología
Plasmodiun………….
A. Ovale RM 2013-B (19): ¿Cuál es el agente etiológico
B. Vivax que se encuentra con mayor frecuencia en la
C. Malariae infección urinaria con litiasis coraliforme?
D. Knowlesi A. Klebsiella spp
E. Falciparum B. Proteus mirabilis
C. E. coli
Denguen D.Staphylococcus saprophyticus
Serotipos E.Enterococcus faecalís

RM 2013-A (50): Los serotipos del dengue, se MEC


reconocen por la variación de la proteína: Alteración del LCR
A. C2 RM 2013-B (36): En la meningitis bacteriana
B. M4 ¿Cuál es ¡a alteración típica del LCR?
C. E4 A.Disminución de ¡a glucosa en LCR < 80 m/dL
D. C3 B.Leucxiíosispolimoribnucieares>10cé¡ulas/pL
E. prM C.Disminución de la concentración de glucosa en
LCR < 40 mg/dL
Nemátodos D. Disminución de concentración de proteínas
Hábitat <0,45 g/L
E. Aumento de la presión de apertura >80 mm
RM 2013-A (84): ¿Cuál de los siguientes
nemátodos durante e! proceso de su patogenia,
se iocaliza en el intestino grueso?
¡Talento a tu servicio!
A.Strongyloides stercoralis www.plus-medica.com
B.Necator mericanus administrador@plus-medica.com
C.Trichuris tríchura

Página 17
www.plus-medica.com administrador@plus-medica.com
Manual de INFECTOLOGÍA PLUS MEDIC A

Celulitis orbitaria
Complicaciones

RM 2013-B (46): Varón de 30 años con


celulitis orbitaria del ojo izquierdo de un día de
evolución, es hospitalizado iniciándose
tratamiento antibiótico. A las 24 horas
presenta ceguera del ojo contralaíeral. ¿Cuál es
el diagnóstico? a) Hipertensión arterial
A.Ruptura de aneurisma cerebral b) Dolor torácico opresivo
B.Oftaimitis c) Abdomen en tabla
C.Exoftalmos maligno d) Daiforesis marcada
D.Neoplasia ocular infectada e) Hemólisis
E.Trombosis de! seno cavernoso
Artritis séptica
2013-A Etiología
42: A ; 46: E ; 50: C ; 84: C ; 92: A
19: B; 36: C ; 46: E RM 2013-B (67): ¿Cuál es el microorganismo
causal más frecuente de la artritis séptica del
adulto?
Leptospirosis A.Escheríchía coli
Diagnóstico B.Streptococois pyogenes
C.Neissetia gonorrhoeae
RM 2013-B (53): Varón de 30 años gasfitero. D.Staphylococcus aureus
Hace 18 días refiere malestar general, alza E. Pseudomonas aeniginosa
térmica, cefalea intensa y dolores
osteomusculares a predominio de pantorrillas. A 2013-B
la semana presenta ictericia, oliguria, 53: C ; 60: C ; 57: E ; 67: D
compromiso dei sensorio y signos meníngeos.
¿Cuál es el diagnóstico más probable?
A.Fiebre amarilla
B.Tifoidea
C.Leptospirosis
D.Hepatitis B
E.Malaria

ITS
Diagnóstico por laboratorio

RM 2013-B (60):¿Cuál de las siguientes pruebas


de laboratorio para sífilis es treponémica?
A. VDRL
B. Khan
C. FTA
D. Mazzini
E. RPR

Loxocelismo
Veneno-efectos

RM 2013-B (57):¿Cuál es el efecto que produce


el veneno de Loxoceles laeta?
Página 18
www.plus-medica.com administrador@plus-medica.com
Manual de INFECTOLOGÍA PLUS MEDIC A

2012
preguntas
CON CLAVE Y POR TEMAS

Sífilis
Tratamiento
B.Diarrea
RM 2012-A (4): En la neurosífilis el tratamiento C.Náuseas y vómitos
de elección es: D.Ninguna sintomatología
A.Penicilina benzatinica 2.4 millones IM E.Diarrea
semanal x 3 semanas
B.Penicilina procaínica 1 '000,000 IM diario por Infección bacteriana
10 días Diálisis peritoneal
C.Penicilina G acuosa 3 - 4 millones EV cada 4
horas por 14 días RM 2012-A (75): ¿Cuál es el agente causal más
D.Ceftriaxona 250 mg EV cada 12 horas x 10 días frecuente de peritonitis en pacientes sometidos a
E.Penicilina benzatinica 7.2 millones IM semanal x diálisis peritoneal?
3 semanas A.Candida albicans
B.Escherichia colli
ITU C.Streptococcus B hemolítico
Etiología D.Staphilococcus coagulasa negativa
E. Klebsiellasp
RM 2012 –A (8): En la mujer adulta, ¿cuál es el
agente patógeno más frecuente causante de 2012-A
infección urinaria esporádica no complicada 4: C ; 8: A ; 19: D ; 65: D ; 75: B
adquirida en la comunidad?
A.Escherichia coli
B.Proteussp
C.Enterobacier sp Infección viral
D.Klebsiella pneumoniae Etiología
E.Pseudomona aeruginosa
RM 2012-B (10): ¿Cuál es el agente causal más
ITS frecuente del resfrío común?
A.Virus sincitial respiratorio
Diagnóstico
B.Virus Coxsackie
RM 2012-A (19): ¿Cuál de las siguientes C.Rinovirus
pruebas hace el diagnóstico de Haemophilus D. Adenovirus
ducreyi chancroide? E. Parvovirus
A.Tinción de Gram del exudado
B.Medio de cultivo de Tayer Martin Dengue
C.Examen en campo oscuro Manejo
D.Estudio en fresco
E. Prueba serológica RM 2012-B (26): Según la Guía Técnica del
MINSA para el manejo del Dengue - grupo B,
Parasitosis intestinal ¿qué acción sanitaria corresponde?
Diagnóstico A.Manejo ambulatorio
B.Referirlo para manejo hospitalario
RM 2012-A (65): Comprobada una ingesta de C.Tratamiento de emergencia
carne de cerdo poco cocida contaminada con D.Tratamiento higiénico dietético
cisticercos. ¿Qué debe esperarse que ocurra en E.Administración de medicación vía IM
los primeros 2 meses?
A.Dolor abdominal

Página 19
www.plus-medica.com administrador@plus-medica.com
Manual de INFECTOLOGÍA PLUS MEDIC A
2012-B
10: C ; 26: B

2011
CON CLAVE Y POR TEMAS

ITS
c. Meníngea
Sífilis
Tratamiento d. Intraventricular
e. Bulbar
RM 2011-A (13): Agente etiológico de la
sífilis: Hidatidosis
a. Treponema pallidum Localización
b. Neisseria gonorreae
RM 2011-A (70): Ubicación más frecuente de
c. Haemophilus ducreyi
quiste hidatídico en abdomen:
d. Chlamydea
a. Páncreas
e. Gardennella
b. Bazo
c. Riñón
INFECCIONES BACTERIANAS
d. Hígado
ITU
Tratamiento e. Mesenterio

RM 2011-A (36): Tratamiento de ITU en Giardiasis intestinal


gestante en 3er trimestre. Tratamiento
a. Cefalosporina
RM 2011-A (39): El tratamiento de la diarrea
b. Nitrofurantoina
aguda por Giardia es:
c. Sulfa
a. Metronidazol
d. Penicilina
b. Cotrimoxazol
e. Metronidazol
c. Eritromicina
d. Nitrofurantoína
METAXÉNICAS
e. Ceftriaxona
ITU
Tratamiento
INFECCIÓN por VIH
RM 2011-A (9): Síntomas de mal pronóstico Diagnóstico
RM 2011-B (75): ¿Cuál es la prueba de
en la enfermedad por Plasmodium Falciparum
confirmación para VIH – SIDA?::
(paludismo):
a. Western bloot
a. Dolor y anuria
b. PCR
b. Diarrea y fiebre
c. ELISA para VIH
c. Anuria y convulsiones
d. Inmunofluorescencia
d. Fiebre y escalofríos
e.Cultivo
e. Dolor abdominal

PARASITOSIS intestinal
Neurocistecercosis ¡Talento a tu servicio!
Tratamiento
RM 2011-A (28): Neurocisticercosis quística
www.plus-medica.com
tratada con ALBENDAZOL en cual de ellas administrador@plus-medica.com
tiene más efectividad este tratamiento:
a. Racémica
b. Parenquimatosa

Página 20
www.plus-medica.com administrador@plus-medica.com
Manual de INFECTOLOGÍA PLUS MEDIC A

Infecciones oportunistas

RM 2011-A (37): Paciente joven, con


antecedente de conducta sexual de riesgo,
presenta tos sin expectoración, fiebre,
crepitantes e infiltrados en Rx tórax .¿ cuál es el
agente causal más probable?.
a. S. Pneumoniae
b. N. Meningitidis
c. H. Influenzae e. S. aureus
d. Pseudomona aeruginosa
e. Pneumocystis jirovecii 2011-A
49: A ;
2011-A 2011-A
13: A ; 36: B ; 9 : C ; 28 : B ; 39 : A 87 : C ; 47 : B
2011-A
75: A ; 37 : E

ANTIBIÓTICOS
Contraindicación en la gestación

RM 2011-A (49): ¿Qué antibióticos están


contraindicados en el primer trimestre de la
gestación?
a. Metronidazol
b. Cefalosporina
c. Ampicilina
d. Nitrofurantoína
e. Amoxicilina

Mecanismo de acción
RM 2011-B (87): Antibiótico que actúa
uniéndose a topoisomerasas:
a.Cefalosporinas
b.Penicilinas,
c.Quinolonas
d.LIpopéptidos
e.Carbapenems

MEC
Tratamiento

RM 2011-B (47): ¿Cual es la causa más


frecuente de MEC en adultos jóvenes?:
a. S. pneumoniae
b. Neiseria meningitidis
c. H. influenzae
d.Listeria monocitógenes

Página 21
www.plus-medica.com administrador@plus-medica.com
Manual de INFECTOLOGÍA PLUS MEDIC A

MICROBIOLOGÍA PLUS MEDIC A


BACTERIAS Las bacterias pueden mantenerse unidas unas
con otras después de la división celular, pero
conservando siempre la independencia celular.
Introducción
Diplococos: Si el plano de división es único,
◆Las bacterias pertenecen al reino podemos encontrar diplococos o cocos en cadena
Procaryotae (pro: primitivo y cariota : núcleo). (microorganismos del género Streptococcus).
◆Son elementos unicelulares sin un núcleo Cocos en racimo: Si los planos de división son
verdadero. muchos, los cocos pueden agruparse en tétradas
◆Su tamaño aproximado es de 1-3 micras. o en racimos (Staphylococcus).

Θ Las bacterias se reproducen por fisión binaria. Bacilos: ◎ Los bacilos pueden ser muy cortos
Θ La mayoría son de vida libre, a excepción de
(cocobacilos) o muy largos. ◎ Sus extremos
algunas que son de vida intracelular (IC) obligada
pueden ser redondeados o rectos; ◎ pueden
como Chlamydias y Ricketsias.
estar aislados, en cadenas, en filamentos o
Procariotas
formando letras chinas (Corynebacterium).
√ No poseen compartimientos intracelulares
Bacilos curvos: Los bacilos curvos pueden
delimitados por membranas por lo que carecen de
membrana nuclear a diferencia de eucariotas. tener forma de coma (Vibrio cholerae).
√ Su ADN es circular y cerrado.

Clasificación
La coloración Gram permite dividir a las
bacterias en dos grandes grupos:
GRAM positivos (color violeta)
GRAM negativos (color rojo)

Gram (+) Gram (-)


(Neumococo) Haemophylus influenzae

La forma de las bacterias al microscopio está Morfología:


determinada por la rigidez de su pared celular. 1. cocos; 2. diplococo; 3. cocos en cadenas; 4. cocos
en racimos; 5. cocos en tetradas; 6. cocobacilos;
Básicamente, se diferencian según su forma en: 7. bacilos; 8. bacilos bordes redondeados; 9. bacilos
bordes rectos; 10. bacilos fusiformes; 11, 12. bacilos
◆ Cocos (esféricas u ovaladas) curvos; 13 al 15. espiroquetas
◆ Bacilos (cilíndrica o de bastones; rectos o
curvos) ¿Cómo se realiza la coloración Gram?
◆ Espirilos (espiral); dentro de estas últimas  Se aplica Cristal violeta (colorante catiónico)
se encuentran: Treponema, Borrelia y Leptos que penetra en todas las células bacterianas
-pira. (tanto Gram positivas como Gram negativas) a través de
Las espirilos varían en el número de vueltas, desde pocas la pared bacteriana.
(Borrelia) a muchas (Treponema).
22
Manual de INFECTOLOGÍA PLUS MEDIC A

 Luego se aplica alcohol-acetona que


sirve para realizar la decoloración.
 Colorante safranina
Los organismos Gram positivos no se decoloran,
mientras que los Gram negativos sí lo hacen.
Para poner de manifiesto las células Gram negativas
se utiliza una coloración de contraste de color rojo
como la safranina o la fucsina.

Elementos facultativos

Cápsula

Estructura polisacárida de envoltura.


Coloración Gram : pasos Es el factor de virulencia de la bacteria.
Protege a la bacteria de la fagocitosis y
facilita la invasión.
Elementos bacterianos Θ Permite la diferenciación en tipos sero-
lógicos.
Los elementos bacterianos se dividen en:
Flagelos
Elementos obligados:
Estructuras proteicas, de mayor longitud
√ Pared bacteriana. que los pilis.
√ Membrana citoplasmática. Θ De estructura helicoidal
√ Citoplasma. Θ De actividad locomotora (responsables de la
√ Ribosomas. motilidad bacteriana).
√ Nucloide (Nucleoide) o cromosoma
bacteriano.
Según la posición de los flagelos tenemos
bacterias:
Elementos facultativos: ◆Monotricas: un flagelo en un extremo o ambos.
◆Logotricas: varios flagelos en un extremo o
√ Cápsula ambos.
√ Flagelos ◆Peritricas: flagelos en toda la superficie.
√ Fimbrias o pili
√ Esporo Pilis o fimbrias
√ Glicocalix
√ Plasmidos Θ Son estructuras cortas parecidas a pelos
√ Transposones que son visibles sólo al microscopio electrónico.
Θ Carentes de motilidad.
Θ Los poseen fundamentalmente las gram-
negativos.
23
Manual de INFECTOLOGÍA PLUS MEDIC A

Θ Intervienen en la adherencia de las bac- La pared puede proteger a la célula de


terias al huésped. las sustancias tóxicas y es el sitio de acción
de algunos antibióticos.
Θ Facilitan el intercambio de ADN du-
rante la conjución bacteriana. MIR 05-06 : ¿ Cuál de los siguientes tipos de
Θ Tiene capacidad antigénica microorganismos NO es un parásito
intracelular obligado y puede crecer en medios
Glicocálix de cultivo artificiales?:
a) Chlammydia.
Entramado de fibrillas polisacári - b) Mycoplasma.
das situadas en posición extracelular. c) Coxiella.
Θ Facilita la adherencia d) Adenovirus.
e) Rickettsia.
Plasmidios
Pared de una célula grampositiva
Los lásmidos (plasmidios) son elementos ◎ Está formada por una única capa homo-
extracromosómicos compuestos por ADN génea de 20 a 80 nm de grosor de
de doble cadena. peptidoglicano o mureína, situada por fuera de
la membrana celular.
Pared de la célula gramnegativa
Θ Con frecuencia circular, autoreplica-
◎ Es más compleja; posee una capa de 2 a 7
tivos y autotransferibles.
nm de grosor de peptidoglicano rodeada por
Transposones una membrana externa.

Pared de los Gram (+)


Son elementos compuestos de
ADN (genes saltarines o móviles) que La gruesa pared celular de las bacterias
pueden moverse de forma autosuficiente a grampositivas está constituida principalmente
diferentes partes del genoma bacteriano.
por peptidoglicano.
Se cree que ésta gruesa capa de peptidoglicano es
Θ No poseen la capacidad de la determinante de que estas bacterias retengan
autoreplicarse pero pueden el cristal violeta de la coloración de Gram.
transferirse a través de plasmidios.

Elementos obligados

Pared bacteriana

Ubicada por fuera de la membrana


plasmática, es una estructura vital para las
bacterias que la poseen.

Θ Los fármacos que bloquean su


formación producen la lisis y muerte de
las bacterias susceptibles.

Excepto los mycoplasmas todas las


bacterias tienen una pared celular que
les da forma y las protege de la lisis
osmótica.
La pared celular de muchos microorganismos
patógenos tiene componentes que contribuyen a su pato-
genicidad. Pared de los Gram (+)
24
Manual de INFECTOLOGÍA PLUS MEDIC A

Ácido teicoico y lipoteicoico Pared de los Gram (-)


Las bacterias gram (+) contienen una gran es mucha más elaborada y compleja .
cantidad de ácido teicoico: polisacáridos Está constituída por tres zonas:
que se unen al ácido N-acetilmurámico (NAM) √ Membrana plasmática ,
o a los lípidos de la membrana plasmática, en √ Espacio periplásmico (que incluye una
este último caso se denomina ácido fina capa de peptidoglicano)
lipoteicoico. √ Membrana externa .
ACIDO TEICOICO: son polímeros de glicerol
que están unidos al peptidoglicano
mediante enlaces covalentes y le dan a la
pared bacteriana una carga (-)
FUNCIÓN: Facilita la adherencia a las
células del huésped

Los ácidos teicoicos tienen un rol en la


virulencia de estos microorganismos, porque
actúan como antígenos de superficie que se
unen a receptores específicos en las células del
huésped.
ACIDO LIPOTEICOICO: ácido teicoico con
lípidos. Es anfifílica o anfipática
FUNCIONES:
√ Fijan la pared a la membrana citoplasmática
√ Favorecen la adherencia a las células
√ Estimula respuestas inflamatorias severas. Espacio periplásmico

Es el espacio entre la membrana


plasmática y la externa de las bacterias
gramnegativas y está ocupado por un gel, el
periplasma.

El ácido lipoteicoico estimula las células del


sistema inmune innato ,las que secretan citosinas
inflamatorias.

25
Manual de INFECTOLOGÍA PLUS MEDIC A

El espacio periplásmico de las bacterias El LPS está constituido por tres partes:
gramnegativas contiene : ◆ Lípido A
◆ Polisacárido central o del core
PROTEÍNAS: que participan en la captación de ◆ Cadena lateral O.
nutrientes. Ejemplos:
◆Enzimas hidrolíticas (proteasas, lipasas, Lipopolisacáridos (LPS)
fosfatasas, β-lactamasas). Es un glucolípido complejo.
Estas enzimas convierten las macromoléculas Tiene 3 regiones:
en productos más pequeños que pueden ser √ Antígeno somático O (LOS): polisacárido
metabolizados por la bacteria. repetitivo
◆Enzimas que participan en la síntesis del √ Core: oligosacárido medular
peptidoglicano y en la modificación de √ Lípido A: disacárido difosfato
compuestos tóxicos que podrían lesionar la
célula.

En especies patógenas, también encontramos a


ese nivel factores de virulencia como
colagenasas, hialuronidasas y proteasas.

Las bacterias grampositivas no tienen un espacio


periplásmico visible y secretan enzimas
denominadas exoenzimas, que corresponderían
a las periplásmicas de las bacterias
gramnegativas.

OF: Una de las siguientes afirmaciones referidas


al lipopolisacárido (LPS) de la membrana externa
de la pared celular de las bacterias gram
negativas es cierta:
a) Es una toxina termolábil que no resiste la
esterilización en autoclave.
b) Tiene una actividad endotóxica que está
asociada con el lípido A.
c) Contiene el antígeno O que es esencial para la
viabilidad celular. El lípido A está inmerso en la
d) Su concentración en sangre no está membrana externa y el resto del LPS
directamente relacionada con la mortalidad por sobresale de la superficie celular.
shock irreversible y colapso cardiovascular.
Rpta. B El core está unido al lípido A.

Membrana externa Cadena O o Ag O (LOS)

Es exclusiva de las bacterias gramnegativas, Consiste en unidades repetidas de una subuni-


es una bicapa lipídica que difiere de otras dad tetrasacárida y es muy variable en su compo-
membranas po r su capa externa, que está sición entre las diferentes familias, especies y aún
constituida por una molécula anfipática: dentro de la misma especie de bacterias gramnega-
el lipopolisacárido (LPS) o endotoxina. tivas.
Θ Además del LPS, la membrana externa Θ El polisacárido del core es constante
contiene fosfolípidos y proteínas que la para un mismo género bacteriano.
unen al peptidoglicano.

26
Manual de INFECTOLOGÍA PLUS MEDIC A

PARA RECORDAR:

El ácido teicoico y lipoteicoico son exclusivos de


la pared de las BGP y la membrana externa,
lipopolisacáridos y porinas son exclusivos de las
BGN.

El polisacárido O por su variabilidad es usado


frecuentemente para la clasificación
serológica de las bacterias.
Peptidoglicanos
Porinas
El peptidoglicano es una macromolécula gigante
Son canales acuosos que se encuentran en la responsable de la rigidez de la bacteria y
membrana externa de las BGN y restringen el paso proporciona resistencia frente a la lisis osmótica.
de determinados antibióticos.
Permiten la difusión de moléculas <700 Espesor
dalton e hidrosolubles -En las bacterias grampositivas es una capa sólida
de 50-100 moléculas de peptidoglicanos.
-En las bacterias gramnegativas tiene un espesor
de solo una o dos moléculas.

Elementos del peptidoglicano


Unidad disacarídica repetitiva (NAG-NAM)
Formada por monosicáridos
√ NAG : N-acetilglucosamina
√ NAM : N-acetilmurámico

SINTESIS del
peptidoglicano
Consta de varias fases:

Síntesis de precursores en el citoplasma


Ensamblaje parcial en membrana
Transporte a la cara externa externa de la
membrana
Ensamblaje final en el exterior, mediante
reacciones que no precisan energía

27
Manual de INFECTOLOGÍA PLUS MEDIC A

1 º SÍNTESIS de precursores solubles Aminoácidos que se adicionan al NAM:


en el citoplasma: se sintetizan los √ L-ala nina
monosacáridos NAM y NAG y las unidades √ D-glu támico
disacáridas (NAM-NAG). √ m-DAP (meso-Diaminopimélico) (u otro diamino-
ácido. p. ej. L-lys en Staphylococcus aureus)
Se sintetizan por separado los √ D-alanil-D-alanina
monosacáridos:
© NAM : N-acetilmurámico Unión del aminoácido terminal del
© NAG : N-acetilglucosamina NAM-pentapéptido: D-alanil-alanina

Los monosacáridos se activan al unirse a El último paso de adición de aminoácidos es


uridín difosfato (UDP). Se forman los NAM- la unión del dipéptido D-alanil-D-alanina, que
UDP y NAG-UDP. se ha sintetizado en dos fases:

Formación de la unidad Θ La enzima D-alanina racemasa convierte la


disacarídica L-alanina en D-alanina.

Los monosacáridos NAM y NAG forman la Θ La enzima D-alanil-D-alanina sintetasa


unidad disacárida repetitiva del genera un enlace peptídico entre dos
esqueleto del peptidoglucano (NAM-NAG) D-alanina.

2ºENSAMBLAJE PARCIAL
en la membrana:
un grupo de aminoácidos conocido como pen -
tapéptido se une al NAM-UDP formándose el
NAM-Pentapéptido.

Formación del NAM-pentapéptido

Luego se va produciendo adición


la
secuencial y ordenada de los distintos
aminoácidos al NAM-UDP (en reacciones que La enzima D-alanina racemasa
requieren energía e iones Mn++) formándose convierte la L-alanina en D-alanina. La
el NAM- pentapéptido. D-alanil-D-alanina sintetasa genera un
enlace peptídico entre 2 D-alanina.

28
Manual de INFECTOLOGÍA PLUS MEDIC A

3 º TRANSPORTE de precursores ¿ Qué es el undecaprenil-fosfato


solubles en el citoplasma: el UDP- (bactoprenol)?
NAM/pentapéptido se transporta a la
superficie externa de la membrana donde se Es un poliisoprenoide de 55 átomos de C
une a un trasportador de membrana
(bacteprenol o Undecaprenil-fosfato). Permite el transporte y ensamblaje de sustancias
que, como los azúcares, son hidrofílicas, y no
Transporte a la superficie podrían pasar por sí mismas la barrera
externa de la membrana hidrofóbica de la membrana.

◆ El UDP-NAM-pentapéptido se TRANSGLUCOSILACIÓN
transporta a la superficie externa de la
membrana y se une a un transportador de Formación del Peptidoglicano lineal mediante la
membrana, llamado undecaprenil-fosfato (que polimerización de varias unidades disacarí -
abreviaremos como Lip-P o BPP y que es dicas del precursor Lip-P-P-NAM/pentapéptido)-NAG.
conocido como bactoprenol), en una reacción
catalizada por una translocasa específica. Θ El bactoprenol “ se da la vuelta ” en
la membrana (una especie de flip-flop desde la capa
◆Una vez que el NAM/pentapétido está unido interna hasta la externa), de modo que logra que el
al bactoprenol (por medio de un pirofosfato), precursor NAM / pentapéptido - NAG - BPP
una transferasa transfiere a éste la NAG desde quede expuesto hacia el medio acuoso exterior a
el UDP-NAG. la membrana.

Se genera pues el enlace ß (1-4) entre NAG y Θ Luego que queda expuesto el precursor -
NAM. Por lo tanto, se obtiene: NAM/pentapéptido)-NAG-BPP tiene lugar la
polimerización de varias unidades disacarídicas:
NAM / pentapéptido-NAG –BPP . ello se logra en una reacción de transglu-
cosilación.
Tanto la traslocasa como la transferasa están
localizadas en el lado citoplásmico de la Consiste en la unión de cada unidad disacarídica
membrana, de modo que el precursor Lip-P- NAM-NAG (con su pentapéptido) unida a su
P- NAM /pentapéptido)-NAG, en este respectivo BPP, con el extremo libre (reductor) de
momento está “colgando” hacia el citoplasma, una cadena preexistente que a su vez está unida a
otra molécula de BPP.
anclado a la lámina interna de la membrana a
través de bactoprenol.
Θ En el proceso se libera uno de los BPP (o sea,
el undecaprenil, pero en forma pirofos-
forilada).

El precursor NAM (pentapéptido)-


NAG-BPP se encuentra en Sobre el Lip-P-P (BPP) actúa una
disposición hacia el citoplasma de la fosfatasa específica, que elimina el
membrana fijado a través del fosfato terminal, regenerándose el
bactoprenol. undecaprenil-fosfato, que queda
dispuesto para otro ciclo como el
descrito.
29
Manual de INFECTOLOGÍA PLUS MEDIC A

Formación del peptidoglicano

30
Manual de INFECTOLOGÍA PLUS MEDIC A

TRANSPEPTIDACIÓN Antibióticos que actúan a nivel de la biosíntesis


El peptidoglicano lineal naciente reacciona de peptidoglucano
por transpeptidación con un peptidoglicano
aceptor preexistente. Producen la acumulación de precursores del
peptidoglicano , que desencadena la activación
En esta reacción se ven implicados el grupo de las autolisinas de la bacteria, que degradan
C=O de la D-ala del peptidoglicano naciente y el peptidoglicano y provoca una lisis porosmótica
el grupo -NH2 libre del diaminoácido del
peptidoglicano aceptor (o del último I.Inhiben la Síntesis de precursores
aminoácido del puente peptídico).
Esto es lo mismo que decir que el enlace peptídico Fosfomicina: actúa inhibiendo la
entre D-ala y D-ala del peptidoglicano naciente se formación del 3-O-D-lactil-éter de la NAG
ve sustituido por otro enlace peptídico, entre dicha D- (o sea, del NAM) al inhibir la acción de la
ala y el diaminoácido del peptidoglicano naciente. enzima D-alanina racemasa (es la que
convierte la L-alanina en D-alanina).
Autolisinas
Muchas bacterias controlan el grado de Cicloserina: Inhibe la adición del
entrecruzamiento de su peptidoglicano maduro. dipéptido D-alanil D-alanina al UDP-
Incluso algunas pueden eliminar totalmente muchos de NAM impidiendo la formación del
los péptidos originalmente unidos al NAM, mediante pentapéptido UDP-NAM, al actuar como un
enzimas conocidas genéricamente con el nombre análogo estructural de D-lanina.
de autolisinas.

II.Inhiben el Transporte
No todos los tetrapéptidos participan en
entrecruzamientos.
Tunicamicina: inhibe la traslocasa que
Θ Las D-ala terminales de los péptidos no
cede el NAM unido hasta entonces al UDP y lo
implicados en tales entrecruzamientos son
pasa al transportador bactoprenol.
eliminadas por una enzima llamada D-D-
carboxipeptidasa.
Θ Esta enzima explica no sólo que en el Bacitracina: se une al undecaprenol-
peptidoglicano maduro existan tetrapéptidos pirofosfato, bloqueando su desfos-
(y no los pentapéptidos originales), sino también forilación, e impidiendo por lo tanto, la
la existencia de tripéptidos. regeneración del transportador de
membrana.
Las enzimas que posibilitan reacciones
auxiliares de carboxipeptidación, se conocen III. Inhiben la translucosilación
con el nombre de proteínas fijadoras de
penicilina (PBP, penicillin-binding proteins). Vancomicina y teicoplanina:
Recubren el extremo D-alanin-D-alanina del
disacárido-pentapéptido, evitando así la
acción de las glucosiltransferasas y
transpeptilasas, y en consecuencia evitan
la elongación del peptidoglucano.

IV. Inhiben la Transpeptidación

ß-lactámicos: inhiben la reacción de


entrecruzamiento por transpeptidación
(inhibe la transpeptidasa PBP (Proteína
ligadora de penicili -na).

31
Manual de INFECTOLOGÍA PLUS MEDIC A

Membrana celular Θ Puede permanecer meses o años en una


situación metabólica de inercia .
Está formada por fosfolipidos y proteinas, y a Θ Cuando las condiciones son más favorables se
diferencia de las eucariotas, no contiene produce la germinación, con la formación de
esteroles (excepto el mycoplasma). una célula única que después se reproduce con
normalidad.
Es una barrera osmótica, selectiva y
activa: PARA RECORDAR:
√ Contiene sistemas de transporte para los
solutos y regula el transporte de productos
celulares hacia el exterior.

Es un sitio de acción de detergentes y


antibióticos polipeptídicos como la
polimixina (Por ejemplo: colistin).

Citoplasma

Está formado por 85 % de agua.


Contiene los ribosomas y el cromosoma
bacteriano.

Ribosomas

Están formados por ARN ribosómico. OF: Una de las siguientes afirmaciones referidas
Su importancia radica en ser el sitio de acción de al lipopolisacá
lipopolisacárido (LPS) de la membrana externa
numerosos antibioticos: de la pared celular de las bacterias es CIERTA:
Θ Aminoglucósidos, tetraciclinas cloranfe-
nicol, macrólidos incosamidas.
a.Es una zona termolá
termolábil que no resiste la
esterilizació
esterilización en autoclave.
Nucleoide (cromosoma bacteriano) b.Tiene una actividad endotóxica que está asociada
con el lípido A.
“Equivalente nuclear” c.Contiene el Ag O que es esencial para la viabilidad
celular.
No posee membrana nuclear d.Su concentració
concentración en sangre no está
está directamente
Θ Está formado por un único filamento de relacionada con la mortalidad .
ADN apelotonado (superenrollado). e.No proporciona resistencia a la fagocitosis
Θ Confiere sus peculiaridades genéticas a la Rpta. B
bacteria.
Θ Regula la sintesis proteica. El antigeno del LPS provoca un amplio espectro
de efectos fisiopatologicos:
Esporas
Estructura presente en algunas especies Cuando la cantidad en sangre es suficiente, el
bacterianas exclusivamente bacilares. lipopolisacarido produce la muerte en una o dos
Le permite a la célula sobrevivir en horas, debido a shock irreversible y colapso
condiciones extremadamente duras. cardiovascular
El material genético de la célula se concentra y es
rodeado por una capa protectora, que hace que la célula
sea impermeable a la desecación, al calor y
numerosos agentes quimicos.

32
Manual de INFECTOLOGÍA PLUS MEDIC A

CULTIVOS Estreptococo pyogenes


(beta-hemolítico)

Agar sangre
Agar EMB
La base de Agar Sangre es un medio utilizado
para el aislamiento y cultivo de una amplia variedad (Eosin Methylene Blue Agar)
de microorganismos.
Este medio (también denominado E.A.M.) es
La Base de Agar Sangre generalmente es utilizado para el aislamiento selectivo de
suplementada con sangre de carnero, conejo o bacilos Gram negativos de rápido desarrollo
caballo al 5-10% para aislar ,cultivar y estudiar y escasas exigencias nutricionales.
reacciones hemolíticas de una amplia variedad
de microorganismos patógenos. Permite el desarrollo de todas las especies de la
familia Enterobacteriaceae
◆Betahemolítico
Hay hemólisis completa. Lactosa y/o sacarosa
Se observan colonias translúcidas u opacas,
grises, pequeñas y mucoides con una zona de La diferenciación entre organismos capaces de
hemólisis (Estreptococos betahemolíticos) utilizar la lactosa y/o sacarosa, y aquellos que
◆Alfahemolítico son incapaces de hacerlo, está dada por los
Hay hemólisis incompleta. indicadores eosina y azul de metileno.
Las colonias de aparecen planas, lisas,
translúcidas, grisáceas y algunas veces Lactosa (+): las cepas poseen centro oscuro con
mucoides rodeadas por una pequeña zona periferia azulada o rosada.
verdosa de alfa hemólisis (neumococos,
Estreptococo viridans). Lactosa (-) : son incoloras.
◆Gammahemolíticos
Enterococcus spp. crece en este medio como
No hay hemólisis.
No se presentan cambios (enterococos). colonias puntiformes y transparentes, mientras
que Acinetobacter spp. y otras bacterias
oxidativas pueden dar colonias de color azul
lavanda. En este medio se obtiene además, un
buen desarrollo de especies de Salmonella y
Shigella.

MIR: Una de las siguientes afirmaciones


referidas al a los ácidos teicoicos de la la pared
celular de las bacterias gram positivas es
incorrecta:

A.Son
A.Son polí
polímeros hidrosolubles de fosfatos de
poliol B.Son Ag de superficie comunes
C.Tiene una estructura de unió
unión a otras bacterias
D. Constituyen una barrera para la penetració
penetración de
determinados antibió
antibióticos dentro de la célula
E:Se unen a receptores especí
específicos en la superficie
de las células del huésped
Rpta. D

33
Manual de INFECTOLOGÍA PLUS MEDIC A

Propiedades de las enterobacterias Vancomicina, colistina, nistatina y


trimetoprima).
ΘE. coli es lactosa (+), catalasa(+), móvil
y fermenta la glucosa y produce gas. Las colonias de gonococos y meningococos
obtenidas en este medio selectivo son
Θ La Salmonella es lactosa (-), catalasa(+), usualmente opacas, grisáceas, convexas, de 1-4
móvil y fermenta la glucosa y produce gas. mm de diámetro. Luego de 48 h de incubación
pueden transformarse en colonias mucoides.
Θ La Shigella es lactosa (-), catalasa(-),
inmóvil y NO fermenta la glucosa . Coloración de Gram: diplococos Gram negativos.

Nemotecnia: Medio de cultivo Lowenstein- Jensen

E. coli : Es esta una base para la preparación de


-Con “ubres” (lactosa +) varios medios destinados al aislamiento,
-Con fimbrias (móvil) cultivo y diferenciación de micobacterias.
-Con una “nariz” que percibe olor (produce gas) Θ El verde de malaquita inhibe a gran parte de
la flora acompañante. Θ Con el agregado de
Shigella :
glicerina se estimula el crecimiento de
-SIN ubres (lactosa - )
Mycobacterium tuberculosis, aunque gran parte
-SIN fimbrias (inmóvil)
de M. bovis es inhibido. Θ Agregando un 5% de
-SIN “nariz”
NaCl, se pueden seleccionar micobacterias
tolerantes a la sal, como es el caso de M.
smegmatis.

Cultivos en medios especiales

Medio de cultivo Thayer Martin

Permite el correcto aislamiento y recuperación VIRUS


de las neisserias patógenas dentro de las 24 h
Introducción
El medio de cultivo se prepara a partir del
Agar Base GC, con el agregado de La palabra virus significa veneno y
corresponde a la denominación que se le dio
hemoglobina, un suplemento de
originalmente a fines del siglo XVIII a ciertas
enriquecimiento (Britalex) y una mezcla
sustancias que tenían poder patógeno.
antimicrobiana (VCNT:

34
Manual de INFECTOLOGÍA PLUS MEDIC A

Características
 Los virus son organizaciones macro-
moleculares constituidas fundamental-
mente por ácidos nucleicos y proteínas.

El ácido nucleico que poseen constituye


el genoma viral.

Las proteínas virales están codificadas por el


genoma viral

 Los virus corresponden a partículas


submicroscópicas de tamaño variable.

PARVOVIRUS: es uno de los virus más


pequeños (producen el eritema infeccioso ). Virus: VIH con sus ligandos GP120 Célula:
Virus POX: es uno de los más grandes linfocito con sus receptores CD$.la unión se
(responsable de la viruela). produce entre el GP 120 y el receptos CD4.

Parvovirus

 Son agentes infecciosos con carácter


estrictamente intracelular; los virus son
capaces de reconocer células e infectarlas.

Invasión del virus a una célula

La presencia de elementos que permiten la


unión del virus a la célula y su El VIH al invadir el linfocito une su ADN al del
posterior penetración. linfocito formando el PROVIRUS (lo que le
permite replicarse).
ΘEn las células: la presencia de
receptores 5.- Los virus son microorganismos capaces de
infectar diversos tipos celulares en los
ΘEn el virus: proteínas ligandos o de organismos vivos.
infectividad (antireceptores)
Pueden infectar bacterias, células vegetales
No poseen la capacidad de multiplicarse y animales. Puede afectar la expresión génica
o de sintetizar por sí mismos sus propios
de las células y contribuir a la variabilidad de
componentes
las especies y, por ende, a su desarrollo y
Θ Al infectar las células vivas aprovechan la evolución.
maquinaria metabólica celular para
“ Virus-like agents ” : son los transposones ,
realizar la síntesis de sus componentes, y de
plásmidos, viroides y priones que comparten
esta manera replicarse generando progenie
algunas características de los virus.
viral.
35
Manual de INFECTOLOGÍA PLUS MEDIC A

Son capaces de autoreplicarse independien- Nucleocápsula (Nucleocápside)


temente del genoma celular.
Es el conjunto de ácido nucleico y proteínas
Por ejemplo los PLÁSMIDOS de una hebra de altamente organizado.
DNA circular se replican en forma similar a los
virus DNA de una hebra. A diferencia de los Se ordena adoptando las siguientes formas:
virus, los plásmidos no son patogénicos y se a) Icosaedro: consiste en un poliedro regular de
transfieren por conjugación entre células. 20 caras planas triangulares.
b) Helicoide: la organización corresponde a una
Los VIRIONES son RNA circulares desnudos de
estructura en espiral o hélice.
una hebra, que causan enfermedades en
c) Compleja: en este tipo de nucleocápsula no
plantas.
hay una simetría regular.
Se presume que se replican por intermedio de
La estructura de la nucleocápsula le confiere a las
la RNA polimerasa del hospedero y causan
partículas virales diversas propiedades, como
efectos patogénicos al interferir con el
estabilidad termodinámica y capacidad de
metabolismo de la célula .
almacenar un máximo de masa en el menor
volumen.

¡Aaaa su ……!

En humanos el agente Hepatitis delta,


dependiente de Hepatitis B, es
estructuralmente similar a un viroide. El
agente delta codifica una proteina
estructural.
Virión
Los PRIONES son agentes infectivos de
naturaleza proteica que son patogénicos en Es la organización física de los virus como
partículas, que corresponde a la particula viral
vertebrados.
completa extracelular.

Estructura viral PROTÓMEROS: unidades estructurales que se


forman al agruparse las proteínas virales.
Los virus están constituidos por macro -
CAPSÓMEROS: unidades morfológicas, que
moléculas, las cuales se organizan de tal manera
integran la nucleocápsula y están formadas por
que le confieren sus propiedades biológicas y
protómeros..
físico-químicas. Estos componentes moleculares
son los siguientes:

- Ácido nucleico: DNA o RNA.


- Proteínas.
- Lípidos.
- Hidratos de carbono.

Estos componentes se organizan constituyendo


Capsómeros
las partículas virales.

36
Manual de INFECTOLOGÍA PLUS MEDIC A

Algunos virus poseen lípidos e hidratos de


carbono que se organizan en bicapas de
Nomenclatura
lípidos con glicoproteínas insertas, en la El nombre de los virus obedece a distintas
misma organización que las membranas consideraciones:
celulares.
√ A la enfermedad que ellos producen: virus
GENÓMA VIRAL polio (porque produce la poliomielitis).
Está constituído por DNA o RNA. √ Palabras compuestas como "papova":
contracción de los nombres papiloma,
Contiene la información genética (genes) polioma y vacuolante.
necesaria para la síntesis de las proteínas √ Al nombre de los descubridores: virus de
virales. Epstein-Barr.

En algunos virus se conoce su secuencia √ Por características estructurales: coronavirus.


nucleotídica completa y en otros sólo la
√ Lugar donde se detectaron por primera vez:
naturaleza de ciertos genes.
Coxsackie o Norwalk.
PROTEÍNAS VIRALES
Clasificación de los virus
Las proteínas que forman parte de la
estructura viral están codificadas en el El ICTV (International Committee on Taxonomy
genoma viral. of Viruses) ha propuesto un sistema universal
de clasificación viral.
Algunos virus, como el de la polio, poseen 4
proteínas y otros más complejos, cerca de 100 El sistema utiliza una serie de taxones
(ej: pox). como se indica a continuación:

Las proteínas virales presentan ciertas Orden (-virales).


propiedades y son responsables de diversas Familia (-viridae)
funciones biológicas. Subfamilia (-virinae)
Genero (-virus)
Algunas de ellas corresponden a la infectividad,
protección del genoma viral, actividad Especie ( ).
enzimática, capacidad patogénica, virulencia, Familias y subfamilias
inmunogenicidad y antigenicidad.
Los virus se agrupan en familias y subfamilias
Variaciones y cambios en las proteínas virales cuyo nombre se ha latinizado; por ejemplo, los
como consecuencia de cambios en el genoma virus herpes se agrupan en la familia
dan origen a variantes genéticas que Herpesviridae.
determinan tipos y cepas, las que presentan
Las subfamilias tienen el sufijo “ nae ” , Ej:
distintas propiedades biológicas y patogénicas.
Herpesvirinae.
La manipulación controlada de los genomas Género
virales y la obtención de partículas virales
con proteínas que presentan determinadas El otro tipo de agrupación es el género, que no
características, han sido fundamentales en se nombra en forma latinizada, por ejemplo,
la obtención de vacunas. herpesvirus.

37
Manual de INFECTOLOGÍA PLUS MEDIC A

Por ejemplo, el virus Ebola de Kikwit se clasifi- OF: ¿Qué estructura separa la pared celular de la
fica como: membrana celular en las células bacterianas?
Orden: Mononegavirales
Familia: Filoviridae A) Péptidoglucano
Género: FilovirusEspecie: B) Porinas
Virus : Ebola Zaire C) Espacio periplásmico
D) PBP
EsSalud 2010 (2) : Que tipo de virus es el de E) Todas las anteriores
la parotiditis: Rpta C
a. Paramixovirus
b. Arenavirus OF: Uno de los siguientes reactivos no es
c. Hepadnavirus utilizado en la tinción de Gram:
Rpta. A A) Violeta de Genciana
B) Azul de Metileno
OF: ¿Cuál de las siguientes bacterias carece de
C) Lugol
pared celular?:
A) Chlamydias. D) Mezcla alcohol – acetona
B) Mycoplasma. E) Safradina
C) Rickettsias. Rpta B
D) Micobacterias.
E) Todas las bacterias tienen pared celular. Bibliografía
Rpta. B
1ºDe Vos, P., Garrity, G., Jones, D., Krieg, N. R.,
OF: Los plásmidos: Ludwing, W., Rainey, F. A., Schleifer, K.H. and
A) Son esenciales para la vida del
Whitman, W. B. (2009). Bergey ’ s Manual of
microorganismo.
Systematic Bacteriology. 2nd Ed. Springer Verlag.
B) Son extracelulares.
C) Nunca se integran en el cromosoma USA.
bacteriano.
2ºMadigan, M.T., Martinko, J. M., Dunlap, P.V.,
D) Son materiales de reserva.
Clark, D.P. (2009). Brock. Biología de los
E) Pueden conferir resistencia a los antibióticos
Microorganismos. 12 ª ed. Pearson Prentice Hall.
Rpta. E
España.
OF: Es el componente específico de la pared
celular de las bacterias GRAM positivas 3ºPelczar, M.J., Reid, R.D. y Chan, E.C.S. (2000).
A) Péptidoglucano Microbiología. 6ª ed. McGraw Hill. México.
B) Ácido teicoico
C) Lípido A
D) Antígeno O
E) Ácido micólico
Rpta. B

OF: El carácter de ácido alcohol resistencia en


el género Nocardia, se debe a la presencia a nivel
de pared celular de:
A) Péptidoglucano
B) Ácido teicoico
C) Lípido A
D) Antígeno O
E) Ácido micólico
Rpta. E
38
Manual de INFECTOLOGÍA PLUS MEDIC A

ANTIBIÓTICOS PLUS MEDIC A

Historia Términos básicos

1877: Pasteur y Joubert reconocen que


algunos productos bacterianos pueden ser -Biodisponibilidad
deletéreos para otras bacterias. % DEL TOTAL DEL FÁRMACO administrado que llega
a la circulación general.
1900: Paul Erlich también sugiere la
presencia de sustancias antibióticas.

1929: Fleming descubre la Penicilina en


cultivos de hongos.

1935: Sulfonamidas. (Domagk -premio


Nobel de Medicina en 1939- descubre las
propiedades antimicrobianas del colorante
prontosil

1939 un grupo de investigadores


franceses descubre que sólo una parte de la
La biodisponibildad del fármaco es como
molécula de colorante rojo era responsable del
el porcentaje de salmones que llegan a
efecto bacteriostático del nuevo producto: la destino para desovar.
sulmanilamida.
¿Cómo se determina la biodisponibilidad?
1941: Primer uso clínico de la Penicilina, en
Londres, Inglaterra.
CON LA GRÁFICA DE CONCENTRACIÓN RESPECTO AL
TIEMPO
1947 : Estreptomicina

1948: clortetraciclina y cloranfenicol

1951: Isoniacida

Fig 1 Biodisponibilidad

Cmax: concentración máxima obtenida

Es la concentración más alta del fármaco


en la sangre después de su administración
oral.

Es directamente proporcional a la
fracción del fármaco absorbido
39
Manual de INFECTOLOGÍA PLUS MEDIC A

Tmax: El tiempo al cual se aprecia la Concentración inhibitoria mínima (CIM)


concentración máxima
Concentración más baja de un
También está estrechamente relacionado con antibiótico para inhibir el crecimiento
de un inóculo standard ( >= 10 5 UFC )
la velocidad de absorción.

Es la medida de la sensibilidad de una


Fármacos con valores de Tmax más pequeño
bacteria a un antibiótico
presentarán respuesta farmacológica en
tiempo más cortos Al igual que el
espantapájaro el
ABC: El área bajo la curva de la CIM del antibióti-
co no elimina la
concentración plasmática respecto al tiempo bacteria sólo la
“espanta”(inhibe
Es la cantidad de fármaco absorbido. su crecimiento)

El ABC es directamente proporcional a la


cantidad de fármaco que ingresa a la
circulación y es independiente de la velocidad Concentración Bactericida mínima (CBM)
de absorción.
Concentración más baja de un
antibiótico para eliminar el inóculo
¿ De que depende la biodisponibilidad de
standar en 99.9%.
un fármaco?

Depende directamente de las propiedades


inherentes al fármaco como:
Clasificación
√ Facilidad de solubilizarse en los fluidos Bactericidas: su acción es letal, llevando a
biológicos (acuosos o lípidicos). la lisis bacteriana.

√La forma en la que el organismo la Bacteriostáticos: impiden el desa-


asimila : como la absorbe, la distribuye, rrollo y multiplicación bacteriana sin llegar a
metaboliza y elimina del organismo. destruir las células.

BACTERICIDAS BACTERIOSTÁTICOS
Tiempo de vida media
Penicilinas Tetraciclinas
Tiempo que necesita el organismo
para conseguir que LA CONCENTRACIÓN Cefalosporinas Eritromicina
PLASMÁTICA DISMINUYA A LA MITAD.
Aminoglucósidos Sulfonamida

Rifampicina Novobiocina

Polimixinas Cloranfenicol

Tabla 1 Bactericidas y bacteriostáticos

40
Manual de INFECTOLOGÍA PLUS MEDIC A

Antibióticos concentración-dependiente y
Comportamiento del antibiótico
tiempo-dependiente
La acción de los antimicrobianos
depende de: Son dos modelos de acción de los
-La relación entre: antimicrobianos a nivel del compartimento
LA CONCENTRACIÓN PLASMÁTICA ALCANZADA (CMAX) central:
y Concentración dependiente
LA CIM QUE PRESENTA EL MICROORGANISMO AL
FÁRMACO QUE SE ESTÁ EVALUANDO.
Su acción se relaciona a la concentración
plasmática

Hay 2 modelos:

 Su efectividad depende de la relación entre


la concentración máxima alcanzada por el
antibiótico y la CIM del microorganismo
que se está tratando (CMÁX/CIM)

AMINOGLUCÓSIDOS,
QUINOLONAS,METRONIDAZOL

Aminoglucósidos
A > Cmáx/CIM > efectividad del antibiótico.
En un estudio se encontró que una relación
Cmáx/CIM < 4, 4 - 10 y > 10 se correlacionó
con un respuesta favorable de 57, 67 y 85%
Fig 2 Modelo farmacocinético respectivamente.

Después de la administración de un antibiótico  Su efectividad depende del área bajo la


se observa una fase en que aumenta la curva (AUC) sobre la CIM del
concentración de mismo, corresponde a la
microorganismo que se está tratando.
infusión, para luego llegar a la
concentración peak. AMINOGLUCÓSIDOS, QUINOLONAS,
Luego esta concentración tiene 2 fases : AZITROMICINA, TETRACICLINA,
-Fase A (de disminución acelerada) : VANCOMICINA
en correspondencia con la distribución del
fármaco en los tejidos . Mide la exposición total de la bacteria
-Fase B (de disminución lenta) : que frente a un antimicrobiano
corresponde a su eliminación del organismo.
Tiempo dependiente
Farmacodinamia
Su acción se relaciona al tiempo en que ellos
Evalúa la eficacia terapéutica en base a la
están presentes en concentraciones
relación entre: superiores a la CIM).

LA CONCENTRACIÓN PLASMÁTICA DEL ANTIBIÓTICO y T > CIM


LA CIM DEL MICROOGANISMO A ESE
ANTIMICROBIANO. BETALACTÁMICOS,
MACRÓLIDOS ,CLINDAMICINA

41
Manual de INFECTOLOGÍA PLUS MEDIC A

Las combinaciones de antimicrobianos se


pueden utilizar para tener efecto:

 Aditivo
 Sinérgico (penicilinas más aminoglucó-
sidos)
 Antagónico (penicilinas más tetraciclinas)
Efecto post-antibiótico (aminoglucósidos y
ciprofloxacino)

Consiste en el retardo de los microorganismos


en recuperarse y re-entrar en fase logarítmica
de crecimiento después del uso de un
antimicrobiano.

Mecanismo de acción
Inhibición de la síntesis de la pared celular

√Penicilinas
√Cefalosporinas
√Cefamicinas Mecanismo de acción de los antibióticos
√Carbapenems
√Monobactams Mecanismos de resistencia de
√Glucopéptidos: Vancomicina los antibióticos

Inhibición de la síntesis de proteínas PENICILINAS Y CEFALOSPORINAS

Producción de beta-lactamasas (las cuáles


√ Macrólidos rompen el anillo betalactámico), cambio en las
√ Tetraciclinas proteínas de unión a las penicilinas (PBP) y
√ Aminoglucósidos cambio en las porinas.
√ Cloranfenicol
√ Fusidato sódico AMINOGLUCÓSIDOS

Inhibición de la síntesis de ácidos nucleicos Formación de enzimas que inactivan las


drogas vía reacciones de conjugación que
transfiere acetil, fosforil o grupos adenil.
√Quinolonas
√ Azoles MACRÓLIDOS y CLINDAMICINA

Inhibición de la síntesis de ácido fólico Formación de metiltransferasas que


después se unen a los sitios de unión de la
√Sulfonamidas subunidad ribosomal. 50s.
√Trimetropin
√Pirimetamina Activa el transporte fuera de las células

42
Manual de INFECTOLOGÍA PLUS MEDIC A

TETRACICLINAS En la última fase de la síntesis del


Actividad incrementada de sistemas de peptidoglicano (Ensamblaje) se produce la
transporte que bombea al antibiótico fuera de la transpeptidación que da lugar al
célula. entrecruzamiento de los peptidoglicanos lineales.
PBP (Proteína ligadora de penicilina).
SULFONAMIDAS
Cambio en la sensibilidad a la inhibición de la LOS BETALACTÁMICOS INHIBEN LA SÍNTESIS DEL
enzima blanco incrementando la formación de PETIDOGLICANO ,AL INHIBIR LA ÚLTIMA FASE
PABA; uso de ácido fólico exógeno. (ENSAMBLAJE) AL BLOQUEAR LAS PBP.

FLUORQUINOLONAS PENICILINAS
Cambio en la sensibilidad a la inhibición de
enzima blanco, incrementando la actividad de Son un grupo de antibióticos de origen natural y
sistemas de transporte que promueve el eflujo semisintético que contienen el núcleo de ácido 6-
del antibiótico. aminopenicilánico, que consiste en un anillo
betalactámico unido a un anillo tiazolidínico. Los
CLORANFENICOL compuestos de origen natural son producidos por diferentes
Formación de acetiltransferasas inactivantes especies de Penicillum spp.

LOS INHIBIDORES DE LA SÍNTESIS DE


LA PARED CELULAR

BETALACTÁMICOS
◆Penicilinas
◆Cefalosporinas
◆Monobactams
◆Carbapenems

Su mecanismo de acción es la inhibición de las Clasificación


síntesis de la pared bacteriana al inhibir la Primero aparecieron las PNC naturales,luego
síntesis del peptodoglicano, a nivel de las antiestafilococo, después aparecieron las
transpeptidación al bloquear a las PBP. PCN con el radical amino, el cual fue
reemplazado posteriormente por el radical
carboxi y éste fue cambiado por el radical ureido.
Θ Penicilinas naturales
GyV
Θ Penicilinas resistentes a las peni -
cilinasas estafilocócicas
Oxacilina, meticilina, dicloxacilina
Θ Aminopenicilinas
Ampicilina, amoxicilina
Θ Carboxipenicilinas
Ticarcilina, carbenicilina
Θ Ureidopenicilinas
Piperacilina, Azlocilina, Nezlocilina

43
Manual de INFECTOLOGÍA PLUS MEDIC A

Espectro antimicrobiano Biodisponibilidad


En la sangre los betalactámicos circulan como
Penicilina G : abarca cocos grampositivos, sustancias libres o unidas a las proteínas
cocos gramnegativos (Neisseria meningitidis) plasmáticas.
y bacilos grampositivos, tanto facultativos Sólo la fracción libre de la droga es activa y
como anaerobios, así como espiroquetas y capaz de penetrar al espacio extracelular.
algunos bacilos gramnegativos anaerobios.
Los betalactámicos son sustancias poco
Penicilinas antipseudomonas: (carboxi y lipofílicas, su penetración intracelular es
ureidopenicilinas) son estables frente a las escasa, no alcanzando casi nunca
betalactamasas cromosómicas propias de concentraciones mayores del 25% al 50% de las
Pseudomonas pero no ante la presencia de concentraciones plasmáticas.
betalactamasas plasmídicas.
Excreción
Es renal. Puede ser bloqueada con la
administración de probenecid, lo que
prolongada la vida media sérica.

Los efectos adversos

 Por lo general bien tolerado


 Diarrea ocasional
 Las reacciones alérgicas incluyendo
anafilaxia (10% de reactividad cruzada con
Absorción oral cefalosporinas)

La penicilina G no se absorbe bien mientras que Penicilinas antiestafilocócicas


la V resiste la inactivación gástrica y se absorbe (oxacilina)
mucho mejor. La amoxicilina se absorbe mejor
que la ampicilina (95% contra 40%) -ENAM 06-  La posesión de cadena lateral acilo impide la
B (6) - Las penicilinas antiestafilocócicas, hidrólisis .
oxacilina y dicloxacilina, son estables al ácido
gástrico y se absorben adecuadamente. Efectos adversos

ENAM 06-B (6) : La presencia de alimentos NO  Ictericia colestásica


altera la absorción de una de las siguientes
 Reacciones alérgicas
penicilinas: Rpta. Amoxicilina
Por lo general menos activa (10-20 veces
La penicilina G benzatínica tiene una absorción contra benzilpenicilina).
lenta desde su depósito intramuscular. Por tanto
solo es adecuada para el tratamiento de Aminopenicilinas (amplio espectro)
infecciones por gérmenes extremadamente
sensibles como Streptococcus pyogenes, y para el
Mecanismos: confiere modificación química
tratamiento de la sífilis.
 Actividad frente a bacilos gram negativos
Las penicilinas se distribuyen en
muchos compartimentos como Por ejemplo, Haemophilus influenzae y
pulmones, hígado, músculo, hueso y coliformes
placenta. La penetración en : ojo,
cerebro, LCR y próstata es pobre en
 Aumento de la actividad contra el
ausencia de inflamación.
enterococo (Streptococo faecalis).

44
Manual de INFECTOLOGÍA PLUS MEDIC A

Efectos adversos Modificaciones en la posición 7 del ácido 7-


aminocefalosporánico están asociadas con la
-Ampicilina suele causar diarrea alteración en su actividad antibacteriana y
-Rash frecuente sustituciones en la posición 3 están asociadas a
-Reacciones alérgicas alteraciones

La combinación de amoxicilina más ácido


clavulánico (inhibidor de la penicilinasa) es
activa frente a estafilococos productores de
penicilinasa, E. coli y H. influenzae. El ácido
clavulánico es asociado con diarrea e ictericia

Penicilinas antipseudomonas
(piperacilina, azlocilina, ticarcilina,
carbenicilina)

Útiles para las infecciones gram-negativos,


especialmente con pseudomonas.

EsSalud 01 (30): : La penicilina actúa en las


bacterias a nivel de: Se definen cuatro generaciones de
cefalosporinas.
a) La síntesis de ADN
b) La síntesis de ácido fólico Las cefalosporinas de primera generación son
c) La pared celular. muy activas frente a los cocos grampositivos;
d) La membrana celular en líneas generales, la segunda y tercera
e) b y d generaciones han perdido parte de esa
actividad, en beneficio de una mayor actividad
Rpta. C frente a bacilos gramnegativos, con algunas
excepciones.
EsSalud 07(47):: ¿ Cuál de los siguientes
antibióticos se asocia con mayor frecuencia a
Todas las cefalosporinas son inactivas frente a
reacciones dérmicas?
enterococos, estafilococos resistentes a la
A) Quinolonas
meticilina y Listeria monocytogenes.
B)Betalactámicos
C) Cotrimoxazol La mayoría de las cefalosporinas son de
D) Eritromicina administración parenteral, aunque existe un
E) Cloranfenicol número creciente de formulaciones para vía
oral como la cefalexina, cefradina, cefadroxil,
Rpta. B cefuroxima ,cefdinir y otras. La absorción
gastrointestinal de estos compuestos es buena.
Se obtienen buenas concentraciones en
CEFALOSPORINAS Y CEFAMICINAS líquidos biológicos y suero. No se obtienen
buenas concentraciones intracelulares.
Son productos de origen natural derivados de
productos de la fermentación del Cephalospo Cefotaxime, ceftriaxona, cefoperazona y
rium acremonium. cefepime entran en el LCR alcanzando
altas concentraciones.
Contienen un núcleo constituído por ácido 7- Todas las cefalosporinas, excepto
aminocefalosporánico formado por un anillo cefoperazona (que tiene excreción biliar),
betalactámico unido a un anillo de dihidrotia- se excretan primariamente por el riñón.
zino.

45
Manual de INFECTOLOGÍA PLUS MEDIC A

Ceftriaxona tiene la vida media más larga (8 Ceftabiprol


horas) lo que permite su administración 1 o 2 Ceftabiprol es una pirrolidinona del grupo de
veces al día. las cefalosporinas que tiene un espectro de
actividad frente a bacterias gram positivas,
Clasificación gram negativas y organismos anaerobios, con
-Primera generación : cefradina, cefalexina, una actividad única frente al staphylococco
meticilin resistente (MRSA) y vacomicin
cefazolina.
resistente (VRSA)
oActividad equivalente a la ampicilina
oIndicación:
Ceftarolina
Profilaxis preoperatoria (cefazolina)
Cistitis aguda Cefalosporina de 5ta generación .La ceftarolina
Como alternativa de la penicilina fosamil se degrada en el plasma a ceftarolina . Es
efectivo contra el S. auresu MR
2ª generación: cefuroxima
ENAM 06-B (5) : ¿ Cuál de las siguientes
oEquivalente a la ampicilina más una mayor
series de cefalosporinas corresponde a un
activi- dad frente a microorganismos
ordenamiento de mayor a menor espectro
productores de be- ta-lactamasa.
antibacteriano?:
oAumento de la actividad contra el Haemophilus.
A. Cefpirome, cefaclor, ceftriaxona, cefalotina
B.Cefpirome,ceftriaxona, cefaclor,cefalotina
3ª generación: C. Ceftriaxona, cefaclor, cetalotina, cefpirome
oCeftriaxona: actividad especialmente frente a D. Ceftriaxona, cefalotina, cefaclor, cefpirome
Haemophilus,Klebsiella y neumococo.
E. Cetalotina, cetaclor, ceftriaxona, cefpirome
-Es de primera elección en pielonefritis aguda ,
NAC con indicación de hospitalización y MEC. Rpta. B
oCeftazidima: actividad contra la Pseudomona
EsSalud 04 (6): Los siguientes son fármacos
aeruginosa -Essalud 09 (69)-
-Se utiliza en infección intrahospitalaria activos contra pseudomona aeruginosa
oCefdinir (uso oral) excepto:

a) Aztrenarn
4ª generación: cefepima y cefpirome.
b) Cefoperazona
oMayor actividad frente a cocos grampositivos,
c)Ceftriaxona
a enterobacterias y Pseudomonas. Indicadas
en monoterapia de neumonía intrahospitalaria d) Piperacilina / Tazobatam
grave y neutropenias febriles e) Imipenen / Cilastatina
Rpta.C
Son agentes antibióticos de amplio espectro
muy potentes y por lo tanto el riesgo de
MONOBACTAMS
infecciones oportunistas es alta, por ejemplo,
Candida albicans y C. difficile (colitis
pseudomembranosa) Aztreonam, el único monobactámico
disponible para uso clínico, posee una
excelente actividad sobre bacterias
gramnegativas aerobias y facultativas.

¿Existe una Carece de actividad


CEF de 5ta frente a grampo-
Generación? sitivos y bacterias
anaerobias.
Si , el cefta-
biprol

46
Manual de INFECTOLOGÍA PLUS MEDIC A

Actividad bactericida
Se extiende a cocos grampositivos incluyendo
Staphylococcus spp. sensibles a meticilina, S.
pneumoniae y otros Streptococcus. Solo
carecen de actividad frente a los estafilococos
resistentes a meticilina, enterococos resistentes
a betalactámicos, algunas especies de
Pseudomonas y Stenotrophomonas malto-
philia.

Es activo sobre la mayoría de enterobacterias y


Se utiliza para tratar infecciones urinarias Haemophilus spp. incluyendo las cepas
complicadas o septicemia. productoras de betalactamasas.

Efectos adversos Tiene una muy buena actividad anaerobicida,


con excepción de Clostridium difficille. En el
-Erupciones alérgicas caso de ertapenem, este no es activo sobre
-Molestias digestivas - Pseudomonas aeruginosa.
Hígado y toxicidad medular
Estos compuestos son de administración
CARBAPENEMS parenteral y suelen alcanzarse con rapidez
concentraciones plasmáticas elevadas. Se
distribuyen ampliamente.
Imipenem ,meropenem , doripenem

¡Qué haría sin ti


Cilastatina!......
Dice que dijo el
Imipenem.

Som una clase única de betalactámicos que El imipenem sufre inactivación por las
presentan el mayor espectro de actividad conocido hidroxipeptidasas renales, por ello se
dentro de este grupo de antibióticos. combina con cilastatina (inhibidor de
hidroxipeptidasas) de manera de lograr
concentraciones séricas adecuadas. No se
Imipenem es un verda- requiere para meropenem.
dero “cañonazo” direc-
to a la bacteria. Se utiliza para tratar la sepsis en el con texto
renal intr. abdominal, especialmente

Son una clase única de betalactámicos que


Imipenem presentan el mayor espectro de actividad
Es el primer carbapenem desarrollado para uso conocido dentro de este grupo de antibióticos.
clínico. Es un derivado semisintético producido
por el Steptomyces spp. OF: En un paciente con celulitis facial ¿cuál es
el antibiótico de elección?:
Otros compuestos más modernos son
A. Metronidazol
meropenem y ertapenem.
B. Imipenem
C. Clindamicina

47
Manual de INFECTOLOGÍA PLUS MEDIC A

D. Amoxicilina con inhibidor de betalactamasas Los glicopéptidos inhiben la síntesis y el


E. Claritromicina ensamblado del peptidoglicano de la pared celular
Rpta. B mediante la formación de un complejo con la
porción D-alanil-D-alanina del pentapéptido
Indicaciones precursor.
BLEE (betalactamasas de espectro Además daña los protoplastos alterando la
extendido producidas por: permeabilidad de la membrana citoplasmática
-E. coli y altera la síntesis de ARN.
-Klebsiella pneumoniae
-Enterobacterias Se une rápida y firmemente a las bacterias y
Resistentes a : ejerce su efecto bactericida sin un período de
-Todas las efalosporinas inducción, sólo sobre microorganismos en
-TMP-SMX multiplicación activa.
-Fluorquinolonas
-Aminoglucósidos Actualmente hay dos drogas en uso clínico:
Esquemas vancomicina y teicoplanina.
-Imipenem 500mg IV c/6 h
0 Los glicopéptidos son activos además sobre
-Meropenem 1g IV c/8 h Streptococcus, corinebacterias, Bacillus
spp.algunos actinomicetales y Clostridium spp.
0
incluído Clostridium difficile.
-Doripenem 500 mg/kg IV c/8h
CID 39:31, 2004
Absorción: La vancomicina se absorbe poco si
se administra por vía oral. No se administra por
vía intramuscular por el intenso dolor que
GLUCOPEPTIDOS causa en el sitio de inyección.

(VANCOMICINA, TEICOPLANINA) Distribución: La vancomicina tiene un gran


volumen de distribución, alcanzando buenos
Inhibe la glucosidación o sea la formación
niveles en fluídos biológicos como líquido
del Peptidoglicano lineal mediante la pleural, ascitis y sinovial. Tiene una escasa
polimerización de varias unidades disacarídicas penetración intracelular.
del precursor Lip-P-P-NAM (pentapéptido)-NAG.
Tiene una penetración variable a nivel del
sistema central, aunque mejora cuando las
meninges están inflamadas. Sin embargo, no
se recomienda como tratamiento único para
las meningitis bacterianas. Se puede
administrar en forma intratecal en caso de ser
necesario.

La penetración ósea es similar en ambos


compuestos : Vancomicina y tecopalnian (15%
a 20%).
Los niveles de teicoplanina alcanzados en hueso
son superiores a los de vancomicina.

En las infecciones osteoarticulares que


requieran tratamiento prolongado es
preferible utilizar teicoplanina debido también
a su menor toxicidad.

48
Manual de INFECTOLOGÍA PLUS MEDIC A

Eliminación: ambos glicopéptidos se y Enterococcus resistente a los betalactámicos


o a aminoglucósidos. No cubre anaerobios -
eliminan por vía renal, por lo que debe
Essalud 04 (20)-
ajustarse la dosis en el caso de insuficiencia
renal.
Dosis
Dosis
Efectos colaterales √ Paciente severamente enfermos
Sindrome del hombre rojo
Dosis de carga:
La infusión rápida de vancomicina puede
25-30 mg/Kg IV y luego 15-20mg/kg IV
dar lugar a una reacción caracterizada por
eritema y prurito en cuello y parte alta del c/8-12h (infusión x 1.5 a 2 h)
tronco, lo que se conoce como el “Sindrome √ Paciente no severamente enfermo,
del hombre Rojo ” . Esto se debe a una con obesidad mórbida y DepCr >50ml/min
secreción exagerada de histamina. –ENAM 10- 30 mg/Kg IV dividido c/9-12h(infusión x
B(96)- 1.5-2h).
√ Paciente severamente enfermo,
con obesidad mórbida
Dosis de carga:
25-30 mg/Kg , luego 15-20 mg/kg IV c/8-
12h (infusión x 1.5-2h).

¿Vancomicina
OF: Sindrome de hombre rojo ¿ Cuá
Cuál es la en tabletas?
causa?
a.Vancomicina Tratamiento de
b.Amikacina colitis
c.Clindamicina pseudomembranosa
d.Estreptomicina
e.Rifampicina 125mg c/6h
Rpta. A
TEICOPLANINA
La nefrotoxidad de la vancomicina ha disminuído Tiene una estructura similar a la vancomicina y
debido al uso de preparados más purificados y a un perfil de actividad también similar -ENAM 07
la monitorización del tratamiento. (9)- Marca comercial: Targocid (Lab. Adventis)
La vancomicina puede producir trombopenia Presentación : Fco-Amp 200 y 400 mg
o neutropenia que desaparece al suspender
el tratamiento. Dosis
La teicoplanina tiene efectos colaterales similares a la Dosis
vancomicina pero de mucho menor frecuencia. Artritis séptica
12 mg/Kg c/24h
VANCOMICINA Endocarditis por S . aureus
La vancomicina es un antibiótico bactericida de Dosis de carga: 12 mg/kg c/12h
espectro reducido (solo actúa sobre bacterias Luego 12 mg/kg c/24h
grampositivas), que se obtiene de
Streptomyces orientales. TELAVANCINA
Glucopéptido, derivado semisintético de la
Hoy en día es una opción terapéutica
vancomicina. Ejerce actividad bactericida
importante contra Staphylococcus meticilino-
rresistente de perfil hospitalario (SAMAR),
dependiente de la concentración frente a
Staphylococcus coagulasanegativos meticilinorre- bacterias sensibles Gram positivas, incluyendo
Staphylococcus aureus sensible a meticilina y
sistentes, Corynebacterium JK (multirresistente)
meticilin-resistentes (SARM).

49
Manual de INFECTOLOGÍA PLUS MEDIC A

Dosis La unión se realiza mediante la formación


Dosis de puentes de hidrógeno entre diferentes
10 mg/Kg C/24h (en 1 h) radicales hidroxilo del macrólido y
Si Dep Cr > 50ml/min
determinadas bases del ARNr.

Esto provoca un bloqueo en las reacciones de


INHIBIDORES DE LA SÍNTESIS DE transpeptidación y traslocación.

PROTEÍNAS Son efectivos contra organismos gram-


positivos (selectivamente acumulada en
comparación con Gram ne gativos)
Inhiben la subunidad 50 S
-E streptograminas Rango similar de acción y eficacia a la
L incosamidas penicilina.
M acrólidos
Indicaciones
O xazolidinoinas
• Como alternativa a la penicilina en caso de
Nemotecnia: “En los 50´S Fleming descubrió alergia -Essalud 2010 (18) -
a ELMO …. el monstruo de los ATB”
•Actividad contra los organismos que causa
neumonía atípica especialmente legionella,
micoplasma y clamidia.

Efectos adversos

•Diarrea (receptor de motilina actividad agonista)

• Ictericia colestásica y hepatitis (sobre todo con


el éster estolato) –ENAM 10-B(51)-

• Puede causar tromboflebitis en la


administración intravenosa.

•Inhibidores del citocromo P450, puede bloquear


Las Estreptograminas Lincosaminas
el metabolismo de la warfarina y algunos anti-
(lincomicina y clindamicina), ,Macrólidos histamínicos.
(eritromicina, claritromicina, azitromicina), y
Oxazolidinoinas son antibióticos que comparten •La azitromicina tiene menor cobertura de gram-
un mecanismo de acción similar pero tienen positivos, pero muy eficaz contra la clamidia, la
estructura diferente. haemofilus y neisseria.

Riesgo de prolongación del intervalo QT


MACRÓLIDOSS
ENAM 04-A (47) : El uso de eritromicina
Mecanismo de acción:
puede producir:
Se unen a la subunidad 50S del ARN
A. Bloqueo AV completo
ribosómico en forma reversible.
B. Alargamiento del QT en el ECG

50
Manual de INFECTOLOGÍA PLUS MEDIC A

C. Bradicardia sinusal D.1990


Aparición de ondas delta en el ECG
E. Presencia de PR alargado Posteriormente, a principios de la década de
1990, los laboratorios Pharmacia-Upjohn
Rpta. B
sintetizaron dos derivados exentos de toxicidad,
con una estructura tricíclica, denominados
Clasificación
eperezolid (PNU-1005929) y linezolid (PNU-
100766).
Los macrólidos se clasifican de acuerdo al
número de carbonos: 14 carbonos (eritromicina Mecanismo de acción
y claritromicina), 15 carbonos (azitromicina)
y 16 carbonos (espiramicina).
Las oxazolidinonas inhiben la síntesis proteica
en una diana distinta a la de otros
antimicrobianos.

Se fijan a la subunidad 50S, en un lugar de


fijación distinto al del cloranfenicol y
lincosaminas, inhibiendo la formación del
complejo de iniciación 70S .

No inhibe la formación del complejo N-


formilmetionina-ARN-T, ni la elongación, ni la
Eritromicina terminación de la síntesis proteica.

Indicaciones clínicas
CLINDAMICINA
-Infecciones de piel y tejidos blandos: en
 Mecanismo de acción similar
infecciones no complicadas de tejidos blandos
a la eritromicina. la tasas de curación es similar a la de la
 Tiene buena acción contra Oxaaciclina (del 97% para S. aureus)
todos los anaerobios.
 Buena penetración en el -Infecciones complicadas de piel y partes
tejido óseo . blandas: tasas de curación clínica y
 Uno de sus efectos adversoss microbiológica prácticamente idénticas (91%
más característicos es la para S. aureus) a las de la oxacilina.
colitis por Clostridium difficile .Infecciones por microorganismos multi-
-ENAM 08-B (74); ENAM 08-B rresistentes: fundamentalmente ERV y SARM,
(74)- linezolid ha sido eficaz, con unas tasas de
curación clínica de aproximadamente el 75% y
de erradicación bacteriológica de alrededor del
OXAZOLIDINONAS
80%.

1987 En infecciones por SARM

El desarrollo de las oxazolidinonas se inició en Linezolid debería considerarse en aquellos


1987 por parte de los laboratorios EI du Pont al casos que presenten toxicidad (insuficiencia
sintetizar dos moléculas bicíclicas, el DuP-721 y renal) o una respuesta pobre a los
el DuP-105, que fueron abandonadas por glucopéptidos y, entre ellas, las infecciones
problemas de toxicidad y de farmacocinética por S. aureus con sensibilidad intermedia a los
experimental. glucopéptidos (GISA).

51
Manual de INFECTOLOGÍA PLUS MEDIC A

Indicaciones TETRACICLINAS
Enterococo resistente a:
-Vancomicina
Inhiben la síntesis proteica, uniéndose de forma
-Ampicilina
reversible a la subunidad 30 S del ribosoma.
-Penicilina
-Gentamicina
Espectro: Tiene amplio espectro de acción
Esquemas para gram positivos y negativos.
√ Linezolid 600mg IV o VO c/12 h x 8 s
o Indicaciones actuales :
√ Daptomicina 8-12 mg/kg IV c/d
NEJM 365:892, 2011  Clamidia
 Micoplasma
S. aureus resistente a:  Vibrio cholerae
√ Vancomicina  Borrelia (enfermedad de Lyme y la
√ Todos los betalactámicos fiebre recurrente)
( excepto ceftaroline)
Esquemas
√ Daptomicina 6-12 mg/kg IV c/d
0
√ Daptomicina 12 mg/kg IV c/d +
√ Ceftaroline 600 mg IV c/8h
AAC 56: 5296, 2012

Inhiben la subunidad 30 S
 Intolerancia simple gastrointestinal,
hepatotoxicidad y pancreatitis.
-GLICIlciclinas
 Piel: eritrodermia y fotosensibilización.
-AMINO glucósidos  Decoloración de los huesos o los dientes (desde
-TETrA ciclinas el embarazo a 12 años de edad)
 Pueden elevar la urea en sangre inhibiendo la
Nemotecnia: “ Lady GLICI AMINO me síntesis de proteínas mitocondriales
dés TETA”  Asociados con la hipertensión intracraneal
benigna

AMINOGLUCÓSIDOS

(Gentamicina, tobramicina,
estreptomicina)

Está definida por la presencia de dos o más


aminoazúcares unidos por enlaces
 glucosídicos a un anillo aminociclitol.

52
Manual de INFECTOLOGÍA PLUS MEDIC A

PLUS MEDIC A : Paciente que ingresa con


el diagnóstico de intoxicación por órganos
fosforados y neumonía aspirativa
indicándosele clindamicina y amikacina
parenteral ¿ Cuál es el efecto adverso
que podría ocurrir?
A. Nefrotoxicidad
B. Ototoxicidad
C. Paro respiratorio
D.Vértigo

PLUS MEDIC A : Si se decide indicar amikacina a


Espectro de acción: un paciente con insuficiencia renal crónica
severa (depuración de creatinina 20 ml/nin) , ¿
Son activos frente a la mayoría de especies cuál es forma menos nefrotóxica de administrar
de Enterobacteriaceae y Pseudomonadaceae este antibiótico?

Los aminoglucósidos generalmente son A. Reducir la dosis diaria al 50%


activos frente a los estafilococos,pero no
B. Reducir la dosis diaria al 30%
deben indicarse como monoterapia.
C. Administrar 200 mg IV c/ 24h
Los enterococos son moderadamente D. Administrar 1g IV c/ 5 días
resistentes a la gentamicina y la estreptomicina. E. Administrar 1 g IV c/ 3 días

Mecanismo de acción: los aminoglucósidos CLORANFENICOL


se unen de forma irreversible a la subunidad
30S del ribosoma.
 Bacteriostático inhibidor de la síntesis de
Interfiere la lectura correcta del código genético proteínas, pero bactericida para los
con el consiguiente bloqueo de la síntesis patógenos meníngeos como Haemophilus
proteica de la bacteria.
Efectos adversos
Puede que el prolongado efecto postantibiótico
que presentan los aminoglucósidos refuerce su -Anemia aplásica ( 1 / 30, 000 dosificaciones).
capacidad bactericida. -Supresión de la médula dosis-dependiente
(inhibe la síntesis de proteínas mitocondriales)
-Síndrome del niño gris.
Tiene pobre penetración en el LCR (a menos
que las meninges inflamadas)
INHIBIDORES DE LA SÍNTESIS DE
Efectos adversos
ÁCIDOS NUCLEICOS

-Nefrotóxicos y ototóxicos QUINOLONAS


-Agravamiento de la miastenia, bloqueando la
transmisión neuromuscular Son antibióticos bactericidas Inhiben la actividad
de las topoisomerasas de tipo 2 (girasa)
Para reducir al mínimo toxicidad bacterianas después de que éstas se han unido al
ADN.
-Ajustar la dosis para el peso y depuración de
creatinina. La mayoría de bacterias contienen 2 clases de
-Evitar la administración con más frecuencia que topoisomerasas de tipo 2, la girasa y la
una vez al día. topoisomerasa IV. Ambas son tetrámeros
constituidos por 2 tipos de subunidades.

53
Manual de INFECTOLOGÍA PLUS MEDIC A

¿ EL ADN bacteriano siendo tan voluminoso e.Glicilciclinas


como cabe en el interior de la bacteria? Rpta. C
No es magia, el secreto está en que se
encuentra densamente enrollado sobre sí Estructura
mismo. La quinolonas son un grupo de antimicrobianos
¿ Qué se requiere para que el ADN pueda que derivan de una molécula básica formada por
replicarse y transcribirse? una doble estructura de anillo que contiene un
Se requiere 2 acciones que se realizan gracias residuo N en la posición 1.
a las topoisomeras de tipo 2.
1.Relajación o “desenrollado” de su estructura
superhelicoidal : a cargo de la girasa.
2.Separación de las moléculas hijas que, de
otra manera, quedarían encadenadas: a cargo
de la topoisomerasa IV .
El proceso incluye la ruptura de la doble
cadena de ADN y su sellado posterior.

Quinolonas (inhiben la DNA girasa )

Nemotecnia: “ QUINa es tan fuerte que Diferentes sustituciones, incluyendo la inclusión


hace GIRAR hasta el DNA de su rival” de residuos de flúor, han derivado desde el
ácido nalidíxico hasta las quinolonas fluoradas.

Mecanismo de acción
Las quinolonas actúan inhibiendo la ADN girasa,
-EsSalud 01 (35)- enzima que cataliza el
superenrollamiento del ADN cromosómico, que
asegura una adecuada división celular.

Mecanismo de acción de la quinolonas

Essalud 2010 (10) : El mecanismo de acción


de las quinolonas es:
a. Inhibe la síntesis de los peptidoglicanos
b. Inhibe la DNA girasa de los Gram ( -)
c. Inhibe la RNA polimerasa
d. Inhibe la subunidad 30·S
e. Inhibe la subunidad 50·S Clasificación y espectro de actividad:
Rpta. B
Primera generación
RM 11-B (100): Antibiótico que actúa (ácido nalidíxico y ácido pipemídico)
uniéndose a las topoisomerasas: Tienen actividad sobre enterobacterias y son
a.Cefalosporinas inactivas sobre grampositivos y anaerobios.
b.Penicilinas Alcanzan concentraciones muy bajas en suero,
c.Quinolonas su distribución sistémica es baja .
d.Macrólidos

54
Manual de INFECTOLOGÍA PLUS MEDIC A

Se indican para casos de infecciones urinarias Es importante su actividad sobre Streptococcus


bajas por su buena concentración urinaria. y especialmente sobre S. pneumoniae.

Segunda generación (norfloxacina y Además tienen una muy buena actividad sobre
ciprofloxacina) gérmenes atípicos.

Son llamadas fluoradas, ya que incorporan un


átomo de flúor y presentan mucha mayor
actividad sobre gramnegativos.

Cuarta generación (moxifloxacina,


trovafloxacina)

Retienen actividad sobre gramnegativos y


aumentan la actividad sobre grampositivos,
especialmente S. aureus y Enterococcus.
Además agregan actividad sobre mi -
croorganismos anaerobios.
Espectro de acción

Gramnegativos incluyendo a las Pseudomonas,


N. gonorrhoeae, S. aureus, S. epidermidis, H.
influenzae, H. ducrey, gérmenes multirresitentes
a penicilinas y cefalosporinas, micobacterias.

La ciprofloxacina es la quinolona con mejor


actividad sobre Pseudomonas aeruginosa.
Tienen una moderada actividad sobre
grampositivos, son activas sobre gérmenes
atípicos y no presentan actividad sobre Mecanismo de acción: las quinolonas
anaerobios. interactúan con dos sitios diferentes pero
relacionados, dentro de la célula bacteriana:
Ciprofloxacino es un inhibidor potente del la ADN girasa y la topoisomerasa IV.
citocromo P450 y afecta el metabolismo de la
teofilina y warfarina. La primera es más sensible a la acción de las
quinolonas en caso de gérmenes
En el caso de norfloxacina, las concentraciones gramnegativos, mientras en grampositivos la
en suero y tejidos son bajas, por lo que no se más sensible es la topoisomerasa IV.
usa en infecciones sistémicas, siendo una buena
opción en el caso de infecciones urinarias no Cuando interacciona con la ADN girasa, la
complicadas. inhibición ocurre rápidamente, mientras que
cuando interacciona con la topoisomera IV la
Tercera generación (levofloxacina, inhibición ocurre más lentamente.
gatifloxacina)
Este efecto es debido a la habilidad de las
quinolonas de estabilizar los complejos de
Retienen la actividad sobre gramnegativos y
ADN y topoisomeras II.
mejoran la actividad sobre grampositivos.

55
Manual de INFECTOLOGÍA PLUS MEDIC A

Útil contra los organismos gram-negativos (E. INHIBE LAS VÍAS METABÓLICAS
coli, Salmonella, Shigella, Neisseria, Pseudo -
monas, Haemophilus y Legionella Inhibidores de la síntesis del ácido
TETRAHIDROFÓLICO
Muerte rápida y sin dolor de las bacterias
Las quinolonas impiden la reparación del ADN una vez √ Sulfonamidas
cortado, lo cual conlleva una serie de respuestas que √ Trimetropin
determinan la degradación del genoma y muerte de la
bacteria. SULFONAMIDAS Y TRIMETROPIN
Las dianas
En los bacilos gramnegativos, la diana primaria es la
Cotrimoxazol (1:5 de trimetoprima-sulfame-
girasa, mientras que en los cocos grampositivos es la
toxazol) es el que se indica con más frecuencia.
topoisomerasa IV.
Mecanismo
RIFAMICINAS Las sulfamidas y la trimetoprima interfieren en
la síntesis de los ácidos nucleicos por un
Las rifamicinas se fijan a la subunidad beta de la mecanismo diferente, inhiben la síntesis del
polimerasa del ARN dependiente del ADN y ácido fólico , lo cual interfiere con la producción
bloquean la elongación de ARNm cuando ésta de nucleótidos, especialmente de timina.
alcanza 2 o 3 nucleótidos.
Las bacterias, los protozoos y Pneumocystis
Rifampicina jirovecii son incapaces de obtener ácido fólico
Tiene eficacia notable en las infecciones del entorno y han de sintetizarlo.
caracterizadas por la formación de biopelículas,
como las que asientan sobre biomateriales. Debe
Dihidropteroato
combinarse siempre con otro antibiótico activo,
El dihidropteroato se convierte en ácido
dado que administrada en monoterapia selecciona dihidrofólico y éste, a su vez, en el principio
con facilidad mutantes resistentes. activo, el ácido tetrahidrofólico (ácido folínico),
Rifabutina por la acción de la reductasa de dihidrofolato,
Es más activa que la rifampicina frente a que es la enzima específicamente inhibida por
Mycobacterium avium-intracellulare, porque penetra la trimetoprima.
mejor en la bacteria.
Las sulfamidas son análogos del ácido
Rifapentina paraaminobenzoico y actúan como un
Tiene una vida media prolongada y ha resultado eficaz, falso sustrato de la sintetasa de
administrada una vez por semana, durante la fase de dihidropteroato.
consolidación del tratamiento de la tuberculosis en
pacientes no infectados por el virus de la La acción conjunta de sulfamidas y trimetoprin
inmunodeficiencia humana con formas pulmonares no en las bacterias sensibles es sinérgica
cavitadas y cultivo negativo a los 2 meses de iniciar el
tratamiento. Las células humanas obtienen el ácido fólico
directamente de la dieta y su reductasa de
Rifaximina dihidrofolato es mucho menos sensible a estos
Es un derivado no absorbible que ha demostrado ser antibióticos que la de las bacterias o
efectivo en el tratamiento de la gastroenteritis causada protozoos.
por gérmenes no enteroinvasivos, la
encefalopatía hepática, los síndromes de Efectos adversos
sobrecrecimiento bacteriano del intestino delgado y el
colon irritable. Los inhibidores de la reductasa pueden causar
anemia megaloblástica (sobre todo a dosis
altas) ,

56
Manual de INFECTOLOGÍA PLUS MEDIC A

Puede prevenirse con la administración de √ Anemia macrocítica (deficiencia de folato)


ácido folínico, sin que se afecte la actividad anti- √ Teratogenicidad (deficiencia de folato)
microbiana.
LESIÓN DIRECTA DE ÁCIDOS
Además: NUCLEICOS Y PROTEÍNAS

√ La intolerancia gastrointestinal NITROIMIDAZOLES (Metronidazol)


√ Reacciones alérgicas
√ Efectos sobre el SNC (reducir el umbral Son antibacterianos sintéticos con actividad
convulsivo) inicialmente antiparasitaria (protozoarios) que
se obtuvo de la azomicina (2-nitroimidazol), de la
que luego de su uso clínico en 1959, se
descubrieron propiedades bactericidas ligadas a
microor- ganismos anaerobios.

Metronidazol (5-nitro-imidazol), es uno de sus


más importantes representantes y es
considerado uno de los antibióticos
más valiosos de uso humano.

Mecanismo de acción

Todos los nitroimidazoles actúan por medio de


la reducción del grupo nitro, para formar
compuestos intermedios inestables y tóxicos.
Durante la síntesis de las bases púricas y
pirimidínicas se requieren grupos metilo, El proceso de reducción requiere seis electrones
metileno o formilo. El ácido tetrahidrofólico es y ocurre con potenciales rédox muy bajos, que
el dador de estos grupos tanto en células procariotas solo lo tienen las bacterias anaerobias. Esto
(bacterias y plasmodios) como eucariotas. produce metabolitos ( radical nitro) que dañan el
Θ Dihidrofolato (DHF) sintasa = inhibida DNA bacteriano mediante oxidación y causan
por sulfamidas muerte celular.
ΘDihidrofolato reductasa = inhibida por
trimetoprima Indicaciones
√Buena biodisponibilidad - tan buena como
COTRIMOXAZOL la utilizada para el tratamiento de la infección
por microorganismos anaerobios
Indicaciones
Por ejemplo después de la cirugía, sepsis
√ Tratamiento de la infección del tracto urinario o intraabdominal, heridas infectadas, infecciones
la exacerbación de la bronquitis crónica pélvicas, abscesos, septicemia anaerobia.
-√Toxoplasmosis
También se utiliza para:
-√Pneumocystis jirovecci

Efectos adversos √ Colitis pseudomembranosa por C. difficile

√Malestar y molestias digestivas ( El metronidazol es de primera elección).


√Reacciones tóxico cutáneas incluyendo a síndro-
mes Steven-Johnson y Lyell. √Tricomoniasis
√Hemólisis en pacientes con deficiencia de G-6-P

57
Manual de INFECTOLOGÍA PLUS MEDIC A

√ Amibiasis Los lipopéptidos ejercen acción bactericida


(Dosis: 750 mg VO c/ 8 horas)x10d dependiente de la concentración
√ Giardiasis 250mgVO c/8h x 5d
Los cocientes Cmáx/CMI o ABC/CMI son
Efectos adversos los parámetros farmacodinámicos que
mejor se relacionan con su eficacia.
-La intolerancia gastrointestinal En el caso de la daptomicina, la administración
-Reacciones alérgicas en una dosis única diaria se asocia, además, con
-Efecto disulfiram-like con el alcohol menor toxicidad muscular.
-Sabor metálico Sin embargo, a la dosis máxima recomendada
-Neuropatía periférica en la administración prolon- (6 mg/kg/día) puede ser algo menos eficaz
gada que otros fármacos antiestafilocócicos en
pacientes con bacteriemia por S. aureus,
Actúan en la MEMBRANA EXTERNA debido aparentemente a su mayor facilidad
para seleccionar mutantes resistentes.
de bacterias gramnegativas o en la
membrana CITOPLASMÁTICA
Por último … ¡ la del estribo!
LIPOPÉPTIDOS

Tienen la capacidad de insertarse en las


membranas lipídicas. Este mecanismo de acción
es completamente distinto al de otros
antibióticos y, por tanto, no hay resistencia
cruzada.
NITROFURANTOINA y FURAZOLIDONA
En el capítulo de las bacterias vimos lo siguiente:
Nitrofurantoína y la furazolidona son
√Las bacterias grampositivas tienen una cubierta de
peptidoglucano gruesa situada por fuera de la membrana derivados del nitrofurano que parecen
citoplasmática. actuar tanto en la síntesis proteica como en los
√Los bacterias gramnegativas tienen una capa más fina mecanismos reparadores del ADN bacteriano.
de peptidoglucano, pero disponen además de una Una vez reducidos en el interior de la
membrana exterior de composición asimétrica. La lámina bacteria, pueden :
interna de esta membrana está constituida por √ Unirse a proteínas ribosómicas y bloquear
fosfolípidos, mientras que la externa contiene un lípido la traducción.
glucosilado especial denominado lipopolisacárido o √ En su forma no reducida pueden dañar el
endotoxina. ADN bacteriano.

Daptomicina
Regulación de los antibióticos en ERC
Es un lipopéptido aniónico, activo sólo frente a
bacterias grampositivas, incluidos SAMR , los
RM 2015 –I B (69): ¿Cuál de los siguientes an-
estafilococos resistentes a los glucopéptidos y los tibióticos no requiere ajuste de dosis en pacientes
enterococos resistentes a la vancomicina. con insuficiencia renal severa?
A.- Ticarcilina
Mecanismo de acción B.- Azitromicina
Los lipopéptidos ejercen una acción bactericida
Se debe a la inserción del antibiótico en la
dependiente C.- Amikacina
membrana citoplasmática
de la concentración gracias
y, por tanto, los cocientes al D.- Ceftazidima
establecimiento de puentes de Ca++ con los E.- Levofloxacino
Cmáx/CMI o
fosfolípidos, la probable oligomerización del No requieren regulación en ERC:
ABC/CMI son los parámetros farmacodinámicos que METROnidazol , MACRÓlido, RIFAmpicina ,
fármaco mediada también por el Ca++ y la
mejor se ANFOtericin B, CEFTRIAXONA y CLINDamicina
formación
r
de brechas por las cuales la célula pierde
K+ y otros elementos esenciales.

58
Manual de INFECTOLOGÍA PLUS MEDIC A

59
Manual de INFECTOLOGÍA PLUS MEDIC A

60
Manual de INFECTOLOGÍA PLUS MEDIC A

ANTIMICÓTICOS PLUS MEDIC A

El número de agentes disponibles para el ANFOTERICINA


tratamiento de las infecciones por hongos
(micosis) se ha incrementado a través del tiempo, Anfotericina A y B producidos por
pero son pocos aprobados para uso clínico. Es Streptomyces Anfotericina A no está en uso
importante tener conocimiento detallado de clínico
cada clase de fármaco. La anfotericina B es un macrólido poliénico
anfótero (polieno = contiene muchos enlaces
POLIENOS dobles; macrólidos = que contiene un anillo de
lactona de 12 o más átomos).
La anfotericina B y la nistatina (uso tópico)
Es casi insoluble en agua por lo tanto es
son los polienos disponibles en la actualidad.
preparada como una suspensión coloidal de
Los polienos se unen al ergosterol de la
anfotericina B y desoxicolato sódico para
membrana celular fúngica. Este proceso altera la
inyección intravenosa. Se han desarrollado varias
permeabilidad de la pared celular formando
fórmulas en los que la anfotericina B se envasa en un
oligodendromes funcionando como poros con la
sistema de entrega de lípidos asociados
consiguiente salida de potasio y moléculas
intracelulares que causan muerte del hongo.

61
Manual de INFECTOLOGÍA PLUS MEDIC A

Farmacocinética Candiduria responde a la irrigación de la


vejiga con anfotericina B, y se ha demostrado
Anfotericina B es pobremente absorbida en el que no producen toxicidad sistémica significativa
tracto gastrointestinal (anfotericina B oral es
efectiva sólo para los hongos en el lúmen). Para el tratamiento de la enfermedad fúngica sistémica,
El medicamento se une más del 90% a anfotericina B se administra por infusión IV lenta en
las proteínas del suero. dosis de 0.5 a 1 mg / kg / día.
Aunque es principalmente metabolizado, algo de
anfotericina B se excreta lentamente en la orina OF: El fármaco de elección para el tratamiento de
durante varios días. la HISTOPLASMOSIS es:
El tiempo de vida media es aproximadamente 15 días. A.Fluconazol
B.Anfotericina B
La insuficiencia hepática, insuficiencia renal y C.Itraconzaol
diálisis tienen poco impacto sobre las D.Caspofungin
concentraciones del fármaco E.Ketoconazol
Rpta. B
Sólo 2-3% del medicamento alcanza el líquido
cefalorraquídeo, lo que en ocasiones requiere un Efectos Adversos
tratamiento vía intratecal para ciertos tipos de Toxicidad inmediata
meningitis fúngica. Son casi universales: fiebre, escalofríos, espasmos
musculares, vómitos, cefalea e hipotensión.
Mecanismo de acción
Pueden ser mejorados al disminuir la velocidad de
Altera la permeabilidad de la célula infusión o la disminución de la dosis diaria.
mediante la formación de poros en la
membrana celular, gracias a su característica Toxicidad acumulativa
anfipática. El daño renal es la reacción tóxica más
importante, puede ser lo suficientemente grave
El poro permite la salida de iones intracelulares y como para necesitar diálisis.
macromoléculas, llevando eventualmente a la muerte Anomalías de las pruebas de función hepática se
celular. observa ocasionalmente, al igual que anemia debido a
la reducción de la producción de eritropoyetina.
Aplicación clínica Después de la terapia intratecal con anfotericina,
Anfotericina B sigue siendo un agente se puede desarrollar, convulsiones y aracnoiditis
útil para casi todas las infecciones química a menudo con secuelas neurológicas
micóticas potencialmente mortales. graves.
Existen nuevas preparaciones de anfotericina
A menudo se utiliza como el régimen de liposomal que producen menor toxicidad, permitiendo
inducción inicial con el fin de reducir rápidamente la utilización de dosis mayores.
la carga micótica y luego se sustituye por uno de los
nuevos azoles para el tratamiento crónico o la ANÁLOGOS DE PIRIMIDINA
prevención de la recaída.
FLUCITOCINA
También se puede utilizar para úlceras micóticas
corneales y la queratitis a través de gotas tópicas, La flucitosina es un análogo de pirimidina
así como por inyección subconjuntival directa. soluble en agua relacionado con el agente
quimioterapéutico fluorouracilo (5-FU). Su
Artritis por hongos ha sido tratada con la inyección local. espectro de acción es mucho más estrecho que el de
anfotericina B.

62
Manual de INFECTOLOGÍA PLUS MEDIC A

Farmacocinética
La actividad antifúngica de los fármacos azólicos
Formulación oral, duración : horas, excreción renal. es la reducción de la síntesis de ergosterol
mediante la inhibición de enzimas del citocromo
Se une pobremente a las proteínas, penetra bien en
P450 de los hongos.
los compartimentos incluyendo líquido céfalo-
raquídeo La inhibición de la enzimas del citocromo P450
puede provocar interferencias metabólicas con ciertas
La dosis en pacientes con buena función renal es 100– hormonas o interacciones con fármacos metabolizados
150 mg/kg/d. bajo este sistema.
Mecanismo de Acción
Los preparados triazólicos requieren una
La flucitosina es captada por las células de dosis de carga para lograr concentraciones
hongos a través de la enzima permeasa de plasmáticas estacionarias y terapéuticas.
citosina. Es convertida intracelularmente primero
en 5-FU y luego a 5-monofosfato IMIDAZOLES
fluorodeoxiuridina (FdUMP) y trifosfato
fluorouridina (FUTP​ ​ ), que inhiben la síntesis de
KETOCONAZOL
ADN y ARN, respectivamente.
Sinérgico con anfotericina Farmacocinética : Oral, tópica.
Efectos adversos Mecanismo de acción
◆Toxicidad sistémica en huésped debido a efectos en el ◆ Bloqueo fúngico de enzima P450 que
DNA y RNA. interfiere con la síntesis de ergosterol
◆Mielosupresión ◆ Es pobremente selectivo, interfiere con la
función de P450 de las células de los mamíferos.
Aplicación clínica
Infecciones por Criptococcus y ◆ Amplio espectro pero la toxicidad restringe su
chromoblastomycosis uso para terapia tópica
Efectos adversos
AZOLES Interfiere con la síntesis de hormonas
esteroideas y el metabolismo del fármaco en
Los azoles son compuestos sintéticos que la fase I .
pueden ser clasificados como imidazoles o
triazoles en función del número de átomos de TRIAZOLES
nitrógeno.
Los imidazoles son: ketoconazol, miconazol, ITRACONAZOL
clotrimazol, las dos últimas drogas se utilizan sólo Similar a ketoconazol, más selectivo que
en terapia tópica. Los triazoles son itraconazol, ketoconazol.
fluconazol, voriconazol y posaconazol.

63
Manual de INFECTOLOGÍA PLUS MEDIC A

Farmacocinética El medicamento está disponible en formulaciones orales


◆Vía oral y endovenoso, duración 1-2 días y por vía intravenosa y se utiliza en dosis de 100-
800 mg / d.
◆Pobre entrada al sistema nervioso central.
El fluconazol es el azol de elección en el
Aplicación clínica tratamiento y la profilaxis secundaria de
Amplio espectro: Candida, Cryptococcus, blastomy- la meningitis criptocócica.
cosis, coccidioidomycosis, histoplasmosis Fluconazol por vía intravenosa ha demostrado ser
Efectos adversos equivalente a la anfotericina B en el tratamiento de
Toxicidad baja la candidemia en pacientes en UCI.

FLUCONAZOL El fluconazol es el agente más utilizado para el


tratamiento de la candidiasis mucocutánea.
Farmacocinética
Actividad contra los hongos dimórficos se limita a
Fluconazol no muestra actividad frente a
la enfermedad coccidioidea, y, en particular para
Aspergillus y otros hongos filamentosos.
la meningitis, donde las altas dosis de fluconazol a
Fluconazol muestra un alto grado de solubili- menudo se obvia la necesidad de intratecal anfotericina B.
dad en agua y una buena penetración de líquido
El uso profiláctico de fluconazol se ha
cefalorraquídeo. demostrado para reducir las enfermedades
A diferencia de ketoconazol e itraconazol, su micóticas en receptores de trasplante de
biodisponibilidad oral es alta. médula ósea y pacientes con SIDA, pero la
aparición de hongos resistentes a fluconazol ha
Las interacciones farmacológicas son menos frecuentes expresado su preocupación por esta indicación.
debido a que el fluconazol tiene el menor efecto de
todos los azoles en las enzimas hepáticas Fluconazole tiene excelente penetración al SNC,
usado en meningitis fúngica.
microsomales.
Por la poca interacción de enzimas hepáticas y
una mejor tolerancia gastrointestinal, el EQUINOCANDINAS
fluconazol tiene la más amplia índice
terapéutico de los azoles CASPOFUNGINA
lo que permite la administración más agresiva en Farmacocinética.
una variedad de infecciones fúngicas. Vía oral, duración días.
El medicamento está disponible en formulaciones Mecanismo de acción
orales y por vía intravenosa y se utiliza en dosis de 100- Bloquea la enzima -glucano sintasa y previene la
800 mg / d. síntesis de la pared celular del hongo Candida sp,
también usado en aspergillosis
Aplicación clínica
Efectos adversos
Fluconazol muestra un alto grado de solubili- Menores efectos gastrointestinales, flushing
dad en agua y una buena penetración de Interacciones: Incrementa los niveles de
líquido cefalorraquídeo. ciclosporina.
Por la poca interacción de enzimas hepáticas y una
mejor tolerancia gastrointestinal, el fluconazol Micafungin, anidulafungin: Micafungin
tiene la más amplia índice terapéutico de los incrementa niveles de nifedipino, cyclosporine.
azoles, lo que permite la administración más Anidulafungina es relativamente libre de esta
agresiva en una variedad de infecciones fúngicas. interacción

64
Manual de INFECTOLOGÍA PLUS MEDIC A

ALLYLAMINA
TERBINAFINA

Farmacocinética
Vía oral, duración días.

Mecanismo de acción
◆Reduce ergosterol
◆Previene la síntesis de la membrana celular fúngica
Aplicación clínica
Infección micótica mucocutánea
Efectos adversos
◆Malestar gastrointestinal, cefalea, hepatotoxicidad.
◆No se han reportado interacciones
Of: No es antimicótico sistémico:

a)Anfotericina B
b) Fluconazol
c) Flucitosina
d) Econazol
e) ketoconazol

65
Manual de INFECTOLOGÍA PLUS MEDIC A

DENGUE PLUS MEDIC A

INTRODUCCIÓN ¿Cuáles son las razones?


Las condiciones que permiten persistencia del
Aedes aegypti (también vector de la fiebre
chikungunya):
Clima
√ El cambio climático
Condiciones sociales
√ La escasa disponibilidad de agua para consumo
√ Urbanización no controlada ni planificada
√ Viviendas inapropiadas en centros urbanos
INCIDENCIA √ Inadecuada disposición de residuos
EN LAS ÚLTIMAS CINCO DÉCADAS, la incidencia de √ Uso cada vez mayor de envases no biodegradables en
dengue SE HA INCREMENTADO 30 VECES1 el medio así como neumáticos en desuso
el inadecuado saneamiento ambiental
1960-69
15 497 Migración
1980-89 √ El tránsito urbano, interprovincial y aéreo intenso.
295 554 √ Las intensas migraciones de áreas endémicas a áreas
2000-07 no endémicas de dengue
925 896 √ Persistencia de actividad epidémica en el interior del
2010
país y en los países limítrofes
2 204 516
1.Geneva: WHO; 2012. Demografía
√ El crecimiento poblacional sostenido
◆Se presenta en áreas previamente no afectadas. Política de los gobiernos
◆ Cientos de miles de casos de dengue grave se √ Deficiente coordinación intersectorial y la poca
presentan cada año con cerca de 20 000 participación de organizaciones y población, por
muertes. considerar que el problema del dengue es un problema
del sector salud.

EN EL PERÚ
Reportes sin confirmar
1700, 1818, 1850 y 1876
2.OPS/ OMS. Washington, D.C: PAHO; 2014. Primeros reportes de brotes de un síndrome febril
compatible con dengue clásico .
3.Washington, DC: PAHO; 2001

Reemergencia del dengue en el Perú


Siglo XX
Debido a la REINTRODUCCIÓN DEL AEDES AEGYPTI en
1984
Fue eliminado en 1956

Epidemia de dengue clásico por DENV-1

66
Manual de INFECTOLOGÍA PLUS MEDIC A

1990 («Arbo» acrónimo del inglés “arthropod-borne”,


Amazonía transmitido por artrópodos)
Casos de dengue y circulación de cuatro
serotipos de dengue ◆Familia : Flaviviridae
2015 ◆Género : Flavivirus.
En casi todas las áreas del país con presencia de ◆Serotipos : 1, 2, 3 y 4.
Aedes aegypti . Serotipo 5
HAY UNA TENDENCIA AL INCREMENTO EN LA ÚLTIMA Se ha reportado la probable existencia de un quinto
DÉCADA (AMAZONÍA y LA COSTA NORTE) serotipo viral
7. Science. 2013
Loreto Piura
EL DENV consiste en una molécula de RNA,
2000 2000 Codifica para :
218 2620 ◆3 proteínas estructurales:
2005 2005 C,  prM/M,  E
1772 37 ◆ 7 proteínas no estructurales (NS) en el
2010 2010 siguiente orden :
1322 8390 ◎5’-C-prM-E- ◎NS1 ◎NS2A ◎NS2B ◎NS3 ◎
2014 2014 NS4A ◎NS4BNS5-3’.
7594 2890

Red Nacional de Epidemiología (RENACE)- RM 2013-A (50): Los serotipos del dengue, se
DGE-MINSA. reconocen por la variación de la proteína:
A. C2
Los casos graves y fatales tienen una B. M4
correlación con el ingreso de un nuevo
C. E4
linaje del DENV-2 genotipo america-
no/asiático4 2010 D. C3
E. B4
4.Rev Peru Med Exp Salud Publica. 2011;28(1):157-9.

20115
22 087 casos de dengue sin señales de alarma (88,5%)
2720 casos de dengue con señales de alarma (10,9%)
58 casos de dengue grave (0,6%)
29 fallecidos
5.Bol Epidemiol (Lima). 2011;20(52).

20146
14 537 casos de dengue sin señales de alarma
4296 casos de dengue con señales de alarma 95 casos
de dengue grave
Con una tasa de incidencia de 61,43 x 100 000
habitantes Cualquier serotipo puede producir formas graves de
6.Bol Epidemiol (Lima). 2014;23(22):442-6. la enfermedad, aunque los serotipos 2 y 3 han sido
asociados a la mayor cantidad de casos graves y
fallecidos.
VIRUS DEL DENGUE
El virus del dengue es un arbovirus

67
Manual de INFECTOLOGÍA PLUS MEDIC A

La inmunidad es serotipo-específica
por lo que :
◆ la infección con un serotipo determinado
confiere inmunidad permanente contra el
mismo (inmunidad homóloga)
◆ Sólo por unos meses contra el resto de los
serotipos (inmunidad heteróloga)

GENOTIPOS Y LINAJES DEL VIRUS


DENGUE CIRCULANTES EN PERÚ
Los 4 serotipos de dengue Aedes aegypti
1990 Iquitos
Se introdujo8 el DENV-1 “IDENTIFAC” DEL Aedes aegypti
8. Am Health Organ. 1992;26(3):201-7. ¿Cómo lo identificas?
Mirale las patas y verás sus características rayas negras
1995 y blancas , también en el dorso.
Se introdujo el genotipo americano9 del DENV-2
9. Lancet. 2002 Se puede decir que
las
2001 patas del Aedes son
Fue reconocido10 el DENV-3 muy
10. Infect Genet Evol. 2008 parecidas a la camiseta
del famoso equipo
2008 italiano Juventus o a
Fue reconocido11 el DENV-4 las coquetas medias de
11. Emerg Infect Dis. 2009 la chica de al lado.

2011 El Aedes aegypti es el más “hogareño” de los


Se aisló un nuevo linaje del genotipo mosquitos , no se aleja más de 300 m del lugar de
americano/asiático en la mayor epidemia de su nacimiento.
dengue en Loreto .
El Aedes aegypti
Existe una ALTA VARIABILIDAD GENÉTICA del virus
es muy “casero”
dengue
más casero que
►EL DENV-3 ES MÁS DIVERGENTE con CINCO LINAJES el muchacho de
►EL DENV-4 MENOS DIVERGENTE con DOS LINAJES, la foto.
respecto a los otros serotipos12 Tiene hábitos
12. Tesis para obtener el grado de Doctor. Facultad de domiciliarios,
Medicina, Universidad Nacional Mayor de San Marcos. por lo que la
Lima, Peru. 2013. transmisión es
predominante -
TRANSMISIÓN mente doméstica.
Vector: mosquito Aedes aegypti
¿A qué hora atacan los
Se desarrolla en envases caseros y en recipientes
mosquitos para preparar
extradomiciliarios .
mi matamosquito?

68
Manual de INFECTOLOGÍA PLUS MEDIC A

Pueden picar a cualquier hora del día aunque Forma de transmisión y patogenia
generalmente lo hacen en la mañana y en horas de la
tarde. ¿Cómo ingresa el virus en el organismo humano?
En algunas ocasiones, el Aedes aegypti se alimenta ◆El Aedes aegypti infectado al picar a una persona
en los ambientes interiores durante la noche si hay le introduce el virus dengue (vía subcutánea) .
luces encendidas.
¿Una vez que ingresa el virus del dengue a donde
se dirige?
CICLO BIOLÓGICO del mosquito
Ganglios linfáticos regionales
◆EL virus invade las células de Langerhans , las cuales
1.La hembra busca recipientes con agua.
2.Huevos: pone los huevos en la zona húmeda
los trasportan a los ganglios linfáticos regionales para
de la pared del recipiente. la presentación de antígeno a los linfocitos T y B
(respuesta inmune adaptativa).
3.Larvas: Cuando los huevos se mojan,
En la sangre periférica
comienzan su desarrollo y luego de 2-3 días se
Invade los monocitos
convierten en larvas .
4.Pupas: luego de 5 a 10 días en el agua, las ◆ También puede invadir otras células del organismo:
hepatocitos, neumocitos tipo II, fibras cardíacas, células
larvas pasan al estado de pupas.
dendríticas, células endoteliales y plaquetas
5.Forma adulta: las pupas finalmente se
transforman en mosquitos adultos.

Ciclo biológico del mosquito


◆El virus invade los monocitos
Casos “autóctonos” son aquellos en los que
la transmisión se produjo en el área, lo que Luego de un periodo necesario para el desarrollo de la
indica la presencia del vector infectado. infección viral en el mosquito (periodo de
incubación extrínseco), éste permanecerá
Los casos “importados” son aquellos en los infectante el resto de su vida y con capacidad de
que el paciente adquirió la enfermedad en un infectar a individuos susceptibles.
área distinta de la que reside o se hizo el La duración de este periodo es variable y,
diagnostico. entre otros factores, depende de la temperatura media
ambiental.

69
Manual de INFECTOLOGÍA PLUS MEDIC A

También puede aparecer en la etapa crítica en


Cuadro clínico forma tardía entre el 6º y 9º día que se localiza en
las palmas de las manos y las plantas de los pies,
La infección por dengue puede ser subclínica o
asociado a un prurito severo.
puede causar una enfermedad que puede ir de una
leve a una severa intensidad.
Forma grave (dengue grave)
◆Período de incubación: 5 a 7 días, √Hemorrágicas
(Se han reportado casos con un período de √Pérdida de plasma debida al aumento de la
incubación de 3 hasta 14 días) -RM 10-A (76)- permeabilidad vascular, (lo que ocasiona
hipovolemia relativa y un aumento del
RM 10-A (76): ¿Cuál es el periodo de hematocrito).
incubación del dengue en el ser humano? √Colecciones líquidas en cavidades serosas
a.Más de un mes (derrame pleural, ascitis y derrame pericárdico)
b.20 - 30 días √ Puede presentarse un cuadro de shock.
c.3 - 10 días
d.6 meses ◆ Clasicamente se conocía que los casos de
e.24 horas dengue grave se presentaban en personas que ya
Rpta. C padecieron dengue por un serotipo (infección
primaria) y se infectan nuevamente (infección
◆ Cuadro clínico: luego del PI podrán aparecer las secundaria) con un serotipo diferente.
manifestaciones clínicas.
◆ Sin embargo la infección primaria puede dar
Sin embargo un porcentaje alto de enfermos
cursarán asintomáticos. lugar a dengue grave, lo que dependerá de la
virulencia de la cepa o inmunodepresión del
Infecciones sintomáticas: hospedero.
 Forma leve : sólo se manifiestan con un cuadro
febril agudo, de duración limitada (2 a 7 días).
ENAM 04-B (87) ; Essalud 05 (87): La fiebre
Formas moderadas hemorrágica por virus del dengue está asociada a:
Además de la fiebre el paciente presentará un gran A. Mayor virulencia de unas cepas que otras
compromiso general caracterizado por malestar B. Infección previa por otra cepa del mismo virus
general, cefalea, dolor retro ocular, dolor muscular C. Plaquetas en sangre normales
y poliartralgias. D. Convulsiones tónico clónicas
E. Secuela neurológica
Exantema (50%) Rpta. B
En el 3er. día desde el inicio de las manifestaciones Puede presentarse además difunción orgánica: encefalitis,
clínicas en la mitad de los casos se observa la miocarditis, hepatopatía e insuficiencia renal aguda.
aparición de un exantema no patognomónico. Al inicio de la fiebre, no es posible predecir si el
paciente tendrá síntomas y signos leves todo el tiempo, o
si evolucionará a una forma grave de la enfermedad.
Por lo tanto:
El equipo de salud debe realizar un seguimiento
estricto del paciente, y éste junto a su grupo
familiar deben estar atentos a los signos de
Exantema pruriginoso escarificado alarma, controlando a todo paciente con dengue
asociado a dengue hasta al menos 48 horas de finalizada la etapa febril.

70
Manual de INFECTOLOGÍA PLUS MEDIC A

ETAPAS DE EVOLUCIÓN DEL DENGUE A la caída de la fiebre, el enfermo puede


mejorar o empeorar. El empeoramiento es precedido
por uno o más signos clínicos conocidos como signos de alarma,
ETAPA FEBRIL
ya que anuncian la inminencia del shock.
◆Duración: entre 3 a 6 días en niños y 4 a 7 días en
adultos. Los signos de alarma son:
En esta estapa la persona es una “fábrica” de virus de
1. Dolor abdominal intenso y sostenido.
dengue el cual circula en el torrente sanguíneo (viremia) ,
2. Vómitos persistentes.
de tal forma que si la persona es picada por un mosquito
vector , al picar a otra persona estará transmitiendo la
3.Derrame seroso (en peritoneo, pleura o
enfermedad. pericardio) detectado por clínica, por
laboratorio (hipoalbuminemia) o por
◆ Síntomas : el paciente presenta fiebre con imágenes (ecografía de abdomen o
escalofríos más los siguientes síntomas:
radiografía de tórax).
4.Sangrado de mucosas
√Dolor osteo-muscular (es conocida como la fiebre
5.Cambio en el estado mental del paciente:
“rompe huesos” )
somnolencia o irritabilidad.
√Poliartralgias
6.Hepatomegalia (> 2 cm).
√Cefalea y astenia.
7.Si está disponible: incremento brusco del
√Exantema, prurito.
hematocrito concomitante con rápida dismi -
√Síntomas digestivos:
nución del recuento de plaquetas.
dolor abdominal leve y, a veces, diarrea.

¿Para qué identificar precozmnte los signos de


Algunos pacientes pueden desarrollar alarma?
manifestaciones hemorrágicas leves tales • Para rehidratar precozmente y adecuadamente por
como epistaxis, gingivorragias, petequias, vía EV, con lo cual se previenen el shock.
púrpuras o equimosis, SIN que correspondan a un
cuadro de dengue grave.
Los sangrados ginecológicos, tanto la menorragia
como la metrorragia, pueden ser de intensidad
variable.

◆Exámenes auxiliares: es frecuente la presencia de


leucopenia con linfocitosis relativa, trombocitopenia e
incremento de las transaminasas.
El período durante el cual se produce la caída de la
fiebre y hasta 48 horas después, es el momento en el
que, con mayor frecuencia, los enfermos pueden
presentar complicaciones,
La extravasación de plasma se hace más
intensa y es capaz de conducir al shock
por dengue.

Durante o después del shock es el momento


cuando se producen con mayor frecuencia las
grandes hemorragias.

Etapas de la evolución del dengue

71
Manual de INFECTOLOGÍA PLUS MEDIC A

ETAPA CRÍTICA

En esta etapa se presenta un aumento de la


permeabilidad capilar lo que da lugar a una
trasudación de plasma hacia el espacio intersticial,
que puede llevar al shock hipovolémico.
La trasudación de líquido origina un incremento de la
fase sólida del plasma lo que determina un aumento del
hematocrito .

Aumento del hematocrito


◆ Es un marcador bastante confiable de la
disminución del volumen plasmático efectivo.

¿Cómo evoluciona el shock?


◆Puede durar algunas horas.
¿A qué se debe la trombocitopenia?
Se debe a un fenómeno autoinmune que consiste
◆Puede ser prolongado o recurrente
en la destrucción de las plaquetas debido a la acción
(más de 12 ó 24 horas y, excepcionalmente, más de 48
de anticuerpos antivirales que por reacción
horas).
cruzada atacan a las plaquetas.
Pueden presentar complicaciones tales como:
√ Hemorragias masivas
ETAPA DE RECUPERACIÓN
√ Falla multiorgánica
√ Coagulación intravascular diseminada (CID) Generalmente el paciente evoluciona con franca
recuperación pero puede ocurrir un estado de
Indicador incial del shock sobrecarga de volumen, además de un
◆Un signo que se avecina la “tormenta” (shock) es : sobreinfección alguna infección bacteriana.
la disminución de la presión arterial diferencial en
≤ 20mmHg
◆ Además de los signos de hipoperfusión como
Exámenes de laboratorio
sudor,frialdad y taquicardia. El hematocrito y el recuento de plaquetas son
marcadores de la evolución de la enfermedad.
Se recomienda un buen monitoreo de los pacientes
El aumento del hematocrito nos indica disminución del
ya que el retardo del diagnóstico de la etapa crítica. volumen plasmático efectivo y la disminución
aumenta la mortalidad. progresiva de las plaquetas nos indica agravamiento de la
enfermedad.
Trombocitopenia
La trombocitopenia que se inicia desde la etapa En el plan de trabajo se deben considerar los
febril suele agravarse en la etapa crítica. exámenes basales bioquímicos y hematológicos:
factores de coagulación, proteínas totales, albúmina,
El agravamiento de electrolitos séricos, AGA, urea, creatinina y
la trombocitopenia transaminasas.
es un marcador de Los estudios por imágenes (radiografía de tórax,
ecografía) tienen por finalidad detectar en forma
empeoramiento del precoz la presencia de líquido a nivel de las
dengue. serosas (derrame pleural,pericárdico o ascitis).
La ecocardiografía no solamente puede detectar la
presencia de líquido en el espacio pericárdico sino
también complicaciones como una miocarditis.

72
Manual de INFECTOLOGÍA PLUS MEDIC A

Dengue grave- criterios


Clasificación del Dengue Sensibilidad 95% y especificidad de 97%.
En las últimas décadas se ha venido
√Extravasación grave de plasma: presencia de shock
utilizando la clasificación acuñada
por la OMS : hipovolémico, y/o por dificultad respiratoria debida al
exceso de líquidos acumulados en el pulmón.
√Fiebre del dengue (FD) √Hemorragias severas
√Fiebre hemorrágica por dengue (FHD) con o sin síndrome √La afectación de órganos: hepatitis grave por
de shock por dengue (SCD). dengue (transaminasas superiores a 1000 unidades),
encefalitis o afectación grave de otros órganos, como la
¿Cuáles son los inconvenisntes de la clasificación miocarditis.
anterior?
√Está demasiado condicionada a resultados de
laboratorio.
√No es inclusiva de enfermos con dengue con otras
formas de gravedad, tales como la encefalitis,
miocarditis o hepatitis grave.
√No es de mucha utilidad para el manejo clínico de los
pacientes con dengue.

Estudio DENCO (Dengue Control) -OMS .


Objetivos:
√Mejorar la clasificación del dengue. EsSalud 05: Marque cuales son los signos de
√Identificar los signos de alarma con la finalidad de alarma a tener en cuenta en el paciente con dengue
mejorar el manejo de casos de dengue. A.Hemoconcentración
B.Hepatomegalia > 2 cm
Nº de casos: C.Somnolencia
2.000 casos de dengue confirmado en 7 D.Todas son correctas
países de dos continentes.
CASO PROBABLE de Dengue
Conclusiones: ◆FIEBRE de menos de 7 días de evolución, sin
√El 18% y hasta el 40% de los casos no podían ser afección de las vías respiratorias superiores ni otra
categorizados mediante la actual clasificación. etiología definida.
√Más del 15% de los casos con shock no podían ser ◆ Vive en, o ha viajado a, un área con
clasificados como casos graves de dengue, transmisión actual de dengue.
dado que no cumplían con los criterios de síndrome de ◆Además de:
shock por dengue (FHD/SCD). 2 ó más de los siguientes signos:
√Anorexia y náuseas
Nueva clasificación propuesta por el estudio
DENCO: √Erupciones cutáneas
 Dengue c/s signos de alarma √Cefalea y/o dolor retro-ocular
Otra contribución del estudio DENCO es : √Malestar general, mioartralgias
la identificación de signos de alarma como √Leucopenia, trombocitopenia
predictores de severidad temprana (gravedad) . √Petequias o prueba de torniquete positivo
√Diarrea y vómitos

73
Manual de INFECTOLOGÍA PLUS MEDIC A

PASO Nº1: Buscar signos de alarma


Una vez que se tiene un caso sospechoso de
dengue , a continuación hay que establecer si
cursa o no con signos de alarma.

Dengue SIN signos de alarma


(Crupo A)

El manejo es ambulatorio.
SI hay comorbolidades de fondo o de riesgo
social que modifiquen el tratamiento se debe
hospitalizar al paciente.

Condiciones co-existentes:
Embarazo, menores de 3 meses, adultos
mayores (70 años y más), obesidad, diabetes mellitus,
cardiopatías, otras condiciones clínicas (hemoglobinopatías,
Prueba del torniquete
etc.)
OF: ¿Ante cuales de las siguientes situaciones Riesgo social:
Usted sospecharía en Dengue? Vivir solo, difícil acceso al hospital, pobreza extrema, otros.
A.Paciente de cualquier sexo y edad que resida o
haya viajado a área endémica que presente fiebre En base a lo anterior el dengue puede clasificarse
de 10 días de evolución con afección de vías aéreas de la siguiente manera:
◆ Paciente con dengue SIN signos de alarma y
superiores.
SIN condiciones co-existentes.
B.Paciente de cualquier sexo y edad que resida en
◆Paciente con dengue SIN signos de alarma y
área endémica sin fiebre sin afección de vías aéreas
CON condiciones co-existentes.
superiores con anorexia,nauseas y leucopenia.
C.Paciente de cualquier sexo y edad que resida o
haya viajado a área endémica con rínorrea, diarrea, RM 12-B (29): Paciente de 13 años, 15 días antes
viajó a Piura. Presenta fiebre desde hace 5 dias,
vómitos y rash en tórax. artralgia, cefalea, dolor retro ocular y erupción
D.Paciente de cualquier edad y sexo que resida o cutánea. ¿Cómo se clasifica el caso?
haya viajado a un área endémica que presente A.Probable dengue con signos de alarma
fiebre de menos de 7 días de duración, sin afección B.Probable dengue grave
de las vías aéreas superiores con cefalea y C.Probable dengue sin signos de alarma
mioartalgias. D.Dengue confirmado
Rpta. D E.Dengue descartado
Rpta. C
CLASIFICACIÓN de CASO PROBABLE
de DENGUE Dengue CON signos de alarma
Según la Guía Técnica: (Grupo B)
“Guía de Práctica Clínica para la atención de
◆Si uno o más de los signos de alarma que
casos de dengue en el Perú”
se mencionaron antes está presente, el
(MINSA 2011)
paciente con Dengue se clasifica como dengue con
signos de alarma y es necesario referirlo a
un Hospital.

74
Manual de INFECTOLOGÍA PLUS MEDIC A

PASO Nº2: En el Dengue con signos de alarma • Si el resultado es negativo no se descarta el


determinar si es dengue grave o no diagnóstico.
Los pacientes con dengue con signos de alarma Se debe solicitar una nueva muestra de suero
pueden clasificarse en: del paciente con más de 5 días de iniciados los
◆ Paciente con dengueCON signos de síntomas
alarma y (en lo posible, la segunda muestra debe ser tomada
◆Paciente con dengue grave
aproximadamente 10 a 15 días del inicio de los
síntomas).
Los criterios para determinar si un paciente con
dengue con signos de alarma se clasifica o no como
dengue grave se presentan a continuación: ◎ Si el resultado de la segunda muestra es
negativo, el caso es negativo.
DENGUE GRAVE ◎Si el resultado de la segunda muestra es
(Grupo C) positivo
se realiza también una prueba de
Criterios: Neutralización en el suero de la primera
Uno o más de los siguientes hallazgos: muestra.
√Shock hipovolémico por fuga de plasma Si el resultado de este estudio es
√Distres respiratorio por acumulación de negativo, se confirma la seroconversión y se
líquidos considera un caso confirmado.
√Sangrado grave
√Daño orgánico importante ◆ Si la muestra es tomada a partir de los 5
días de iniciados los síntomas:
CONFIRMACIÓN DEL DENGUE Se realiza detección de anticuerpos IgM de virus de
dengue
Se considera :
Caso Confirmado a todo paciente sospechoso que tenga un • Si el resultado es negativo
diagnóstico confirmatorio de dengue por laboratorio. Se descarta el diagnóstico de dengue.
Se considera Confirmado también por nexo epidemiológico, • Si el resultado es positivo
Se pide una segunda muestra para confirmar
según la situación epidemiológica.
mediante prueba de Neutralización.
Confirmación del diagnóstico de dengue
Los métodos dependerán de la situación epidemiológica En un área CON confirmación de brote de
de la región, y del momento entre el inicio de los dengue
síntomas y la toma de la muestra: En un área en la que ya se ha confirmado un brote
de dengue:
En un área SIN confirmación de brote de El diagnóstico de laboratorio se realiza para vigilancia
dengue y se no se considera para el manejo de los casos.
◆ Si la muestra es tomada antes de los 5 días de ◎ Sólo se debe tomar muestras para el diagnóstico
iniciados los síntomas: etiológico a una pequeña proporción de los
Se realiza aislamiento del virus y/o detección del casos sospechosos a fin de :
genoma viral de muestras de suero y/o tejidos. ►Monitorear la duración del brote en el tiempo y
►Vigilar la potencial introducción de nuevos serotipos
√Si el resultado es positivo, se confirma en el área.
el caso de dengue. ◎ También debe realizarse diagnóstico
etiológico a todos aquellos casos graves y/o mortales
atribuidos al virus dengue.
75
Manual de INFECTOLOGÍA PLUS MEDIC A

En pacientes post mortem:


Caso de Dengue SIN signos de Alarma
√Aislamiento viral y/o detección de genoma (GRUPO A)
viral por PCR en muestras de suero obtenida
por punción cardiaca, o tejidos. √Reposo aislado en cama con aislamiento del
√Serología IgM e IgG paciente para evitar las picaduras de los mosquitos.
(en par de sueros). √Adecuada ingesta de líquidos (1 ½ l ó más por día)
√Inmunohistoquímica en muestras de tejidos √Paracetamol 0.5-1 g PO c/6 h (máximo 4g)
en formol bufferado.
NO INDICAR :
Tratamiento del paciente con Dengue .AAS ni otros AINES
.Antibióticos
13.Guía técnica: guía de práctica clínica para la EVITAR la vía intramuscular
atencion de casos de dengue en el Perú MINSA
2013 √Dar información oral y escrita sobre los signos de
alarma.
Manejo según nivel de complejidad y √Control diario
capacidad resolutiva: √Buscar signos de alarma en cada consulta hasta 48
horas después de la remisión de la fiebre.
Caso de Dengue SIN signos de Alarma
(GRUPO A)
Exámenes de laboratorio
Estos pacientes pueden ser manejados en el -Hemograma diario o cada 2 días.
hogar -Tiempo de protrombina (TP)
◎ Si toleran volúmenes adecuados de líquidos por -Tiempo de Tromboplastina Parcial Activada
la vía oral ( TTPA)
◎Han orinado por lo menos una vez cada 6 horas
◎No tienen signos de alarma y se descarta en ellos la Pacientes con dengue SIN signos de alarma CON
existencia de co-morbilidad y riesgo social. condiciones co-existentes o riesgo social

El tratamiento de este grupo de pacientes puede ser


Caso de Dengue CON signos de Alarma hospitalario ya que los pacientes con dengue sin signos de
(GRUPO B) alarma que tienen condiciones co-existentes (como por
◎ Pacientes que deben ser inmediatamente ejemplo, embarazo) o que tienen riesgo social, pueden
hospitalizados, sin embargo el tratamiento con requerir una atención diferente que, en muchos casos, no
fluidos por vía endovenosa debe iniciarse en el es factible brindar en el domicilio.
lugar en donde se identifican los signos de alarma.
Caso de Dengue Grave (GRUPO C) Caso de Dengue CON signos de Alarma
◎ Pacientes que requieren tratamiento de emer - (GRUPO B)
gencia
◎ Después de estabilizado el choque, deben ser √ El tratamiento de estos pacientes debe ser siempre
referidos a una unidad especializada de manejo de hospitalario ya que los signos de alarma son
paciente crítico (UCI). indicadores indirectos de la existencia de
extravasación de líquidos.
EN PACIENTES CON DENGUE SIN SIGNOS
√Obtener Hto antes de proceder a la rehidratación.
DE ALARMA Y, SIN CONDICIONES CO -
EXISTENTES

76
Manual de INFECTOLOGÍA PLUS MEDIC A

√En el caso de que el paciente con dengue no tolere la En pacientes adultos:


vía oral, se indica hidratación endovenosa . El primer paso es iniciar una carga con
√Administrar solución salina 10ml/Kg/h por 1 hora. cristaloides
◎Solución salina isotónica al
√Si persisten los signos de alarma repetir el
procedimiento 1 ó 2 veces más. 0.9% o
√Si hay mejoría de los signos de alarma y disminución ◎ Rínger Lactato
del Hto: a 20 ml/kg en 15-30 minutos.
Reducir el goteo a 5-7 ml/Kg/día por 2-4 h.
√Si continúa la mejoría reducir a 3-5ml/Kg/día por 2-4
h.
Posteriormente se debe evaluar al paciente :
√Si continúa la mejoría :
√Si el paciente mejora:
reducir la dosis a 2-3ml/Kg/d por 24-48 h más e iniciar l
Se puede seguir con :
avía oral.
solución salina isotónica al 0.9% o Rínger Lactato a
√Si hay un empeoramiento de los signos vitales y/o 10 ml/kg por 1 hora.
aumento del Hto manejarlo como Dengue GRAVE.
Si sigue la mejoría después de la hora, continuar
√Si el empeoramiento cursa con disminución del Hto reduciendo el aporte como en el dengue con signos de
considerar una hemorragia. alarma.
RM 12-B (26): Según la Guía Técnica del MINSA √Si el paciente no mejora y el Hto sigue alto:
para el manejo del Dengue - grupo B, ¿qué acción Se le debe repetir la carga con solución salina
sanitaria corresponde? isotónica al 0.9% o Rínger Lactato
A.Manejo ambulatorio a 20 ml/kg en 15-30 minutos y reevaluar:
B.Referirlo para manejo hospitalario
C.Tratamiento de emergencia √ Si el paciente mejora:
D.Tratamiento higiénico dietético Se debe seguir con cristaloides a 10 ml/kg en 1 hora
E.Administración de medicación vía IM y seguir como en el tratamiento de pacientes con
Rpta. B dengue con signos de alarma.
√Si el paciente no mejora y el Hto sigue alto:
Caso de Dengue Grave (GRUPO C) Se debe iniciar el aporte de coloides a 10-20 ml/kg
En los casos en los que el paciente presenta signos de en 30-60 minutos y reevaluar:
gravedad, se debe rehidratar enérgicamente al √Si el paciente mejora:
paciente y evaluar estrechamente su evolución Cambiar a cristaloides a 10 ml/kg en 1 hora y seguir
controlando: como en el caso de tratamiento de pacientes con
• Signos vitales dengue con signos de alarma.
• Tiempo de llenado capilar √Si el paciente no mejora
• Hematocrito Se debe continuar con coloides a 10-20 ml/kg en 1
• Diuresis. hora más y reevaluar.
√Si el paciente no mejora
Si el paciente se encuentra en un servicio de salud Se debe considerar el uso de drogas vasoactivas.
ambulatorio: √Si el paciente no mejora y el Hto baja
Inicie la expansión endovenosa antes y durante la √Se debe pensar en la presencia de sangrado (casi
referencia al hospital. siempre el sangrado es digestivo)
√ Siempre se debe obtener Hto antes de expandir al √Se debe indicar la transfusión de glóbulos rojos, en
paciente forma urgente.
(excepto que se encuentre en un servicio de salud √El tratamiento de hemorragias severas se hará de
ambulatorio y no tenga laboratorio) acuerdo al criterio clínico.

77
Manual de INFECTOLOGÍA PLUS MEDIC A

En niños con shock ◎ Manejo adecuado de recipientes con aguas


Se iniciará la expansión con cristaloides estancadas y para que adopte además conductas de
bolo inicial de 20 ml/kg. autocuidado, como el uso de :
Si el niño se estabiliza: ►Mosquiteros en puertas y ventanas y el uso de
la pérdida es menor al 20%. repelentes.

√ De persistir los signos de shock


Repetir bolo de 20 ml/kg.
◆Si el niño se estabiliza en forma transitoria :
la pérdida suele ser entre el 20-30%
Estos pacientes requerirán una transfusión con
glóbulos rojos desplasmatizados a 10 ml/kg.
◆ Si luego del segundo bolo de cristaloides el niño
persiste en shock: Para aquellas localidades en las que exista escasez de
la perdida de volumen es mayor al 30- 40%, por agua, se debe proporcionar información sobre
lo que estos pacientes requerirán reposición inmediata de
cómo almacenar agua de manera adecuada: en
sangre y mayor cantidad de cristaloides.
recipientes bien tapados, sin orificios por donde puedan
ingresar los mosquitos.
PREVENCIÓN DE DENGUE EN LA FAMILIA Y LA
COMUNIDAD La vigilancia de los Síndromes Febriles
Con relación al control del mosquito y los criaderos Por la dificultad del diagnóstico de dengue, en zonas
El control del vector, una vez notificados los casos de de riesgo es de suma importancia hacer vigilancia de
dengue, está en general a cargo de organismos los síndromes febriles inespecíficos.
municipales, provinciales o nacionales. Esta vigilancia permitiría identificar los casos de
Estos organismos deben realizan tareas de bloqueo dengue que han desarrollado las formas menos graves o no
mediante la aplicación de : hemorrágicas de la enfermedad.
►Larvicidas y
►La fumigación con insecticidas. Aislamiento entomológico de los pacientes

Es muy importante que el equipo de salud: Informar a la población acerca de la importancia de evitar
que aquellos pacientes enfermos de dengue sean picados
◎ Verifique que los responsables de las actividades de por los mosquitos mientras se encuentren febriles, para
control del vector estén realizando las tareas evitar la propagación de la enfermedad utilizando barreras
correspondientes en el área en la que reside el caso como telas mosquiteras.
detectado.
◎Informe y estimule a la población a realizar actividades ENAM 08-A (28): En zonas endémicas el recojo de
que controlen el desarrollo de los mosquitos. inservibles y la búsqueda activa de aguas
◎Se debe brindar información a la población sobre : estancadas (botellas, floreros, tanques) se realiza
►El mosquito y su proliferación para detectar la presencia del vector del:
► El modo de transmisión y A. Plasmodium vivax
►Los métodos de prevención. B. Virus del dengue
◎ Se debe estimular a la población para que adopte C. Virus de la hepatitis B
hábitos y prácticas que reduzcan el riesgo de convivir con D. Clostridium tetani
los mosquitos, como: E. Vibrion choleare
◎Recolección de residuos Rpta. B

78
Manual de INFECTOLOGÍA PLUS MEDIC A

Bibliografía

1a.Guía técnica: guía de práctica clínica para la atencion


de casos de dengue en el Perú MINSA 2018

1.World Health Organization. Global strategy for


dengue prevention and control 2012-2020. Geneva:
WHO; 2012

2.PAHO/WHO. Los casos de dengue en las Américas


se quintuplicaron en diez años, según nuevos datos de
la OPS/ OMS. Washington, D.C: PAHO; 2014
Complicaciones (cronología)
3. Schneider J, Droll D. A timeline for dengue in the
ENAM 2017 –A (88): ¿En qué dias de la Americas to december 31, 2000 and noted first
evolución natural del dengue se presentan las occurences. Washington, DC: PAHO; 2001
complicaciones de la enfermedad?
4.Durand-Velazco S, et al. Impacto de la epidemia de
a. Los 3 primeros dengue con un nuevo linaje del DENV-2 Genotipo
b. Después de los 15 Americano / Asiático en la demanda de servicios del
hospital de Apoyo de Iquitos “Cesar Garayar García”.
c. 7° a 9° días Rev Peru Med Exp Salud Publica. 2011;28(1):157-9.
d. En el 4° y 5° día
e. 10° a 14° días 5. Perú, Ministerio de Salud. Incidencia acumulada de
dengue sin señales de alarma, con señales de alarma y
Rpta D dengue grave por regiones en el Perú 2011. Bol
Epidemiol (Lima). 2011;20(52).

6. Perú, Ministerio de Salud. Resumen de las


enfermedades o eventos bajo vigilancia epidemiológica
en el Perú. Bol Epidemiol (Lima). 2014;23(22):442-6.

7. Normile D. Tropical medicine. Surprising new


dengue virus throws a spanner in disease control efforts.
Science. 2013

8. Phillips I et al. First documented outbreak of dengue in


the Peruvian Amazon region. Bull Pan Am Health Organ.
1992;26(3):201-7.

9. Kochel TJ et al. Effect of dengue-1 antibodies on


American dengue-2 viral infection and dengue
haemorrhagic fever. Lancet. 2002 Jul 27;360(9329):310-
2.

10. Kochel T, et al. Molecular epidemiology of dengue


virus type 3 in Northern South America: 2000--2005.
Infect Genet Evol. 2008 Sep;8(5):682- 8. doi:
10.1016/j.meegid.2008.06.008.

79
Manual de INFECTOLOGÍA PLUS MEDIC A

11. Forshey BM, et al. Dengue virus serotype 4,


northeastern Peru, 2008. Emerg Infect Dis. 2009
Nov;15(11):1815-8

12. Mamani E. Identificacion de genotipos y linajes de


los cuatro serotipos del virus dengue en el Peru durante
los anos 1998-2012. Tesis para obtener el grado de
Doctor. Facultad de Medicina, Universidad Nacional
Mayor de San Marcos. Lima, Peru. 2013.

80
Manual de INFECTOLOGÍA PLUS MEDIC A

Sd MONONUCLEÓSICO PLUS MEDIC A


Introducción La edad en que se presenta la infección es
directamente proporcional a las condiciones
El Síndrome mononucleósico abarca sanitarias
diferentes entidades clínicas con distintos
√En poblaciones de condición socioeconómica baja: es
agentes causales :
más frecuente en los primeros años de la vida.

a)Mononucleosis infecciosa” (MI) propiamente dicha


√En las poblaciones con mejores condiciones de sanidad,
la infección por EBV ocurre en adolescentes y adultos
relacionada con el virus de Epstein-Barr (SM-VEB)
jóvenes (15-24 años).
b) Otras enfermedades que no están producidas por el
VEB (SM-nVEB) pero que se asemejan a la MI y que en
conjunto se han denominado “síndromes mononucleosis- Primer pico: En menores de 5 años
like” (Hurt C, 2007).
La transmisión se produce a
través de fómites
Epidemiología contaminados con saliva.

El virus de Epstein-Barr En los niños el cuadro clínico es atípico y puede


infecta el 90% de la po- cursar como una enfermedad exantemática
blación mundial, usual - inespecífica.
mente su transmisión
es a través del beso (al
Segundo pico: En la segunda década de la
compartir la saliva).
vida
Portador : infección persistente asintomática en la
orofaringe e infección latente de por vida en La transmisión se produce por contacto íntimo
linfocitos tipo B. (“enfermedad del beso”), en condiciones de
Puede ser transmitido por contacto sexual (se ha hacinamiento.
aislado el virus en secreciones genitales.
El cuadro es mucho más intenso y la
seroconversión con aparición de anticuerpos
¡Te pareces tanto a heterófilos ocurre en las primeras dos semanas.
él ……......!
pero no te preocu-
pes el Ebstein
Barr sólo infecta ¡Ampay !
al ser humano. El beso es el me-
canismo de trasmi-
sión de la MI

Entre el 30-50 % de los infectados desarrollan la


enfermedad
La infección primaria por EBV ocurre durante la SEROCONVERSIÓN
infancia siendo generalmente asintomática, en la √En lugares con malas condiciones higiénicas se
adolescencia se presenta como Mononucleosis presume que hasta 95% de la población haya
infecciosa. seroconvertido en la infancia.

81
Manual de INFECTOLOGÍA PLUS MEDIC A

√En los países desarrollados puede ser positiva en 2ºCitomegalovirus (CMV) (5-7%)
más de 50% a la edad de 25 años ( en mayores de 35 Puede ser responsable del 7%, el cuadro clínico es
años es muy rara). muy similar al ocasionado por EBV.

La diseminación por fómites en adultos no es Ambos son agentes infecciosos oportunistas en el


común por tratarse de un virus muy lábil en paciente inmunocomprometido.
fuentes ambientales. Otras causas
※Síndrome agudo de VIH
※Adenovirus
La transmisión de persona a persona :
※Virus Herpes Humano 6
√Portadores asintomáticos (10-20% de la
población) ※Sarampión alemán
√Portadores transitorios, en quienes la ※Hepatitis viral
enfermedad pudo manifestarse o no, y quienes ※Faringitis por EBHGA
pueden persistir como tales, produciendo virus ※Toxoplasmosis
activos hasta por 18 meses. Posterior a su replicación inicial en la faringe, el
EBV infecta a los linfocitos B a través de su receptor
Etiología CD21, distribuyéndose en todo el sistema
linforeticular.
El agente etiológico más frecuente de la MI es
el virus Ebstein Barr o herpes virus humano Cuadro clínico
tipo-4 [HHV-4]
◎ Período de incubación: de 30 a 50 días (adultos) y
1ºVirus Ebstein Barr (EBV) (80-90%) puede disminuir a la mitad del tiempo en los niños
√Gama-herpes virus.
√Tropismo : por el epitelio escamoso de la ◎Manifestaciones clínicas:
orofaringe, el epitelio ductal parotideo, linfocitos B La MI es un síndrome clínico caracterizado por la TRIADA
y epitelio escamoso cervical uterino. (98%):
√La infección puede generar linfoma de Burkitt y
carcinomas nasofaríngeos. Faringitis (69-91%)
Fiebre (63-100%)
Adenomegalia (93-100%)

√La fiebre persiste durante 10-14 días; sólo 2% y 1%


de los casos persisten con fiebre a los 2 y 6 meses,
respectivamente.
√La adenomeaglia es inguinal, axilar y retro-
auricular, levemente dolorosa . Linfadenopatía cervical
se mantiene en 55%, 39% y 29% a 1, 2 y 6 meses
Virus Ebstein Barr después del diagnóstico.
√ La faringitis persiste en 47% y 21% de los casos a 1,
En segundo lugar está el Citomegalovirus.
2 y 6 meses,respectivamente.

82
Manual de INFECTOLOGÍA PLUS MEDIC A

Los signos con mayor especificidad, en orden


decreciente, son:

√Esplenomegalia (99%)
√Petequias en paladar (95%)
√Adenopatía axilar (91%)
√Adenopatía cervical posterior (87%)
√Temperatura corporal ≥ 37.5 C (84%)
√Adenopatía inguinal (82 %)

Triada de la MI

La triada clínica se considera como un Petequias en el


síndrome mononucleósico. paladar

Además de la triada, los pacientes presentan:


√Esplenomegalia entre el 50% -75%
√Hepatomegalia en 72-87.5%. Exámenes auxiliares
Con predominio de esplenomegalia en adolescentes y
adultos jóvenes sobre la hepatomegalia. Los tres criterios clásicos de laboratorio para la
La hepatoesplenomegalia remite en la mayoría de los confirmación de mononucleosis infecciosa son:
casos al mes de evolución. √Linfocitosis
√Linfocitos atípicos en (≥10%)
Otros signos:
√Prueba de Paul Bunnel (+)
√Petequias en el paladar (50 %)
Las petequias se observan entre la unión del paladar
blando y el paladar duro
Duración: entre 3-4 días.

Signo de Hoagland
Edema periorbitario, especialmente de los párpados
superiores
Puede ser encontrado en aproximadamente 30% de
los pacientes en los primeros días de la
infección.

Cuando se asocia linfocitosis > 50% con la presencia


de > 10% de linfocitos atípicos la sensibilidad es
de 61% con especificidad de 95% para el diagnóstico
de MI.

Hemograma
◎Linfocitos atípicos
Las características morfológicas de estos
Signo de Hoagland linfocitos atípicos son:
Θ Incremento en el tamaño

83
Manual de INFECTOLOGÍA PLUS MEDIC A

Θ Núcleos grandes con disminución de la razón Puede presentarse también trombocitopenia leve a
núcleo/citoplasma moderada.
Θ Apariencia de menor densidad del núcleo en
comparación con los linfocitos normales. Prueba de Paul Bunnel
Esta prueba detecta anticuerpos no específicos,
denominados heterófilos:
√ Son producidos por la activación policlonal de
linfocitos B infectados y no están dirigidos contra el EBV
√ Reaccionan con antígenos que se encuentran en
eritrocitos de otras especies animales llevando a su
aglutinación.
◆ La prueba de Paul-Bunnell determina anticuerpos
Linfocitos atípicos heterófilos a partir de la 2 semana de la infección.
◆ El nivel máximo de anticuerpos es detectado por esta
Son células características de la MI, en su técnica a la 6a semana y niveles bajos hasta un año
mayoría linfocitos CD8 activados en forma posterior a la infección primaria.
policlonal.
Dependiendo de la técnica de dilución,
Si bien los linfocitos atípicos pueden encontrarse títulos mayores de 1:40 o 1:28 se consideran
en otras enfermedades como : como positivos.
√ Solo el 10-30% de los niños < de 2 años
Infección por CMV, rubeola, toxoplasmosis, desarrollan anticuerpos heterófilos durante la
hepatitis viral, parotiditis y roseola. primoinfección
√ 50-75% de los niños de 2-4 años desarrollan
◎Leucocitosis o leucopenia anticuerpos heterófilos durante la primoinfección.
√Leucocitosis entre 10 y 20 mil/mm3 (40 a 70%).
Más común en pacientes menores de 2 años. Dado que la infección por EBV es perene,
existen diversos sistemas de anticuerpos
√Leucopenia: menos frecuente. contra diferentes antígenos que pueden
indicar un determinado estadio de la infección.
◎Linfocitosis Θ Los antígenos tempranos (EA) son
Se presenta en el 80-90% de los pacientes producidos en los estados iniciales de la
Es común encontrarla ≥50% y los linfocitos atípicos infección lítica del virus, antes de la síntesis del
≥10% DNA.
Incrementándose en la segunda y tercera semana de Θ Los antígenos tardíos son producidos
la enfermedad, con duración en algunos casos hasta 6 después de la síntesis de DNA e incluyen los
semanas. antígenos capsulares (VCA) que son las proteínas
Cabe señalar que en su mayoría estos estructurales de la misma.
linfocitos En sangre periférica son Θ Durante la infección latente solo pocos
linfocitos T CD8 maduros con función antígenos son producidos y entre ellos el EBNA1.
supresora o citotóxica,la mayoría de ellos
dirigidos contra las células B infectadas por el Inmunofluorescencia (IF)
mismo EBV. ΘSe han utilizado diversos formatos para la
identificación de anticuerpos contra EBV
Hay además una neutropenia absoluta y relativa en Θ La prueba más sensible hasta el momento la
60-90% de los casos. inmunofluorescencia (IF)

84
Manual de INFECTOLOGÍA PLUS MEDIC A

Θ La técnica de ELISA tiene una sensibilidad √ Neuritis óptica


adecuada para la mayoría de los anticuerpos (90%) y
especificidad (95%), lo cual permite una Las complicaciones agudas y graves con riesgo de
determinación rápida para un mayor número de muerte, son poco frecuentes, entre ellas:
muestras. √ Ruptura esplénica (0.5-1%)
√ La infección aguda por EBV es caracterizada por √ Obstrucción grave de las vías respiratorias debido a
un incremento rápido en los anticuerpos dirigidos hiperplasia linfoide y edema de mucosas (1%)
contra VCA. √ Síndrome hemofagocítico desencadenado por EBV
√ La elevación de IgM es primaria pero √ Síndrome de Guillain-Barré
generalmente transitoria, detectándose usualmente √ Meningoencefalitis
por 4 semanas pero puede perdurar hasta por 3 √ Meningitis aséptica
meses.
√ Mielitis transversa
La determinación de IgM-VCA es el ensayo
Neoplasias malignas asociadas
serológico más específico
indica una infección aguda y es suficiente
※Linfoma de Burkitt
para confirmar el diagnóstico. ※Cáncer de CAVUM
※Linfoma B Hodgkin
La elevación de IgG -VCA usualmente es
máxima durante el estadio agudo de la El linfoma de Burkitt es una rara forma
enfermedad, declinando gradualmente en los meses de cáncer del sistema linfático—asociado a linfoci-
siguientes y permaneciendo estable durante el resto tos B—descrita más frecuentemente en África
de la vida. central, parece estar asociada con
◆ La ausencia de estos anticuerpos (anti -EBNA) en la infección del virus de Epstein Barr.
presencia de IgM y/o IgG VCA, indican una
infección relativamente reciente.
◆ La presencia de anticuerpos indica una infección
previa en cualquier momento de la vida.

Complicaciones
Cáncer de cavum
Hematológicas (0,5-3% %)
√ Anemia hemolítica Pronóstico
√ Trombocitopenia
√ Síndrome urémico hemolítico El pronóstico de la MI es favorable en la mayoría de
√ Ccoagulación intravascular diseminada los casos, incluyendo a los pacientes que
presentan complicaciones leves.
Digestivas
√ Hepatitis anictérica (50-80%) Tratamiento
√ Hepatitis ictérica (5%)
◆ Mantener hidratación adecuada
Neurológicas (1-5%) ◆ Anti-inflamatorios no esteroideos: Ibuprofen o
√ Parálisis del nervio facial paracetamol.
√ Neuritis periférica
√ Cerebelitis

85
Manual de INFECTOLOGÍA PLUS MEDIC A

◎ Estos medicamentos son eficaces, bien ※Anemia hemolítica autoinmune


tolerados y seguros para el tratamiento de la fiebre y ※Meningitis.
dolor en niños y adultos. Prednisona 1mg/k/d (máximo 60mg/d) o un corticosteroide
◎ Se debe evitar el uso de ASA en niños por el equivalente por 7 días seguido de un régimen de disminución
riesgo de Síndrome de Reye. gradual otros 7 días.
Se han publicado casos con trombocitopenia severa
No prescribir antibióticos, especialmente igual o menor a 3000/mm3 asociada a MI, el uso de
Ampicilina y Amoxicilina, y en general corticoesteroides con incremento paulatino de
todos los beta-lactámicos.
semanas antes que incremente la cuenta plaquetaria sobre
Considerar que los pacientes con MI que reciben este
30,000/mm3.
tipo de antibióticos se complican con
exantema.
Inmunoglobulinas
Se ha utilizado Inmunoglobulina en casos refractarios a
Aciclovir
corticoesteroides y pulsos de metilprednisolona en
El aciclovir es una droga que inhibe la síntesis de DNA
estados clínicos con riesgo de muerte.
viral en forma competitiva.
Algunos de los virus del grupo herpes son sensibles La evidencia es insuficiente para recomendar
y su administración limita su cuadro clínico. el uso rutinario de esteroide e
inmunoglobulina en forma aislada ó en
El aciclovir no altera el curso clínico de conjunto con antiviral en los casos de MI.
la enfermedad.
Sin embargo su uso debe ser individualizado de acuerdo
◆ En enfermedad grave, como obstrucción de la
al riesgo de muerte por complicación de MI versus efectos
vía aérea la administración de esteroides y
aciclovir mostro mejoría evidente. secundarios.
◆ La excreción faríngea de partículas virales
se reduce en forma significativa al final de la ENAM 03-A ( 5): El diagnóstico clínico de
terapia con aciclovir mononucleosis infecciosa por virus Epstein-
Esta excreción se reinicia y es similar a los grupos Barr, incluye fiebre y ...
comparados a la semana de haber terminado la A. Faringitis, adenomegalia
terapia. B. Artralgias
C. Faringitis, visceromegalia
En los pacientes con MI grave, que cursen con D. Artralgias, visceromegalia
obstrucción de la vía aérea E. Adenomegalia, visceromegalia
Evaluar el tratamiento con el uso Rpta. A
concomitante de aciclovir y esteroides.
ENAM 03-A (81) : La infección por el virus de
Dosis: Epstein Barr se ha asociado a los siguientes
10 mg/kg/dosis 3 veces al día o procesos neoplasicos malignos: :
800 mg Vía Oral, 5 veces al día por 10 días. A. Linfoma de Hodgkin – carcinoma basocelular
B. Sarcoma de Kaposi – leucemia
Corticooides C. Linfoma de Burkitt – Carcinoma nasofaríngeo
Sin embargo los cursos cortos (2 semanas) de D. Carcinoma epidermoide - linfoma de Hodgkin
corticoesteroides se han indicado para algunas E. Carcinoma basocelular - leucemia
complicaciones específicas de MI: Rpta. C
※Obstrucción de vía aérea
※Trombocitopenia complicada con sangrado,

86
Manual de INFECTOLOGÍA PLUS MEDIC A

Bibliografia
1.Balfour HH )r et al. Age-specific prevalence of
Epstein-Barr virus infection among individuals aged 6 -
1 9 years in the United States and factors affecting its
acquisition. ) Infect Dis. 20 1 3 Oct 1 5;208(8) : 1 286-
93. [PMID: 23868878]

2. Draborg AH et al. Epstein-Barr virus in systemic


autoimmune diseases. Clin Dev Immunol. 2013; 20 1
3 :535738. [PMID: 24062777] Green M et al.

87
Manual de INFECTOLOGÍA PLUS MEDIC A

FIEBRE AMARILLA PLUS MEDIC A


Es una enfermedad infecciosa aguda, febril, ocasionada Fue el primer virus en el que se demostró la
por el virus del mismo nombre, un Arbovirus, cuyos transmisión por mosquitos.
órganos blanco principales son el sistema fagocítico
mononuclear y el hígado. √ Familia Flaviridae
√ Género Flavivirus, del que forman parte los agentes
Historia responsables del dengue y la fiebre del Nilo occidental.
√ Carlos Finlay, médico La familia Flaviviridae, cuyo prototipo es el virus de
la
cubano, en 1881 pos- FA (flavus: amarillo) comprende unos 80 virus, cuarenta
tuló la trasmisión de la de los cuales producen enfermedad en humanos, veintidós
FA por mosquitos. son transmitidos por mosquitos, trece por garrapatas y
cinco no tienen vector conocido.

√ Walter Reed norteamericano demostró lo que había √ Serotipo del virus de la FA : sólo existe
postulado Finlay en 1900 , además la presencia de un uno.
agente filtrable en la sangre de los pacientes. Se han identificado cinco genotipos diferentes, tres en
√ Mahafy y Bauer lograron en 1927, la transmisión del África y dos en Suramérica, todos los cuales producen el
virus a macacos y el aislamiento del virus, lo cual dio mismo espectro de enfermedad.
origen a la cepa Asibi, utilizada en la preparación de la
vacuna contra la FA.
Epidemiología
√ Max Theiler en 1927 y 1931 realizó trabajos de
investigación que permitieron una estrategia vacunal , lo
◆Prevalencia:
que le permitió ganar el premio Nobel.
La Organización Mundial de la Salud calcula que que se
La combinación de la estrategia vacunal con producen en el mundo doscientos mil casos y 30 000
las campañas de control del vector dio como muertes por FA, sobretodo en Africa subsahariana.
resultado la eliminación de la FA en casi todo
el mundo, con excepción de algunas regiones ◆Ciclos de transmisión
de África, Suramérica y el Caribe. La FA tiene dos ciclos de transmisión
√ De 1988 a 1991. se produjo la reemergencia de la diferentes:
enfermedad por un descuido en los programas de
-El selvático y el urbano.
vigilancia y control.
Ciclo selvático
Se mantiene de manera enzoótica entre :
Etiología Θ Monos infectados y
El virus de la FA fue el primer agente filtrable Θ Mosquitos Haemagogus spp. en América y Aedes
identificado como causante de enfermedad en los pp. en África.
humanos y el primer Flavivirus aislado y cultivado. El hombre se infecta al incursionar en este ciclo, en el que es
picado por los mosquitos infectados.

En la selva el mono es el
reservorio del virus de la
FA
◎Manechi (mono aullador)
◎Marimono (mono araña)

88
Manual de INFECTOLOGÍA PLUS MEDIC A

Vector selvático ◎ La sangre de los enfermos es infectante desde el


(jungla) día antes del inicio de los síntomas y hasta el tercero
(Haemagogus a quinto día de enfermedad.
jantinomys, Sabethes), Lo anterior orresponde al período de viremia (período
que también transmiten en que el virus permanece en la sangre).
el virus ◆ El virus se multiplica en el tejido linfático
transováricamente, regional
viven en las copas de los ◆ Desde allí el torrente hemolinfático lo lleva a
árboles diversos sitios como hígado, bazo, riñones, médu- la
ósea, músculo cardíaco, músculos esqueléticos y,
Ciclo urbano ocasionalmente, cerebro.
Sucede en áreas más pobladas y generalmente es
trasmitido por Aedes aegypti. En el hígado
Se produce cuando personas o mosquitos infectados El hígado es el órgano blanco principal de la agresión viral,
penetran en las áreas pobladas, dando origen a la donde se presentan tempranamente cambios citopáticos
transmisión mosquito-humano-mosquito. en los hepatocitos.
En el bazo
EsSalud 2013 (58): El vector transmisor de la fiebre En el bazo ocurre necrosis linfocítica en los centros
amarilla urbana es: germinales periarteriolares.
En los ganglios linfáticos
A) Anopheles En los ganglios linfáticos hay necrosis de los centros
B) Lutzomya verrucarum germinales y se multiplica ampliamente en los linfocitos
C) Aedes aegypti y los monocitos.
D) Haemagogus spegazzinil En los riñones
E) Triatoma infestans El daño de los riñones se produce tempranamente por
Rpta C cambios en el flujo sanguíneo renal
Secundarios a hipovolemia, mientras que en forma tardía se
Ciclo intermedio debe a necrosis tubular, ocasionada por el colapso
Acontece en las sabanas húmedas y semihúmedas circulatorio.
africanas, durante las estaciones lluviosas, con
aumento de la densidad de vectores y producción de Se produce trastormos ácido-básicos ,
ciclos epidémicos y zoonóticos. hidroelectrlíticos, en el músculo cardíaco
La urbanización de la FA puede ocurrir a partir y en la función cerebral.
tanto del ciclo selvático como del intermedio. Diátesis hemorrágica
Ocurre :
◆ Disminución de los factores de los factores de
Patogénesis coagulación dependientes de vitamina K, debido a
Luego de cumplir su ciclo en la hembra del la lesión hepática
mosquito, el virus es inoculado por vía ◆ También puede ocurrir coagulación intravascular
intracapilar a un individuo susceptible.
diseminada (CID) y alteración de la función
◆ Período de incubación: Varía de 3 a 6 días
plaquetaria.
después de la picadura del mosquito infectante.
Algunas infecciones producidas en el laboratorio
presentan un período de incubación de hasta 10 Manifestaciones clínicas
días. Formas clínicas:
◆Asintomáticas (40-80%)
◆Período de transmisibilidad: ◆Formas clínicas leves
◆Formas fulminantes.

89
Manual de INFECTOLOGÍA PLUS MEDIC A

Fig 1 Cuadro clínco de la FA

◆Sintomáticas
Las sintomáticas se clasifican como:
ΘLeves
ΘModeradas
ΘGraves o malignas
►Su letalidad varía de 5 a 10%
►Si se tienen en cuenta los casos ictéricos, aumenta hasta ►Si hay fiebre ...la FC debe
aumentar
20% y llega a 50% cuando se consideran los casos graves
►Si Lionel Messi patea un
que requieren hospitalización. penal ...si o si , debe ser gol
!Sin embargo a veces esto
◆Sintomáticas no ocurre!
Se dividen en:
ΘInfección Casos leves
ΘRemisión Pueden presentarse como con un aparente
ΘIntoxicación. resfriado común.
Fase de infección Casos graves
◆ Comienzo : súbito Cefalea intensa, fiebre alta, signo de Faget
◆ Fiebre, escalofríos, cefalea, malestar ge- (bradicardia a pesar de la fiebre), náuseas, vómito
neral, congestión conjuntival, dolor lumbar y negro y uno de los tres signos cardinales de la FA
vómitos; Duración: aproximadamente tres días.
Signo de Faget Signos cardinales :
Se caracteriza por bradicardia relativa o sea OLIGURIA, ICTERICIA Y HEMORRAGIA
una disociación pulso temperatura. El aumento de la
temperatura no ocasiona un aumento de la Puede haber :
frecuencia de pulso. ◆ Inyección
Generalmente cuando hay fiebre por cada grado que conjuntival y
aumenta la temperatura corporal le corresponde un congestión facial
aumento de 15 pulsaciones (máscara amarilla),

90
Manual de INFECTOLOGÍA PLUS MEDIC A

Factores de mal pronóstico


√Fiebre alta comienzo de la enfermedad
√Paso rápido al período de intoxicación
√Aumento de la bilirrubina
√CID
√Hipoglicemia
√Hiperpotasemia
√Acidosis metabólica
◆ Fotofobia y lengua saburral, afilada y enrojecida en la √Necrosis tubular aguda
punta y en los lados. √Hipotensión arterial de inicio temprano
Esta etapa dura de uno a tres días. √Choque
√Coma
Fase de remisión
Se caracteriza por mejoría transitoria de los síntomas, √Convulsiones
disminución de la fiebre y ausencia de ictericia. √Hipo de difícil tratamiento y respiración de
Dura de uno a dos días √Cheyne Stokes.
Puede pasar inadvertida y en ella puede remitir la enfermedad.
Los pacientes hospitalizados deben ser
Fase de intoxicación protegidos con mosquiteros durante los primeros
Coincide con la : cinco días de la enfermedad y se deben tener en
√ Desaparición de la viremia y cuenta las llamadas precauciones estándar.
√ Aparición de los anticuerpos y se presenta aproxi - La duración promedio de la enfermedad es de unos
madamente en 15% de los pacientes sintomáticos. dieciocho días y la de la hospitalización es de dos
Se presenta entre el tercero y el sexto días de la semanas.
enfermedad. ◆Período de convalescencia
Puede ser moderada o grave y se caracteriza por: Los pacientes que sobreviven entran en un período de
cefalea global intensa, malestar general, fiebre muy alta convalecencia caracterizado por astenia, adinamia,
(39 a 40ºC), signo de Faget, náuseas, vómito negro, pero con recuperación paulatina y total de
las ,funciones orgánicas.
epigastralgia, ictericia, oliguria y diátesis hemorrágica.

En los casos graves ocurre disfunción orgánica Diagnóstico


múltiple :
Hepática, renal, cardiovascular, encefalopatía y
Caso probable
Paciente con :
coagulopatía
Entre el quinto y el décimo días de la enfermedad. ◆ Cuadro febril agudo (hasta 7 días), de
inicio súbito, acompañado de:
En estos casos hay acidosis metabólica, hipoglicemia,
◆Ictericia
hiponatremia, delirio, estupor o coma, convulsiones o
◆manifestaciones hemorrágicas
hipotensión refractaria. independientemente del estado vacunal para fiebre
amarilla.
Exámenes auxiliares
◆ Paciente con cuadro febril agudo (hasta 7
Las aminotransferasas de aspartato (AST) y las
días), ◎ Residente o proveniente de un área con
transferasas de alanina (ALT) se elevan.
evidencia de transmisión viral (aparición de casos
En nocasiones se aumentan más las AST, debido
al daño cardíaco y musculoesquelético. humanos, epizootias o de aislamiento viral en
mosquitos) en los últimos 15 días
◎ No vacunado contra fiebre amarilla o con estado
Pueden presentarse también otras complicaciones como
vacunal desconocido.
arritmias cardíacas, infecciones del tracto
gastrointestinal y respiratorio y parotiditis.

91
Manual de INFECTOLOGÍA PLUS MEDIC A

Caso confirmado Diagnóstico diferencial


Todo caso probable con al menos una de En los casos leves o moderados debe establecerse con:
las siguientes condiciones: Influenza, dengue clásico, malaria, fiebre tifoidea y las
◆Criterio por laboratorio infecciones de los tractos digestivo, respiratorio o urinario.
Diagnóstico virológico: En los casos graves deben tenerse en cuenta otros
√ Aislamiento del virus del suero del enfermo por diagnósticos diferenciales como:
cultivo en células c6/36 o por inoculación Leptospirosis, malaria, dengue hemorrágico, hepatitis
intracerebral a ratones viral, sepsis con ictericia y las fiebres hemorrágicas
Lo cual se consigue con mayor facilidad durante los suramericanas (producidas por los virus Junín, Machupo y
primeros 4 días de enfermedad. Guanarito).
√ Detección de ácido nucleico viral mediante
técnica de PCR. Prevención
Diagnóstico serológico
√ Demostración de IgM específica contra el virus Prevención primaria
de fiebre amarilla mediante una técnica de Elisa de En las zonas de alto riesgo se basa en :
captura (MAC, Elisa). √ Uso de ropas que cubran la mayor parte del cuerpo y
La IgM se puede demostrar dentro de los 5 días de de colores que no atraigan los mosquitos (rojo, verde o
comenzada la enfermedad e igualmente 5 días azul).
después de la aplicación de la vacuna. √ Uso de sustancias repelentes y mosquiteros.
√ Demostración de anticuerpos anti fiebre amarilla Son muy importantes las medidas tendientes a eliminar
mediante fijación del complemento, inhibición de los criaderos de las larvas (floreros, llantas, latas, tiestos) y
la hemoaglutinación, pruebas de neutralización combatir los insectos adultos.
e inmunofluorescencia indirecta.
Para los primeros es particularmente útil el estudio de
Prevención secundaria
sueros pareados con demostración de un aumento de
√ Consiste en la vacunación.
4 veces el título de anticuerpo entre el suero inicial y
otro tomado 2 a 4 semanas después.
Vacunas antiamarílicas
Diagnóstico patológico
la vacunación se constituye en la principal
Hallazgos histopatológicos con necrosis me-
estrategia de prevención y control de la
diozonal o necrosis masiva y estudio
fiebre amarilla
inmunohistoquímico que revele presencia de
antígenos virales.
√ Es un virus vivo atenuado, cepa 17d o asibi.
Caso confirmado por nexo epidemiológico √ Eficacia : 99%
◆Un individuo asintomático u oligosintomático detectado en Permite la formación de anticuerpos protectores
búsquedaactiva sinantecedentevacunal quepresente serología después de 7 a 10 días de haber sido recibida.
(MAC-Elisa) positiva para FA. √ Se recomienda un refuerzo pasados 10 años
◆Correspondea todo caso probable de fiebre amarilla que Aunque los anticuerpos pueden persistir hasta por 35
Fallece antes de 10 días sin confirmación por labo- años o incluso de por vida.
Ratorio duranteelinicio o cursodeunbrote en que otros casos ya La vacuna se mantiene liofilizada a 4º C y una vez
hansido confirmados. reconstituida debe aplicarse preferiblemente en la primera
Caso descartado hora, pues el título viral decrece rápidamente porque el virus
◆ Caso probable con diagnóstico por laboratorio es termolábil.
negativo , siempre que se compruebe que las muestras fueron Después de reconstituida debe ser mantenida a temperaturas
tomadas y transportadas adecuadamente; o un caso probable con preferiblemente alrededor de 2º C, y protegidas de la luz. Se
diagnósticoconfirmadodeotraenfermedad. inyecta por vía subcutánea.

92
Manual de INFECTOLOGÍA PLUS MEDIC A

que una vez que se restablezca el suministro se regresará a las


Contraindicaciones prácticasconvencionales.
► No se recomienda usar la vacuna en :
×Pacientes HIV+ Tratamiento
×Inmunocomprometidos ►No existe tratamiento antiviral.
×Embarazadas ►Por tanto, la terapia es sintomática, dirigida a la
En quienes no se debe emplear ninguna vacuna con virus corrección de las complicaciones y al alivio de los síntomas.
vivo.
►Tampoco debe aplicarse a niños menores de un año, Bibliografía
mayores de 60 años y pacientes con historia de 1.World Health Organization. Wkly Epidemiol Rec
hipersensibilidad al huevo de gallina o sus derivados. 2017; 92: 345-350
2.World Health Organization. Wkly Epidemiol Rec
Recomendaciones para la vacunación 2. 2014; 89:297-308
√ Vacunar a 100% de los habitantes mayores de 3.PROTOCOLO DE VIGILANCIA Y CONTROL
un año residentes en las zonas con evidencia de
DE FIEBRE AMARILLA. INS
circulación viral.
√ Vacunar a todos los viajeros que se desplacen a
las zonas con evidencia de circulación viral.
√ Evitar la penetración de susceptibles en los
lugares donde se ha presentado recientemente un caso.
√ Considerando las características ambientales (índices
de infestación aédica, desplazamiento de personas), se
debe vacunar en los centros urbanos próximos
infestados con A. aegypti en los casos de riesgo de
urbanización de la fiebre amarilla.
√ Se debe desencadenar una intensificación de
vacunación y realizar acciones de emergencia de
control del vector A. aegypti

Dosis fraccionadas de vacuna


Scientific Advisory Group of Experts (SAGE) en octubre
de
2016
◆ La base científica del uso de fraccionadas (off-
label) es desde la perspectiva teórica, la potencia
mínima para dosis de vacuna no debiera ser
menos de 1000 UI y las vacunas convencionales
contienen concentraciones mucho mayores.
Se espera, por tanto, que la dosis
reducida sea equivalente a la dosis
convencional en términos de seguridad,
inmunogenicidad y efectividad.

Estas vacunas no sirven, al menos de momento, como


estrategia a largo plazo o para reemplazar las prácticas rutinarias
de vacunación (inmunización de viajeros a áreas endémicas), por lo

93
Manual de INFECTOLOGÍA PLUS MEDIC A

ENFERMEDAD DE CHAGAS PLUS MEDIC A


En las heces encontró unos :
Definición ►Protozoos flagelados
►De alrededor de 20-30 micrones
La Trypanosomiasis americana o enfermedad de
Chagas es una infección parasitaria ► Muy móviles
Que tenían un tremendo parecido al tripanosoma
√ Causada por el Trypanosoma cruzi
africano.
√ Transmitida por insectos hematófagos de la familia
Reduviidae REDUVIIDAE conocidos común- mente, en
la región suroccidental del Perú, como chirimachas.

Historia
Tripanosoma cruzi
Carlos Chagas fue un médico
Brasileño experto en malaria Seguidamente examinó la sangre de todos los animales
que a inicios del siglo XX se domésticos de las casas donde había encontrado a los
encontraba haciendo estudios insectos infectados.
sobre malaria en el estado
brasileño de Minas Gerais . Después de examinar a muchos perros final-
mente encontró el protozoario en la
En cierto momento de- sangre ……!de un gato!.
cidió estudiar al
Triatoma infestans co-
nocido en la zona como
“barbeiro”
Le conocen como “barbeiro” porque acostumbraba a
esconderse en las grietas de las paredes de las casas
rurales y al amanecer cuando las personas aún dormían les
picaban en la cara.

Seguramente Chagas pensó que ese insecto también


era un vector de la malaria.

Pero lo que encontró al examinar las


heces del insecto fue sorprendente.

1909
Chagas examina a :
►Una niña llamada Berenice
►De dos años de edad, con fiebre y hEpato-
esplenomegalia clínicamente diagnosticada de malaria.

94
Manual de INFECTOLOGÍA PLUS MEDIC A

El hallazgo ,al examinar la sangre de la niña, fue realmente La incidencia de la enfermedad en toda
sorprendente ,encontró los mismos protozoarios que había América Latina se ha reducido en más del
aislado en las heces de los triatoma infestans. 65%
Por su parecido con el tripanosoma africano lo
denominó Tripanosoma cruzi ( cruzi en honor a su ► La prevalencia en niños y jóvenes ha disminuido
colega y amigo Oswaldo Cruz). ► La distribución de esta infección se localizaría en
población adulta, quienes se encontrarían en la fase
indeterminada o crónica.

Efecto Supermán
En esta historia Carlos
Chagas hizo las cosas
al revés primero des-
cubrió el agente etioló -
gico de una enferme-
dad que no se conocía
y luego descubrió la en-
fermedad. Enfermedad de Chagas en américa

Enfermedad de Chagas en el Perú


Caso de Tahuamanú (primer caso diagnosticado en
el Perú)
“Efecto superman” En 1919, el Dr. Escomel describe un caso humano de
Enfermedad de Chagas procedente de Tahuamanú del
En la investigación médica el proceso es inverso. Departamento de Madre de Dios.
primero se descubre la enfermedad y luego el agente
etiológico. Enfermedad de Chagas en el Perú precolombino
Estudios de momias demuestran lesiones cardiacas y
Chagas hizo las cosas al revés. Esto se conoce digestivas producidas por el Trypanosoma cruzi.
como el ´ Efecto superman” . Las evidencias más antiguas provienen de las momias
Superman es el único hombre que al vestirse hace de Chinchorro, en Perú, con dataciones de 9.000 años,
descriptas por Aufderheide y colaboradores en el
las cosas al revés , primero se pone el pantalón y
2004
después el calzoncillo.

Epidemiología
La enfermedad de Chagas es endémica en muchos países
de América Latina
Se extiende desde México hasta el sur de Argentina.
En esta región existen :
► 4,8 y 5,4 millones de personas muestran cuadros
atribuibles a la Enfermedad de Chagas Existe una momia procedente del Cuzco que fue llevada a
► 25 millones están en riesgo de adquirirla.
Florencia durante la época colonial, en la cual se
encuentran las alteraciones llamadas megaesófago y
megacolon.

95
Manual de INFECTOLOGÍA PLUS MEDIC A

Asimismo el estudio histológico del corazón permite, por


microscopía de luz y electrónica corroborar la presencia de
Etiología
nidos de amastigotes.
El agente etiológico es el Trypanosoma cruzi
► Protozoo flagelado
Enfermedad de Chagas en el Perú actual.
El área chagásica más importante y conocida del Perú se ► De la familia Trypanosomatidae
encuentra en : ► De la superclase Mastigophora
La Macro Región Sur ► Del orden Kinetoplastida.
(región sudoccidental). El microorganismo es :
√ Forma: alargado, fusiforme
La podemos llamar también región del “TAMI”: √ Tamaño: aproximadamente 20 micras
√ Posee un núcleo grande cerca de la parte central
Tacna √ A lo largo de su cuerpo tiene una membrana
Arequipa ondulante bordeada por un flagelo que se inicia en el
cinetoplasto y que sale del parásito.
Moquegua
Estadíos:
Ica
Tripomastigote
-Forma: alargada
Sobretodo en áreas rurales
-Núcleo: central
►Las viviendas de adobe tienen grietas donde
se esconde el triatoma infestans. -Quinetoplasto: organelo muy prominente en su
extremidad posterior del cual emerge un flagelo que
bordea una membrana ondulante hasta su
salida por la extremidad anterior.
-Tamaño: de 20 micrones
-Forma: de una “C o S”.

Esta forma es muy móvil


Arequipa es el departamento que presenta
el mayor porcentaje de población en En preparaciones en fresco, de sangre de animales o
riesgo de adquirir esta infección. humanos, el desplazamiento del parásito se detecta por el
movimiento de los glóbulos rojos.
Estudios realizados en el valle de Vítor han
reportado índices epidemiológicos que evidencian la
presencia de Enfermedad de Chagas en esta zona.

ENAM 06-B (5) : La enfermedad metaxé metaxénica de Tripomastigote


alta prevalencia en la regió
región sudoccidental del
Perú
Perú es: El trypomastigote se encuentra en la sangre
A. Leishmaniosis periférica del hombre o de los animales
B. Paludismo infectados y en las heces del insecto vector,
C. Enfermedad de Carrión considerándosele la forma infectante.
D. Dengue
E. Enfermedad de Chagas El trypomastigote no se reproduce y permanece
Rpta. E viable en la sangre de donantes infectados, aún
conservada a temperatura de refrigeración.

96
Manual de INFECTOLOGÍA PLUS MEDIC A

Epimastigote Triatoma infestans


Es similar al trypomastigote Son los triatominos más comunes.
Se diferencia porque : Es el principal vector desde la línea ecuatorial hacia el
Es más largo y el quinetoplasto está siempre cerca al Sur, y Rhodnius prolixus y Triatoma dimidiata al
núcleo. norte de la línea ecuatorial.
►El flagelo bordea la membrana ondulante y emerge
por la extremidad anterior.
►El epimastigote se encuentra en el intestino del
vector, donde se reproduce por división binaria y
también es la forma predominante en el cultivo del
parásito.
►Es más alargado y el nucleoplasto está en el mismo
extremo del flagelo.
◆ Tamaño : de los adultos varía entre 1,5 y 2 cm de
longitud y el color es variable según las especies.
La cabeza es alargada y termina en una
probóscide recta que durante el reposo se dobla en
ángulo agudo contra la parte ventral del cuerpo, que se
Epimastigote extiende en el momento dela picadura.

Amastigote ◆Tórax : es quitinoso y su segmento anterior o pronoto


Es una célula redonda, muy pequeña, de 2-4 micrones de tiene forma de escudo.
diámetro, presenta núcleo y quinetoplasto, no presenta Las alas son dobles y se mantienen dobladas sobre el
flagelo visible, es intracelular y se reproduce por división dorso y pocas especies no tienen alas; en general los
binaria. triatominos son más caminadores que voladores.
Los nidos de amastigotes contienen gran número de
parásitos en los tejidos lesionados, lo que facilita su ◆ Reproducción: mediante huevos y hacen una
identificación. metamorfosis incompleta pasando por cinco estados
antes de llegar a adulto..

PARA RECORDAR: El ciclo evolutivo completo varía con las


especies y por lo general dura entre 84 y
134 días.
►Si se cuenta a partir del huevo, la vida oscila entre
300 a 350 días.
►Una hembra puede poner entre 1.200 y 1.400
huevos.
►La sangre es indispensable para los insectos hemató-
fagos para producir la ecdisona u hormona de la meta
morfosis.
►La picadura es indolora y se efectúa principalmente
VECTOR: en la noche.
Esta enfermedad es transmitida por unas 120 especies de
Triatominos .
Son hemípteros, estrictamente hematófagos de la familia
Reduiviridae, subfamilia triatominae y géneros rhodnius,
triatoma y panstrongylus, conocidos en nuestro país como
chirimacha.

97
Manual de INFECTOLOGÍA PLUS MEDIC A

Ciclo biológico 5 Los amastigotes forman nidos en el TCSC y se


transforman en tripomastigotes que pasan a la
1 El insecto vector al tomar sangre, ingiere los
sangre.
Trypomastigotes.
2 Los tripomastigotes se transforman en el estómago
del insecto en epimastigotes, que pasan al intestino,
donde se reproducen por división binaria y en la porción final
del mismo se transforman en trypomastigotes que reciben el
nombre de trypomastigotes metacíclicos.
3 Los tripomastigotes metacíclicos encuentran en las
heces del insecto, siendo las formas infectantes para el
hombre y los reservorios.

6 Los trypomastigotes, pasan a la sangre, a


través de la cual alcanzan a los tejidos de diferentes
órganos, siendo los más afectados corazón y plexos
nerviosos intramurales del tubo digestivo

El triatoma infestans es muy glotón


El triatoma infestans al
momento de alimentarse
(picar) defeca, pues debe
desocupar el intestino
para poder acumular la
mayor cantidad de sangre
posible .

La puerta de entrada se genera al momento de


rascarse.
4 El hombre o animal picado, se rasca el sitio de la En los tejidos ocurre nuevamente la transformación
picadura y provoca excoriaciones en la piel, estas del tripomastigote en amastigote en el interior
estas lesiones permiten el ingreso del de las células, posteriormente, este intercambio de formas
trypomastigote al tejido celular subcutá- neo, de amastigote intracelular y de trypomastigote
donde se introduce en las células y transforma en sanguíneo, caracteriza la evolución del parásito en el
amastigote en el interior de las mismas y allí se interior del organismo del hombre y reservorios
reproducen.

98
Manual de INFECTOLOGÍA PLUS MEDIC A

Patogenia y cuadro clínico


FASE AGUDA
Chagoma
La lesión inflamatoria localizada en la puerta de entrada,
es visible como un chancro de inoculación y se conoce
con el nombre de chagoma Sistema reticuloendotelial

Cuadro clínico
►Período de incubación: 4 y 12 días.
►Formas clínicas:
√ Proceso infeccioso general: fiebre, hepato-
Chagoma de inoculación esplenomegalia, mialgias y disnea.
√ Síntomas inespecíficos leves (más frecuente)
La inflamación se extiende a los ganglios √ Miocarditis (10%): trastornos de la conducción
regionales
La fase aguda es muy rara en los adultos
Se bloquean los canales linfáticos y se produce edema y en el 70% de los casos, o más, afecta
local. a los niños menores de 10 años.
Signo de Romaña
Cuando el chagoma de inoculación se localiza en los ENAM 06-B (11) : Adolescente de 12 años de edad, edad,
párpados , produciendo edema inflamatorio procedente de un valle costero de la regió región
bipalpebral unlateral se conoce como el signo de suroccidental del Perú
Perú. Presenta al examen: edema
Romaña. bipalpebral unilateral,
unilateral, adenopatí
adenopatía preauricular,
hiperemia de la conjuntiva y dacriocistitis. ¿Cuá
Cuál es el
diagnó
diagnóstico más probable?:
A. Loxocelismo
B. Enfermedad de Chagas crócrónica
C. Enfermedad de Chagas aguda
D. Latrodectismo
E. Escorpionismo
Signo de Romaña Rpta. C

Posteriormente, se encuentran parásitos intracelulares en: Definición de caso Agudo sospechoso


►Otros ganglios linfáticos y ◆Persona con fiebre prolongada (superior a 7 días)
►Órganos como bazo, médula ósea, corazón, tubo digestivo, y una o más de las siguientes manifestaciones clínicas:
suprarrenales, cerebro, ocasionalmente ovarios, testículos y √ Edema de cara o de miembros
tiroides. √ Exantema
√ Adenomegalia, hepatomegalia, esplenome
Sistema reticuloendotelial galia
Los histiocitos fijos, células musculares, células adiposas, √ Cardiopatía aguda, manifestaciones hemorrágicas,
células gliales y en general las células del sistema ictericia, signo de Romaña o Chagoma de inoculación
reticuloendotelial sufren destrucción debido al
√ Sea residente o visitante de área con presencia de
crecimiento y multiplicación de los parásitos.
triatomineos

99
Manual de INFECTOLOGÍA PLUS MEDIC A

O también: Este período va desde :


√ Haya recibido recientemente por una transfusión de el final de la fase aguda hasta la aparición
sangre, componentes sanguíneos de los primeros síntomas de la fase crónica
√ Haber recibido trasplante (s) (latente o indeterminado).
con una duración media de 10 años.
√ Haya ingerido alimento sospechoso de contamina-
ción por el T. cruzi
En esta fase el paciente es asintomático a pesar de
√ Sea recién nacido de madre infectada (transmisión las alteraciones que se presentan en los :
congénita) plexos parasimpáticos del corazón y del tubo
Criterio parasitológico digestivo.
◆ T. cruzi circulante en la sangre periférica
identificado por examen parasitológico directo, Hay poca parasitemia y parásitos en tejidos.
con o sin identificación de cualquier señal o
síntoma CRÓNICA propiamente dicha
Se caracteriza por una reducida parasitemia y lesiones
Criterio serológico típicas en corazón y tubo digestivo.
◆Serología positiva con anticuerpos IgM
Definición de caso confirmado
anti-T. cruzi con :
◆Persona que presente :
√ Evidencias clínicas y epidemiológicas o
√ 2 resultados serológicos concordantes y po-
√ Serología positiva con anticuerpos IgG anti T.
sitivos
cruzi por IFI con alteración en la concentración
(basados en dos técnicas serológicas de principios distintos)
de al menos tres títulos en un intervalo mínimo
para la detección de anticuerpos IgG antiT.cruzi.
de 21 días en muestras vinculadas, en presencia
de evidencias clínicas y epidemiológicas.
Durante ésta fase la patología más importante es
√ Seroconversión en muestras vinculadas con la cardiopatía chagásica.
intervalo mínimo de 21 días, es decir serología
negativa en la primera muestra y positiva en la
segunda, por cualquier método.

Criterio clínico epidemiológico


◆Exámenes parasitológicos y serológicos inicial -
mente negativos en presencia de cuadro febril
con manifestaciones clínicas compatibles con
chagas agudo.

Mortalidad
El índice de mortalidad en la fase aguda es
aproximadamente cerca del 10%.

Las muertes ocurren principalmente por miocarditis,


meningoencefalitis y otras complicaciones como
bronbronconeumonía . Fase crónica de la enfermedad de Chagas

FASE CRÓNICA Cardiopatía chagásica


Indeterminada Inicialmente hay compromiso de las cavidades
Después de la fase aguda ocurre una respuesta inmune derechas con miocarditis y desintegración de las
que provoca disminución de la parasitemia y mantiene la fibras miocárdicas.
infección en algunos focos selectivos.

100
Manual de INFECTOLOGÍA PLUS MEDIC A

Los nidos de parásitos generan un Luego decrece la parasitemia y se mantiene


infiltrado mononuclear que favorece la la respuesta inmune.
liberación de antígenos y sustancias
tóxicas que causan edema intersticial. La población predominante de células inflamatorias en
el tejido cardíaco de pacientes crónicos, es la de los linfocitos
También se producen autoanticuerpos contra el T CD8+ (relación 1:3 con respecto a los linfocitos T CD4).
endocardio, los vasos sanguíneos y el intersticio del músculo
estriado. Las citoquinas dominantes in situ, tanto en la fase
aguda, como durante la fase crónica de la miocarditis y
√ La inflamación alcanza el subendocardio, que contribuyen al intenso reclutamiento de células T
el tejido adiposo del epicardio y los ganglios activadas, CD8+ que son:
nerviosos. √ El factor de necrosis tumoral alfa, el inter- ferón
√ A nivel del tejido de conducción también se gamma y las quimosinas inducidas por el interferón
pueden encontrar nidos de parásitos, edema e gamma.
infiltrado. Las quimiokinas afectan muchos otros fenómenos
√ Existe además fibrosis e infiltrado, con inmunológicos
predominio de células mononucleadas. Juegan un papel importante en la regulación inmune,
√ El sistema de conducción del corazón, mediando la activación leucocitaria, la
principalmente la rama derecha del haz de His, coestimulación y la diferenciación durante las respuestas
también se encuentra alterado, con fibrosis e inmunes innata o adaptativa.
infiltrado linfocitario, del mismo.
Diagnóstico
Las lesiones y sus manifestaciones guardan
estrecha relación con la duración de la
La mejor prueba diagnóstica para las 3 fases es la
infección; generalmente adoptan la forma
de una insuficiencia cardíaca y de arritmias. PCR (Reacción en cadena de polimerasa).

Megaesófago El diagnóstico de la fase crónica es


eminentemente serológico.
En el megaesófago se distinguen hasta 4 etapas en la
evolución del mismo, de acuerdo con la alteración
►La fase congénita se diagnostica con la detección
funcional del esófago, etapas detectables por el
de Ig M.
estudio de la función, mediante radiografías con
sustancia opaca. ►En la fase aguda la serología recién es (+) a
partir de la 3º semana.
Megacolon
En el megacolon se reconoce dos etapas:
◆Primera etapa : aumento de volumen de la víscera
◆ Segunda etapa: con dolicomegacolon, donde al
aumento de volumen de la víscera, se añade la elongación de
la misma.

INMUNOLOGÍA
Parasitemia y respuesta inmune tipo I
Al iniciarse la infección puede existir :
◆ Parasitemia notoria y
◆ Respuesta inmune de predominio tipo I
Duración: varias semanas

101
Manual de INFECTOLOGÍA PLUS MEDIC A

Xenodiagnóstico Electrocardiograma
√ Bloqueo de rama derecha del haz de Hiss

Este método consiste en utilizar al vector libre de


infección
√ Se hace que pique a la persona sospechosa de
infección Nemotecnia:
√ Se espera en caso de enfermedad, que el parásito se
reproduzca en el intestino del vector
√ Al examinar las heces del mismo ,luego de un
mes ,se puede demostrar la presencia del
parásito.

Xenodiagnóstico Patrón “M” en V1-2 : rSR’


Utiliza : Patrón “W” en V5-6: S ancha o mellada
√ Una caja donde se colocan 10 ninfas del
vector de 3ro a 4to. estadío de desarrollo.
√ La caja se sujeta con una ligadura al brazo
de la persona y se espera de 20 a 30 minutos,
lo cual es suficiente para que el vector se alimente.
√ Es a partir del mes que se comienza a
examinar las heces de las ninfas colocadas

PCR
La reacción en cadena de la polimerasa (PCR) que
permite la ampliación del ADN o ARN (ácido
desoxirribonucleico, o ribonucleico) involucrados en la
reproducción del parásito.

Es útil en los casos de infección crónica en Imágenes


que hay escasa parasitemia.
Cardiopatía chagásica: aumento de la silueta
Al amplificar el poco material de ácido nucleico del cardiaca por dilatación del corazón
parásito que haya en sangre, puede hibridizarse con
sebadores e indicarnos la presencia del parásito.

También nos indica la raza, y cepa de que se


trata.

Este método es útil en la confirmación de los casos


crónicos o en portadores.

102
Manual de INFECTOLOGÍA PLUS MEDIC A

Megaesófago : dilatación del esófago semejante a Derivado nitrofuran


acalasia √ Dosis: 15 mg/Kg /día divididos c/8 h en
2-3 meses.
√ Efectos secundarios:
Dolor abdominal, náuseas, vómitos, anorexia, pérdida de
peso. Inquietud, insomnio, mioclonias, , parestesias y
convulsiones.

Contraindicaciones
El benznidazol y el nifurtimox no deben
administrarse a las embarazadas ni a las personas con
insuficiencia renal o hepática.
Tratamiento El nifurtimox también está contraindicado en personas con
◆Benznidazol antecedentes de enfermedades neurológicas o
◆ Nifurtimox trastornos psiquiátricos.

Son eficaces casi al 100% para curar la Curación


enfermedad si se administran al comienzo de La negatividad serológica se considera como
la infección en la etapa aguda, incluso en los marcador de curación.
casos de transmisión congénita.
El tiempo necesario para la negativización varia en
Sin embargo, su eficacia disminuye a medida que función de la fase de la enfermedad:
transcurre más tiempo desde el inicio de la infección. √ Infección congénita: 1 año
√ Infección aguda: 3-5 años
también está indicado en caso de reactivación √ Infección crónica: 5-20 años
de la infección (por ejemplo, por
inmunodepresión) y en los pacientes al principio Estrategias
de la fase crónica. Prevención primaria
-►Detección de la infección pot T. cruzi en las mujeres en
Benznidazol edad fértil, previo al embarazo
Derivado nitroimidazol -►Tratamiento de los casos positivos.
√ Dosis: 5 mg/Kg/día/ 2 veces diarias Inconvenientes::
X 30-60 días -►Baja cobertura
√ Efectos secundarios: -►Dificultad para asegurar una curación
Gastrointestinales, neuropatía periférica, rush,
granulocitopenia. Prevención secundaria
-►Detección de la infección por T. cruzi en las madres en
el seguimiento del embarazo / ingreso para el parto.
-►Seguimiento del recién nacido de madre positiva y
detección precoz de la posible infección congénita.
-►Tratamiento con Benznidazol lo antes posible , en el
primer año de vida.

Ventajas
-►Cobertura casi total en las mujeres embarazadas.

Nifurtimox
103
Manual de INFECTOLOGÍA PLUS MEDIC A

-►Seguimiento del recién nacido.


-►Permite detectar otros “colectivos de riesgo”

EsSalud 2011 (95): El agente etiológico de la


Enfermedad de Chagas se denomina:
a)Triatoma infestans.
b) Pastrongylusherreri.
c) Yerisinapestis.
d) Tripanosoma cruzzi.
e) Tripanosoma gambiense .
Rpta. D

Bibliografía
1.Almeida, E. A. (2010). Chagas’ disease and HIV co-
infection in patients without effective antiretroviral therapy:
prevalence, clinical presentation and natural history.
Transactions of the Royal Society of Tropical Medicine
and Hygiene, 104(7), 447-452.

2.Ministerio de Salud de la Nación. (2012). Guías Para La


Atención Al Paciente Infectado Con Trypanosoma cruzi
(Enfermedad De Chagas) Argentina. Secretaría de
Programas Sanitarios. Argentina.

3.Savioli, L., & Daumerie, D. (2013). Sustaining the Drive


to Overcome the Global Impact of Neglected Tropical
Diseases: Second WHO Report on Neglected Tropical
Diseases (Vol. 2). D. W. T. Crompton (Ed.). World
Health Organization. Recuperado
http://www.who.int/neglected_diseases/9789241564540/e
n/

4.Sociedad Argentina de Cardiología. Consejo de


Enfermedad de Chagas “Dr. Salvador Mazza” (2011).
Consenso de Enfermedad de Chagas-Mazza. Revista
Argentina de Cardiología / vol 79 nº 6 / noviembre-
diciembre 2011.

104
Manual de INFECTOLOGÍA PLUS MEDIC A

BARTONELOSIS PLUS MEDIC A


Esta enfermedad se caracteriza por afectar
DEFINICIÓN principalmente a niños menores de 14 años en más del
Bartonelosis es una enfermedad infecciosa producida 60% de los casos. Afecta a nativos principalmente, pero
por la bacteria del género Bartonella. también a foráneos en quienes la enfermedad puede ser
más grave.
Las especies del género Bartonella
causan enfermedades tales como la : Tabla Nº 2 Bartonellas patógenas para el hombre,
►Enfermedad de Carrión, Fiebre de las Trincheras y sus vectores y enfermedades que producen
Enfermedad por Arañazo de Gato
►Así como otras enfermedades tales como :
Angiomatosis Bacilar, Peliosis Hepática, Bacteremia
Crónica, Endocarditis, Linfadenopatía Crónica y
Enfermedades Neurológicas.

MICROBIOLOGÍA
El género Bartonella y la clasificación
taxonómica de B. bacilliformis
Actualmente el género Bartonella contiene 14
especies (tabla Nº 1)
►De los cuales 7 han sido reportado en los últimos 5 años
►Siete son patógenas para el humano y producen
diversas enfermedades y sindromes (tabla Nº 2):
B. bacilliformis, B. quintana, B. vinsonii, B. elizabethae,
B. henselae, B. clarridgeiae y B. grahamii; las otras siete
especies han sido aisladas sólo en mamíferos Características Microbiológicas de la
pequeños y aparentemente no producen enfermedad
Bartonella Bacilliformis
Bartonella bacilliformis fue
en el hombre, aunque no se descarta que en el futuro se descrita en 1909 por Alberto
reporten éstas u otras nuevas especies como patógenas Barton.
para el hombre.

Tabla Nº 1 Especies y subespecies de


Bartonella

Es un bacilo gram negativo, pleomórfico, intracelular.

√ Tamaño: de 0.2-0.5 por 1-2um.


√ Aaeróbico: no fermentativo
√ Unipolar con 2 a 16 flagelos que le confiere alta
movilidad.
B. elizabethae produce endocarditis infecciosa.
►El reservorio y su vector son desconocidos

105
Manual de INFECTOLOGÍA PLUS MEDIC A

√ Se tiñe de color rojo con Giemsa Durante el Fenómeno el Niño de los años
√ Puede ser cultivada sobre medios sólidos a partir 1997-1998, se produjeron cambios climatológicos que
de muestras de sangre, biopsias de lesiones erupti- incrementaron la densidad del vector en varios valles
vas o nódulos subcutáneos. interandinos
√ Crece lentamente en medios enriquecidos con ◆ Se presentaron rebrotes después de muchos años en
sangre y aminoácidos esenciales, como el de los valles de Cañete-Yauyos en Lima, Pataz en Trujillo,
Seneckii o agar Columbia,la temperatura óptima de Quillabamba en Cusco y en zonas donde no había
crecimiento es 28°C.
reportes anteriores de enfermedad como el Valle del
√ Es una bacteria no reactiva en test bioquímicos Urubamba en el Cusco.
usados para identificación de bacterias.No utiliza
los carbohidratos. Es una bacteria intracelular de
Lambayeque
las células endoteliales y eritrocitos.
El año 2002 se presentó otro brote, en una zona no
endémica, en la localidad de Cañaris, departamento de
EPIDEMIOLOGÍA
Lambayeque.
La Bartonella bacilliformis, es transmitida por la picadura de
Madre de Dios
mosquitos del género Lutzomyia.
Durante el 2004 se han presentado casos no autóctonos
Lutzomyia es más pequeña
en Madre de Dios.
que otros mosquitos,
Huarochiri
midiendo de 2 a 4 mm.
Entre marzo y abril del 2005, se presentó un brote
Como todos los dípteros,
después de muchos años en Huarochirí – Lima Este; y
tienen un solo par de alas,
recientemente en setiembre de 2006 se presentó un caso
que son ovaladas, en forma de
fatal en una persona que visitó la cuenca de Santa Eulalia.
V y densamente cu biertas por
La Enfermedad de Carrión o Bartonelosis humana
pelos. -Essalud 08-
actualmente es considerada como una enfermedad
re-emergente en el Perú.
La bartonelosis ha sido
reportada en Perú, Ecuador y
Colombia. CARACTERÍSTICAS
En el Perú, la enfermedad es ►Rango de hospederos: Humanos.
endémica a una altitud entre ►Modo de transmisión: Por la picadura de mosquitos
los 500 y 3200 metros sobre del género Lutzomyia, o por transfusión sanguínea.
el nivel del mar. ►Período de incubación: Usualmente de 16 a 22 días,
Los casos en el Perú se ha reportado de valles ocasionalmente de 3 a 4 m.
interandinos y algunas regiones de la selva alta. ►Transmisión: No se ha documentado transmisión de
persona a persona, la sangre del paciente permanece
Los vectores reconocidos para esta enfermedad son los infecciosa para el mosquito por varios meses .
insectos: ►Reservorio: Humanos
Lutzomyia verrucarum, L. maronensis, L. peruensis, L. ►Zoonosis: No
serrano, L. pescei y L. bicornutus. ►Vectores: Mosquitos del género Lutzomyia.
El hombre es el único reservorio conocido para
la enfermedad. ASPECTOS CLÍNICOS
a) Fase Anémica, Febril, Fiebre de la
Oroya, Enfermedad de Carrión
Existen zonas endémicas en Ancash, Cusco, Cajamarca,
◆Período de incubación : promedio de 61 días
Amazonas, Lima, Piura, La Libertad, Huancavelica,
(rango de 10-210).
Huánuco, Ayacucho y Junín.

106
Manual de INFECTOLOGÍA PLUS MEDIC A

◆ Síntomas prodrómicos: son inespecíficos, Puede presentarse, además,


►Generalmente de inicio gradual anasarca, edema pulmonar no cardiogénico, sangrado
►Siendo los más frecuentes sensación febril, fiebre pericárdico, miocarditis, delirio y coma, considerados
irregular, malestar general, escalofríos leves, como factores de riesgo para morir.
mialgias, artralgias, cefalea, náuseas, vómitos.
b) Periodo Intercalar Asintomática

Sigue a la fase anémica


Se caracteriza por que desaparece la fiebre,
se detiene la hemólisis, hay mejoría o
desaparecen los síntomas y signos, y no
encuentran bacterias circulantes.
Es habitualmente asintomática y puede durar de 1 a
3 semanas, excepcionalmente puede durar varios
meses.

En esta fase inicial debe realizarse el diagnóstico


diferencial con fiebre tifoidea, tifus murino,
leptospirosis, malaria, brucelosis, hepatitis viral,
meningitis, anemia hemolítica, anemia aplásica

La evolución es rápida y en pocos días


puede llegar a presentar anemia severa.

Complicaciones
Durante la segunda semana de la enfermedad se
presentan las complicaciones infecciosas y no c) Fase Eruptiva, Verruga Peruana
infecciosas, con compromiso pulmonar, hepático, Se observa generalmente en niños que viven en
renal, cardiovascular, neurológico y en los casos se- áreas endémicas.
veros falla orgánica multisistémica y muerte.
Signos
Fase eruptiva
En algunas zonas es la principal forma clínica de
Palidez, ictericia, linfoadenomegalia, hepatome galia,
presentación, con baja letalidad, ejemplo Callejón de
esplenomegalia.
Huaylas y Conchucos en Ancash y en la provincia de
Manabi en el Ecuador
Infecciones oportunistas
Durante la fase aguda cerca del 30% de los casos
hospitalizados presentan infecciones
oportunistas intercurrentes, siendo las más
frecuentes infecciones por Salmonellas,
toxoplasmosis, histoplasmosis diseminada,
tuberculosis miliar, sepsis por Staphylococcus aureus,
Enterobacter, Shiguella dysenteriae, pneumocistosis
y Plasmodium vivax.

107
Manual de INFECTOLOGÍA PLUS MEDIC A

En otras áreas predomina la forma


anémica con alta letalidad y muy pocos casos de la
forma eruptiva, ejemplo las provincias de La
Convención y Urubamba en Cusco, Tingo María en
Huánuco, San Ignacio en Cajamarca y la provincia
de Zamora-Chinchipe en el Ecuador.

Por lo anterior se plantea la existencia de


varias cepas de B. bacilliformis, unas serían
poco virulentas y su principal forma clínica de Fórma nodular o subcutánea
presentación seria la forma verrucosa, y las otras
serían más virulentas y su forma clínica Es frecuente encontrar en un mismo pacientes una
combinación de los diferentes tipos de lesiones,
predominante seria la anémica.
pueden infectarse secundariamente dificultando el
La fase eruptiva puede presentarse después de la fase
diagnóstico diferencial con granuloma piogeno o piodermitis,
aguda o directamente; los síntomas son leves . involucionan en aproximadamente treinta días con
Los más frecuentes antes de la erupción son tratamiento y en 2-6 meses sin ningún tratamiento;
malestar general leve, cefalea, febrículas, artralgias y independientemente del tipo de lesión, no dejan ninguna
mialgias o más frecuentemente en forma cicatriz.
asintomática.
Brotes post fenómeno del niño en 1992 y 1998
Las lesiones pueden ser de tres tipos: Se ha observado que los casos eruptivos luego del
Miliar “fenómeno del niño”en 1992 y 1998 son más
Los verrucomas son de severos habiéndose presentado en ambos brotes
< 3 mm de diámetro. una forma seudovariceliforme caracterizada por
la presencia de múltiples lesiones en diversas
etapas de evolución (polimorfismo regional),
muy sangrantes, con predominio de presentación
en las extremidades, pueden llevar fácilmente a la
muerte por el sangrado de las lesiones por lo que ha
Mular sido necesario la transfusión de sangre en estos casos.
Los verrucomas son de 5 mm o más, estas son
frecuentemente sésiles, erosionadas y muy
sangrantes.

Forma seudovariceliforme

 Nodular o Subcutánea Clásicamente se decía que la presentación de la forma


Nódulos localizados principalmente sobre
anémica o eruptiva aseguraba la protección contra la
superficies extensoras de brazos y piernas,
generalmente múltiples, muy raramente únicas.
enfermedad de por vida. Sin embargo, el haber
sufrido la enfermedad aguda o eruptiva no
descarta la posibilidad de ataques posteriores.

108
Manual de INFECTOLOGÍA PLUS MEDIC A

Complicaciones de la Fase anémica


Son frecuentes y pueden clasificar en:
◆Infecciosas y no infecciosas

Ambos tipos de complicaciones pueden estar presente


en un mismo paciente.
La letalidad en este grupo de pacientes es
muy alta.
Complicaciones no infecciosas

-►Respiratorias : bronquitis y neumonía,


insuficiencia respiratoria aguda, síndrome de distrés
respiratorio del adulto, asociado a hipoxemia severa,
neumonía intersticial, ,
-►Cardiovasculares disnea de esfuerzo
palpitaciones, tos, taquicardia, insuficiencia cardiaca
congestiva, edema agudo de pulmón , pericarditis con
derrame, taponamiento cardiaco, shock cardiovascular.
-►Hematológicas anemia severa, púrpura,
plaquetopenia, anemia hemolítica autoinmune.
-►Gastrointestinales: dolor abdominal, estre-
ñimiento, hepatomegalia, esplenomegalia, pruebas
hepáticas alteradas, coluria, vómitos, ictericia, ascitis.
-►Neurológicas: cefalea, alteración del estado de
conciencia, fotofobia, vértigo ambliopía, amaurosis,
somnolencia, convulsiones, signos meníngeos, coma,
hipertensión endocraneana y renales.

Complicaciones infecciosas
◆Bacterianas
La salmonelosis es una de las principales
complicaciones de la fase anémica de la Enfermedad de
Carrion.
Otras : Shiguella dysenteriae, Enterobacter sp. y
Staphylococcus aureus, bacteremia por Klebsiella.

109
Manual de INFECTOLOGÍA PLUS MEDIC A

OF: Paciente varon de 16 años, procedente de Frotis de sangre periférica


caraz (ANCASH),
(ANCASH), que presenta anemia aguda ►Durante la fase aguda, este es el método más
severa con plaquetopenia,
plaquetopenia, fiebre cuadro clinico práctico, aunque no el más sensible.
de insuficiencia cardiaca congestiva con bloqueo La tinción puede realizarse con Wright o Giemsa.
de rama derecha segú según ECG, y derrame
El inconveniente que es poco sensible y depende
pericardico;
pericardico ; imagen de consolidació
consolidación en principalmente de dos factores:
radiografí
radiografía de torax. El médico que lo atiende √ Una buena toma de muestra
sospecha de bartonellosis ¿CuáCuál de los siguientes √ De personal debidamente capacitado para la lectura
metodos diagnó
diagnósticos no serí
sería aportativo? del frotis.
a. Frotis sangre periferica
b. Hemocultivos La sensibilidad, especificidad y valor predictivo
positivo del frotis en fase aguda de 36%, 96% y
c. Cultivo de hisopado farí
faríngeo
44% respectivamente.
d. Aglutinaciones
e. PCR
►Un resultado de frotis positivo confirma el
diagnóstico
La infección por Mycobacterium tuber- ►Un resultado negativo no lo descarta
culosis es otra complicación frecuente el diagnóstico en la fase aguda es clínico y se
produciendo generalmente tuberculosis debe iniciar el tratamiento inmediatamente,
miliar. aún si el resultado del frotis fuera negativo.

Otra enferemedad asociada es la Leptospirosis con ►En la fase eruptiva la sensibilidad del frotis es
cuadros purpúricos y presentación clínica inusual, todavía menor, siendo inferior al 10%.
infecciones urinarias, sepsis, fiebre tifoidea,
bronconeumonía.
Por lo tanto en áreas endémicas de transmisión el
diagnóstico de la fase eruptiva también es
-►Vírales: Se han reportado complicaciones por herpes
clínico y debe considerarse como caso a todo paciente que
diseminado y hepatitis B post transfusión.
cumpla con la definición de caso.
-►Parasitarias: La toxoplasmosis es una de las
complicaciones parasitarias descritas en la fase aguda de El tratamiento también debe iniciarse en el mo-
la enfermedad de Carrión. mento de hacer el diagnóstico, previa toma de muestra
Se han reportado casos de neumonía por para frotis y hemocultivo.
Pneumocystis carinii y de Malaria por P.
vivax . Cultivo
►Es más sensible que el frotis, pero se necesita
d) Micosis profundas: histoplasmosis diseminada mayor infraestructura y tecnología.

DIAGNÓSTICO POR LABORATORIO Requiere de:


Las pruebas para confirmar las infecciones por ◎ Preparación de medios de cultivo muy enriquecidos,
Bartonellas dependerán de : que fácilmente se contaminan (tasa de contaminación
√ La fase en que se encuentra la enfermedad de 7-20%).
√ Tipo de muestra colectada ◎Ambientes especiales como cámara de flujo laminar
√ Disponibilidad de equipos e infraestructura, y la para realizar la siembra, una temperatura de incubación
existencia de personal entrenado. de 28 ºC.

110
Manual de INFECTOLOGÍA PLUS MEDIC A

◎Periodos de incubación prolongados hasta por seis


semanas y subcultivos sistemáticos de los cultivos
primarios (-).

La única manera de poder identificar las


diferentes especies de Bartonellas es
aislando la cepa a partir de una
muestra biológica y realizando su posterior
secuenciamiento genético de los pro ductos
amplificados por PCR.
Las muestras biológicas para aislar B. bacilliformis
pueden ser sangre venosa, aspirado de medula ósea,
biopsia de lesiones verrucosas o de órganos, liquido
pericárdico, liquido cefaloraquídeo.

El aislamiento de Bartonellas no requiere


de medios especiales Diagnóstico histopatológico
Sin embargo el crecimiento es lento y se necesita un Con tinción de hematoxilina-eosina o con tinción de plata de
tiempo de 2 a 6 semanas para el aislamiento primario. Warthin-Starry
Las cepas aisladas deben ser identificadas por pruebas En la práctica sólo es posible realizarlo en :
moleculares. ►Biopsias de lesiones de la fase eruptiva o
en muestras de necropsias realizadas
Los cultivos permiten aislar e identificar ► Pacientes que fallecieron en fase anémica
las Bartonellas sp. por las características
morfológicas de las colonias hasta el nivel es útil en fase eruptiva en cualquiera de sus
de género, pero no de especie. formas, miliar, mular o nodular y en algunos
►La identificación de especie y las cepas dentro de pacientes en fase anémica complica-
cada especie solo es posible realizarlo a través de da.
PCR
El posterior secuenciamiento genético de El diagnóstico histopatológico con la tinción de
los fragmentos de DNA amplificados o a Warthin-Starry sólo confirma la presencia de bacterias
e identificarlas hasta nivel de género pero no de
través del perfil de ácidos grasos de la pared celular
especie.
por cromatografia de gas liquida.
Diagnóstico serológico por ELISA
√ B. bacilliformis puede ser cultivada en Agar ►Necesita la preparación de un antígeno altamente
Columbia enriquecido con 5% de sangre de carnero
purificado que requiere de tecnología complicada y es de
o conejo e incubado a 28°C.
alto costo.
√ Debido a que su crecimiento es muy lento, los
Los primeros estudios sobre la inmunidad humoral hacia B.
cultivos deben ser mantenido hasta por 8
bacilliformis fueron realizados a través de la prueba de
semanas para ser considerado como negativo.
fijación del complemento y por el test de
√ En nuestro país aun no se realiza cultivos hemaglutinación.
celulares para aislamiento de B. bacilliformis a partir
de muestras clínicas. Ventajas
Teóricamente es más sensible que los cultivos en La detección de anticuerpos contra
agar. Bartonellas tiene las ventajas de que evita
los problemas asociados con otros métodos
como:
111
Manual de INFECTOLOGÍA PLUS MEDIC A

√ Periodos largos de incubación para los cultivos


√ Obtención de muestras difíciles de obtener (biopsia
de tejidos u órganos).
√ Implementación con equipos altamente espe-
cializados y ambientes especiales para realizar
cultivos y pruebas por biología molecular.
※ La desventaja es que la respuesta de
anticuerpos es inespecífica en los humanos
※ Hay reacción cruzada entre las diferentes
especies de Bartonella y entre B. bacilliformis y
Chlamydia psittaci, entre B. quintana y Chlamydia
neumoniae, Chlamydia trachomatis, Chlamydia
psittaci y Coxiella burnetii . ►El seguimiento de los pacientes debe realizarse a
El método de diagnóstico por laboratorio través de la evaluación clínica y con frotices de
a utilizarse depende también de la fase en controles.
que se encuentra la enfermedad al momento de ►En la fase anémica ambos deben realizarse a los
realizar el diagnóstico.
3, 7, 14 y 21avo día de iniciado el tratamiento.
►En la fase eruptiva al terminar el tratamiento, a
OF: Dentro de las manifestaciones clí clínicas que
presenta el paciente de la pregunta anterior,
los 30 y 60 días después.
¿Cuá
Cuál no es complicació
complicación de la fase ané
anémica de la
bartonellosis? :
Falta de respuesta al tratamiento
En pacientes con diagnóstico de bartonelosis en fase aguda
a. Insuficiencia cardiaca congestiva
Θ Falta de respuesta clínica al tratamiento
b. Bloqueo de rama derecha
c. Derrame pericardiaco
(ya que no se ha demostrado resistencia al antibiótico por
d. Neumoní
Neumonía laboratorio)
e. Trombocitopenia Θ si después del quinto día de iniciado el
Rpta. B tratamiento se observa persistencia o
incremento del índice de parasitemia en frotis de
Comentario sangre periférica y no hay mejoría clínica.
La insuficiencia cardiaca es la complicacoió
complicacoión
cardiovascular más frecuente, no se ha reportado Para la fase eruptva, se considera falta de
BCRDHH asociado a bartinelosis.
respuesta al tratamiento si :
ΘDespués de 30 días de terminado el tratamiento aún
persiste o se incrementan las lesiones.
TRATAMIENTO
Una de las principales complicaciones que presentan los
Esquemas de tratamiento
El tratamiento oportuno de los pacientes disminuye las pacientes en la fase anémica son las superinfecciones
complicaciones infecciosas y no infecciosas de la por bacterias, hongos, parásitos o virus.
Enfermedad de Carrión.
Los esquemas de tratamiento propuestos se han ¿Cuándo se debe sospechar de una sobreinfección?
basado en la evaluación de la respuesta a un en aquellos pacientes que después de 72 horas
antibiótico en una serie de casos sin tener un grupo de iniciado el tratamiento, se observa:
testigo. ΘDisminución significativa o ausencia de parasitemia pero
sin mejoría o empeoramiento del cuadro clínico.
Puesta clínica al tratamiento usualmente se observa a
las 24 a 48 horas después de iniciar el tratamiento, Debe solicitarse hemocultivos seriados, biopsias de
la mejoría del estado general del paciente es órganos o tejidos y toma de muestras de suero para títulos de
espectacular. anticuerpos de los agentes etiológicos que se sospeche.

112
Manual de INFECTOLOGÍA PLUS MEDIC A

ENAM 04-A (79): De acuerdo Esquemas


al Manual del MINSA,
La letalidad en estos pacientes es muy alta si no se Cuadro clínico
el medicamento de elecciónsugeridos
para el tratamiento de
administra el tratamiento específico. la bartonelosis aguda en adolescentes mayores de
Fasey Anémica
14 años kg)Ciprofloxacina:
adultos (» 4S ◆ es:
OF: Además de la Salmonelosis ¿Qué otras no Complicada
A. Ciprofloxacina 500mg c/12 hrs VO por
infecciones secundarias acompañ
acompañan fre-
fre- B. Penicilina procaínica10 días.
cuentemente a la Fiebre de la Oroya?
Oroya? C. Estreptomicina
A.- Sarna costrosa – Encefalitis viral. D. Tetraciclinas ◆Cotrimoxazol:
B.- Hepatitis viral - Influenza. E. Cloranfenicol 800 sulfametoxazol /
C.- Paludismo - Tuberculosis. Rpta. A 160 trimetroprin c/12
D.- Micetorna - Leishmaniasis.
E.- Criptococosis – dengue hrs VO por 14 días
ENAM 05-B (9): 2. Los nichos verrucógenos de la
Rpta. C Bartonelosis o enfermedad de Carrión se ubican entre:
◆Ampicilina:
A. 100 y 500 msnm 500mg c/12 hrs VO por
ENAM 03-A (44): Paciente de 40 años de edad, B. 1.000 y 3.000 msnm
que viene directamente desde Abancay al 14 días.
C. 3.000 y 5.000 msnm
servicio de Emergencia, en donde se evidencia D. 500 y 900 msnm
paciente soporoso, con fiebre de 40°C. Severa ◆ Amoxicilina:
E. 3.500 y 4.000 msnm
palidez de piel y mucosas, subictérico. 500mg c/8 hrs VO por
Rpta. B
Hepatoespleno- megalia. Análisis: Hemoglobina: 14 días.
4 g%; leucocitos: 18.000, reticulocitos: 15%; test
de Coombs: negativo; test de Ham: negativo; Fase Anémica ◆Ciprofloxacina:
gota gruesa: negativo. La posibilidad Complicada 200mg c/12 hrs EV por
diagnóstica es: 10 días.
A. Infección por Clostridium perfringens
B. Anemia hemolítica autoinmune ◆ Ceftriaxona:
C. Hemoglobinuria paroxística nocturna 1 gr c/12 hrs EV por 10
D. Malaria falciparum días.
E. Bartonelosis
Rpta.E Fase Eruptiva ◆Eritromicina:
25-50mg/kg/d,
dividido en 4 dosis VO
por 14 días.

◆Ciprofloxacina:
500mg c/12 hrs por
7-10 días.

113
Manual de INFECTOLOGÍA PLUS MEDIC A

LEISHMANIOSIS PLUS MEDIC A

Introducción La frecuencia es mayor en los adolescentes y


adultos jóvenes.
◆ La forma cutánea andina afecta pre-
◆ Las leishmaniasis son un conjunto de
dominantemente a los menores de 15 años -en
enfermedades muy diferentes entre sí
especial, a los niños menores de 5 años-
Son producidas por distintas especies de un protozoario ◆la forma mucocutánea, al grupo mayor de 15 años.
perteneciente al género Leishmania. La leishmaniasis es considerada como una
◆ Estas enfermedades de evolución crónica se enfermedad ocupacional en las personas que se
caracterizan por comprometer : trasladan por motivo de trabajo a las áreas
la piel, mucosas y vísceras dependientes de la especie de endémicas.
Leishmania causante y de la respuesta inmune del
huésped. No existe predilección por alguna raza ni sexo.
-►Agente causal : alguna especie de Leishmania Sin embargo, la incidencia es más alta en los
-►Vector : insectos dípteros hematófagos hombres, posiblemente como resultado del contacto
-►Reservorio : vertebrados ocupacional.

Epidemiología Agente etiológico


Es un protozoario del :
En el Perú, la leishmaniasis es :
-► Género : Leishmania
◆ La 2° endemia de tipo tropical y
◆ La 3° causa de morbilidad por enfermedades -► Reino : Protista
transmisibles luego de la malaria y la tuberculosis. -► Subreino : Protozoa
-► Orden Kinetoplastida
En el Perú -► Familia: Trypanosomatidae
La leishmaniasis es endémica y constituye un grave
problema de salud pública. En la actualidad, el género Leishmania se
divide en dos subgéneros, según su desarrollo en
Existen las formas cutáneo andina y la mucocutánea o el intestino de los flebótomos vectores:
selvática Leishmania, en el intestino medio o anterior, y
Son endémicas en: Viannia, en el intestino posterior, medio y anterior
Ancash, Ucayali, Junín, Loreto, San Martín, Amazonas, de los flebótomos.
Ayacucho, Cajamarca, Cusco, Huánuco, Cerro de Pasco y
Madre de Dios; primariamente en los Andes y en la Las leishmanias se presentan bajo dos formas
diferentes:
Amazonía. Promastigote
√ Móvil y flagelado
Leishmaniasis cutánea
La zona endémica se extiende : √ Libre, alargado
√ A través de los Andes y √ De 10 a 14 por 1,5 a 3,5 mm
Se multiplica en el vector y migra a la parte anterior
√ Los valles interandinos entre los 600 y los 3000 msnm,
del mosquito y está allí hasta ser inoculada.
Leishmaniasis mucocutánea.
Las zonas de selva alta y selva baja por debajo de los
2000msnm.

La mayoría de los casos en el Perú es causada por L.


brasiliensis y, ocasionalmente, por L. peruviana.

114
Manual de INFECTOLOGÍA PLUS MEDIC A

Amastigote Subgénero Leishmania


√ Es inmóvil, intracelular, dentro de los macrófagos y Leishmania (Leishmania) chagasi *
otras células del sistema reticuloendotelial del huésped L. (L) enrietti
vertebrado. L. (L) mexicana*
√ Redondeada u ovoide L. (L) pifanoi*
L. (L) hertigi
√ De 2,5 a 5,0 por 1,5 a 2,0 mm
L. (L) amazonensis*
L. (L) deanei
L. (L) aristidesi
L. (L) garhami*
L. (L) venezuelensis*
L. (L) forattinii
En el Perú se han identificado 5 especies de
leishmania: Subgénero Viannia
1) Leishmania (V) brasiliensis Leishmania (Viannia) brasiliensis*
2) Leishmania (V) guyanensis L. (V) peruviana*
3) Leishmania (V) peruviana L. (V) guyanensis*
4) Leishmania (V) lainsoni L. (V) panamensis*
5) Leishmania (L) amazonensis L. (V) lainsoni*
* Especies que infectan al hombre
Leishmaniasis selvática o espundia
Ciclo biológico
La leishmania es heterogénea y completa su ciclo
biológico usando dos huéspedes. Ciclos:
Θ Uno, principalmente silvestre, en el que la leishmania
circula entre los reservorios naturales y mantiene el
ciclo con la participación de los vectores propios de la zona
endémica.
ΘEn un segundo ciclo, los vectores infectados
pueden atacar al hombre y a los animales domésticos o
peridomésticos.
ΘSe puede producir un tercer ciclo, en el que el propio
enfermo con leishmaniasis se constituye en reservorio.

1.El vector ingiere sangre con macrófagos


infectados con amastigotes
En la amazonía se reconocen 3 especies como
agentes causantes de leishmaniasis llamada ►El ciclo empieza cuando el vector pica a un
vertebrado infectado
también:
►Con la sangre ingiere macrófagos infectados
L. (L) amazonensis, L. (V) guyanensis y L. (V) brasiliensis. con amastigotes
Entre los 600 y 2 000 msnm (Ayacucho, Pasco, San La transformación del amastigote a
Martín, Huánuco) se ha reportado la presencia de L. (V) promastigote ocurre dentro de las
lainsoni. siguientes 24 a 48 horas.

Forma cutánea andina (UTA) ►Los promastigotes se multiplican activamente por


Es causada por la L. (V) peruviana, que se desarrolla división binaria longitudinal.
entre los 600 y 3 000 msnm. ►Después de la replicación en el intestino, los
En América Latina, los subgéneros de Leishmania y Viannia
promastigotes migran al esófago y la faringe
contienen numerosas especies de las que sólo algunas
del mosquito.
infectan al hombre:

115
Manual de INFECTOLOGÍA PLUS MEDIC A

Θ Los amastigotes libres entran en nuevas células del


sistema fagocitario mononuclear, donde se
multiplican de nuevo.

EL VECTOR

La leishmaniasis es transmitida por


la picadura de flebótomos,
pequeñas moscas que abundan todo
el año en las zonas tropicales y en el
verano, en las zonas templadas.

Se reconocen 5 géneros de flebótomos principales:


Phlebotomus, Sergentomya, Lutzomyia, Warileya y
Brumptomya.

Pero, se reconocen como vectores de la


leishmania solo a 2:
2.Cuando el vector infectado pica a un huésped le ►En Europa, Asia y África, el género
inocula entre 10 y 100 promastigotes presentes Phlebotomus
en la proboscis y que penetran en la dermis. ►En América, el género Lutzomya.

Flebotomus
√ Mosquito pequeño
√ De 1,5 a 3 mm de tamaño
√ Su cuerpo está cubierto de pelos
√ Tiene las alas erectas en forma de 'V'.

Flebótomus:
tiene alas en
forma de “V”
Con “V” de
vengadora

En el Perú, a la Lutzomyia se la conoce


con el nombre de 'manta blanca' o 'titira'.
3. El promastigote es fagocitado por el
macrófago. ¿Dónde vive?
4. El promastigote se transforma a amastigote en
√ En áreas desérticas, en la floresta y en áreas
el interior del macrófago.
peridomésticas.
5. El amastigote se multiplica en las células de
√ Prefiere los lugares húmedos oscuros, en los
diversos tejidos (incluyendo los macrófagos).
que existe abundante vegetación.
Θ Los amastigotes se multiplican por fisión binaria
dentro de los macrófagos. La cantidad de ¿Cómo es su día?
amastigotes puede llegar hasta 200, lo que ocasiona √ Descansa de día en los rincones, anfractuosidades
la distensión y ruptura del macrófago. de las piedras, muros o troncos de los árboles y
vuela al atardecer.

116
Manual de INFECTOLOGÍA PLUS MEDIC A

Las hembras son las únicas hematófagas TRANSMISIÓN


y más activas a la caída del día. Todas las especies de Lutzomyia pueden ser
potencialmente vectores de las leishmanias y
En el Perú se han descrito 131 especies de dependerán de sus preferencias por alimentarse.
Lutzomyia, de las cuales 5 son vectores de ►La mayoría de las especies es silvestre y sólo ataca a
la leishmaniasis tegumentaria. los hombres que penetran en su hábitat.
►Las hembras son las responsables de la transmisión
La especie que predomina es la : de la enfermedad.
-►Lutzomyia peruensis, que es vector de la
leishmania en las provincias de Huarochirí, Otuzco y
Bolognesi. Cuadro clínico
-►L. ayacuchensis, en las provincias de Lucanas y
Parinacochas.
-L. verrucarum, en las provincias de Bolognesi y Huarochirí.
-►L. tejadai, en las provincias de Ambo y Huánuco
-►L. pescei se correlaciona geográficamente con
algunas áreas de leishmaniasis.

Las características clínicas de la


Leishmaniasis tegumentaria en el Perú
presentan diferencias de acuerdo con la
región geográfica donde se presentan:
(no existe Leishmaniasis visceral kala-azar)

UTA
Siendo predominantemente cutánea pura
en las regiones andinas (forma benigna
llamada “UTA”

Evolución
Con fuerte tendencia a curación espontánea en el
curso de un año, dejando cicatrices características) .

ESPUNDIA
Cutáneo-mucosa metastásica en la selva
(forma maligna, llamada “ESPUNDIA”

Evolución
De evolución crónica de difícil control terapéutico),
aunque también en ambas regiones pueden
presentarse las dos formas clínicas.

RESERVORIO Es por estas condiciones que se ha aprobado para el Perú


Existe una gran variedad de animales silvestres y la siguiente clasificación propuesta por Burstein para
domésticos
denominar las diferentes formas clínicas con
En las áreas andinas, se ha encontrado infectado
criterio epidemiológico (procedencia), aspecto
naturalmente al perro y a roedores, como la rata.
En la selva, aún no se ha podido determinar los
clínico y tiempo de evolución.
reservorios.

117
Manual de INFECTOLOGÍA PLUS MEDIC A

Leishmaniasis Cutánea La úlcera característica de la leishmaniasis es:


Θ Redondeada
Periodo de incubación : Θ Indolora
Es entre 2 y 3 semanas Θ Con bordes bien definidos levantados y cortados
(de 2 semanas a 2 meses o más). en forma de sacabocado e indurada que recuerda la
imagen de un cráter.
Cuadro clínico
Lesión dérmica inicial
Después aparece una pequeña lesión inicial
Cuando se desprende la costra :
frecuentemente visible, pero no siempre, que tiene
Se observa :
asiento en las partes descubiertas, principalmente en la
◆ Un fondo granulomatoso, limpio, con exudado seroso
cara y en las extremidades.
no purulento, sin tendencia al sangrado, de color rojizo, a
Características:
veces amarillento cuando hay depósito de fibrina.
► El aspecto típico de la lesión inicial es un leve
►No hay signos inflamatorios, como edema o calor local.
enrojecimiento circunscrito, frecuentemente pruri -
ginoso, ►La piel alrededor de la lesión presenta aspecto y
coloración normales.
► A los pocos días, se convierte en una leve
infiltración papulosa de unos 3mm de diámetro y
Las lesiones se estabilizan y a medida que
con mucha frecuencia con 1 ó 2 diminutas vesículas empieza a prevalecer la respuesta inmune
► Puede dar lugar a una diminuta excoriación por del huésped, la enfermedad tiende a evolucionar
el rascado, que se transforma en una exulceración y a la curación espontánea, en un periodo de
posible punto de partida de un proceso ulcerativo. seis meses a tres años.

Algunas veces, la lesión LA LEISHMANIASIS CUTÁNEA ANDINA


regresiona espontáneamente y PRODUCE USUALMENTE SÓLO LESIONES CUTÁNEAS
origina una fase de silencio Sin embargo, las membranas mucosas pueden estar
sintomático algo prolongado. ocasionalmente comprometidas, directamente relacio-
Un trauma local puede activar
nadas a la contigüidad de una lesión con la mucosa, en el
una infección latente.
caso de lesiones producidas en la cara.
Nódulos linfáticos
Signo precoz en los casos de leishmaniasis ENAM 04-A (2): Clínicamente las lesiones de
cutánea Leishmaniosis cutánea se caracterizan porque:
El inicio de los signos linfáticos puede aparecer antes, al A. Son muy dolorosas .
B. No dan adenopatía regional
mismo tiempo o después de la ulceración, y, en casos muy
C.Curan espontáneamente sin reactivación
raros, puede ser el único signo de infección de leishmaniasis.
D. Evolucionan a úlceras o nódulos
E. Su período de incubación es menor de 7 días
Úlcera dérmica
Después de varios días, la lesión inicial se ulcera Rpta. D
espontáneamente y se cubre de un exudado
amarillento y adherente, que dará lugar a la costra. Leishmaniasis Mucocutánea
►Debajo de la costra, la lesión se extiende en
Las manifestaciones clínicas de la forma mucocutánea se
superficie y profundidad. presentan muchos meses o años después de haber
►Pueden aparecer lesiones satélites que al unirse a la cicatrizado la forma cutánea
inicial, originan una úlcera grande. ►Ocasionalmente aparecen cuando todavía existen las
manifestaciones en la piel.
►Las lesiones mucosas se inician principalmente a nivel
del tabique nasal cartilaginoso y, raramente, en el piso
de la nariz.

118
Manual de INFECTOLOGÍA PLUS MEDIC A

pueden comenzar en otras partes de las vías Las lesiones del paladar son más frecuentemente
aéreas superiores. proliferativas que destructivas; la úvula suele
√Al inicio sólo se aprecia una discreta secreción de moco, hipertrofiarse, ulcerarse o destruirse; pero, las lesiones
como una rinitis o un resfriado. linguales son muy raras.
√Luego, se produce la inflamación de la mucosa, que se
vuelve eritematosa, edematosa y dolorosa; la lesión se
profundiza y produce una pericondritis.
►Hay hipertrofia vascular y de los orificios
pilosebáceos, que produce abundante seborrea.
►Cuando las lesiones están avanzadas, se presenta
exudación y ulceración de la mucosa.

Luego, se compromete el cartílago y se


produce la perforación del tabique Afección de la faringe
Si destruye parcial o totalmente el tabique Cuando se afecta la garganta, la voz es
determinará la caída de la punta de la ronca y hay dificultad para respirar y
nariz. deglutir los alimentos.
►También se puede hallar compromiso gingival e
“Nariz de tapir” interdentario.
El eritema, edema y la ►Las lesiones de la hipofaringe, laringe y tráquea se
infiltración producen au- caracterizan por un compromiso de los repliegues
mento del volumen ariteepiglóticos y aritenoides, que dan lesiones
de la punta de la hipertrofiantes que producen disfonía, afonía y asfixia.
nariz y el ala, que ►La epiglotis también puede estar comprometida y
puede sobrepasar el las cuerdas vocales infiltradas.
surco nasogeniano, Si no hay tratamiento, la enfermedad puede
dando el aspecto de “nariz llevar a la muerte.
de tapir”.
►La perforación del tabique nasal y el achatamiento de Leishmaniasis cutánea difusa
la nariz sin ulceración son propias de la leishmaniasis
mucocutánea (espundia) La leishmaniasis cutánea difusa ocurre en un huésped
►No son observadas en la leishmaniasis cutánea enérgico con pobre respuesta inmune celular.
÷La enfermedad se inicia bajo la forma de lesiones
andina, en la que, de preferencia, las alas de la nariz
localizadas, de aspecto nodular o en placa
son carcomidas. infiltrada, que poco a poco se diseminan a todo el
En un tercio de los casos, las manifestaciones
cuerpo.
mucosas son primarias, sin antecedente de
Apariencia de lepra lepromatosa
lesión cutánea.
La presencia de ciertas lesiones le dan el
aspecto de lepra lepromatosa
Compromiso nasal
Θ Nódulos aislados o agrupados
Los pacientes con compromiso nasal presentan, como
Θ Máculas
sintomatología:
Θ Pápulas
Catarro nasal, ardor, prurito y respiración
forzada. Θ Placas infiltradas
Al examen, se aprecia la mucosa nasal congestionada, una Θ Úlceras y, algunas veces, lesiones verrugosas
costra hemorrágica o una úlcera granulomatosa infiltrada. Si de límites imprecisos,
hay infección sobreagregada, la secreción es purulenta. La enfermedad no invade órganos
internos.
Si la enfermedad progresa y se profundiza, el proceso se
extiende del vestíbulo al labio superior, paladar, pilares,
Las lesiones no curan espontáneamente y
úvula y la garganta. El labio superior suele ulcerarse y tienden a la recaída después del
destruirse poco a poco y compromete parte de la nariz. tratamiento.

119
Manual de INFECTOLOGÍA PLUS MEDIC A

Leishmaniasis Visceral Existe una linfadenopatía generalizada, en especial de


los ganglios mesentéricos, epistaxis, hemorragia
Sinónimo: Kala Azar gingival, edema y ascitis.
Etiología : complejo L. donovania La leishmaniasis visceral a menudo es fatal
incluye a la si no se efectúa tratamiento adecuado.
►L. donovani en el subcontinente Indio, Asia y África La piel se encuentra hiperpigmentada.
►L. infantum, en el mediterráneo
►L. chagasi, en Sudamérica. Diagnóstico
Período de incubación Se debe considerar lo siguiente:
Después de la picadura del vector, existe un periodo de
1°. Antecedentes epidemiológicos:
incubación que varía de 4 a 10 meses. procedencia, visita a áreas endémicas, presencia
En muy pocos casos se encuentran lesiones en la de lesiones cutáneas anteriores que pueden
puerta de entrada, ya que la mayoría de las veces haber sido catalogadas como leishmaniasis o no,
pasa desapercibida y tiene una evolución crónica. que demoraron en cicatrizar teniendo el
antecedente de haber estado en un área endémica
Cuadro clínico de leishmaniasis.
La progresión a leishmaniasis visceral típica 2°. Diagnóstico clínico: que de acuerdo a las
usualmente ocurre entre los 3 y 8 meses características mencionadas nos inclinarán a si es
después de la infección; aunque se han reportado una leishmaniasis cutánea o mucocutánea.
casos tempranos, como de dos semanas. 3°. Diagnóstico de Laboratorio: se usan los
Sin embargo, después de la infección la mayoría de los métodos directos (métodos parasitológicos) y los
casos permanece asintomática o está asociada con métodos indirectos (métodos inmunológicos).
síntomas leves que, eventualmente, se resuelven en
forma espontánea. DIAGNÓSTICO DE LABORATORIO

Manifestaciones clínicas de la leishma- Métodos directos o parasitológicos


niasis visceral típica En el diagnóstico parasitológico hay dos
◆ Fiebre: casi siempre es progresiva y elevada, alternativas:
remitente o intermitente, que dura semanas y se alterna ◆Demostrar que el paciente está albergando
con periodos febriles, que también duran semanas. la leishmania
SÍNTOMAS TÍPICOS: Mediante la visualización, en el frotis o en la
►Fiebre-Anemia histopatología, de amastigotes en tejidos
►Esplenomegalia masiva infectados.
►Hiperpigmentación Es el examen más utilizado actualmente para el
diagnóstico de la leishmaniasis en el país.
◆Intentar el aislamiento directo de
los promastigotes
En cultivos in vitro de las lesiones sospechosas.
◆Inoculación de animales de laboratorio
A partir de los que se puede aislar y caracterizar a la
Leishmania.

Métodos Indirectos o Inmunológicos


-►Respuesta inmune celular (intradermorreacción de
Montenegro o leishmanina) .
-►Respuesta inmune humoral a través de anticuerpos
específicos desarrollados como consecuencia de la
enfermedad (Elisa/DOT Elisa, inmunofluorescencia
indirecta (IFI)).
Esplenomegalia masiva

120
Manual de INFECTOLOGÍA PLUS MEDIC A

Además: PCR, anticuerpos monoclonales y/o Después del tratamiento y la cura clínica en
electroforesis de isoenzimas. ambas formas de la enfermedad, los títulos caen
o desaparecen completamente.
Intradermorreacción de Montenegro Un tercio de los pacientes permanecen
SINÓNIMO: Prueba de la leishmanina seropositivos después de los 30 años de
Es una reacción de hipersensibilidad tardía que evalúa enfermedad.
la inmunidad mediada por células.
►Consiste en la aplicación de un antígeno extracto La IFI puede presentar reacción cruzada
soluble preparado a partir de promastigotes (falsos positivos) con :
procedentes de cultivo. ►Leishmaniasis visceral, Enfermedad de
Chagas u otras infecciones por
tripanosomideos, Hanseniasis, Malaria.

Reacción en cadena de la polimerasa (PCR)


Es un método que se está usando rutinariamente
para confirmar el diagnóstico de leishmaniasis.
►La identificación puede ser hecha de una biopsia sin
requerir necesariamente un cultivo.
►Se aplica intradérmicamente en la cara anterior del
Los resultados comparativos entre la PCR y
antebrazo izquierdo del paciente y se hace la lectura a las los métodos de detección parasitológicos
48 a 72 horas. muestran una mejor sensibilidad del primero
para fines de diagnóstico.

ENAM 03-A (84): La prueba de oro en leish-


maniosis es :
A. Prueba de ELlSA
B. Hallazgo de amastigotes en el frotis
C. Análisis de isoenzimas
D. Buena respuesta al tratamiento con glucan
E. Reacción intradérmica de Montenegro positiva
Se considera positiva si es mayor de 10mm.
Rpta. B
►La prueba aparece positiva 1 a 3 meses después
de haber adquirido la infección
►Permanece positiva de por vida en pacientes con LCL y Tratamiento
LCM
►Es negativa en los pacientes con LCD, forma visceral y Primera línea: Antimoniales Pentavalentes.
en inmunosuprimidos. Segunda línea: Anfotericina B.
Tiene un 96% de positividad en los tres Los esquemas de tratamiento se aplican de acuerdo
primeros años de iniciada la enfermedad. al diagnóstico clínico epidemiológico de los casos, ya
sean estos casos de:
Inmunofluorescencia indirecta (IFI) y leishmaniasis cutánea andina o selvática y la
pruebas enzimáticas Elisa leishmaniasis mucocutánea.
Estas pruebas detectan anticuerpos
antileishmania circulantes en el suero del
paciente a títulos bajos, el método más empleado
es la inmunofluorescencia indirecta (IFI).
►Las lesiones múltiples, tanto cutáneas como
mucosas, están asociadas a títulos más altos.
►De otro lado, las lesiones mucosas presentan
títulos más altos que las lesiones cutáneas y muestran
títulos elevados persistentemente.

121
Manual de INFECTOLOGÍA PLUS MEDIC A

Tratamiento de la Leishmaniasis cutánea ►En los casos de lesión cutánea hasta alcanzar una dosis
andina o Uta acumulada total entre 1 a 1,5gr.
Antimoniales Pentavalentes: ►Debe ser administrado por vía EV diluido en 500 ml de
√ Antimoniato de N-metil-glucamina dextrosa al 5%.
√ Estibogluconato de sodio
◆Dosis: 20mg SbV/Kg de peso/día. Tratamiento de la Leishmaniasis visceral
►Tiempo: 10 días, por vía IM o EV. Antimoniales Pentavalentes:
►Aplicar diariamente en una sola dosis. √ Antimoniato de N-metil-glucamina
►El uso de la vía IM puede causar mucho dolor por lo √ Estibogluconato de sodio
que es preferible usarla sólo cuando hay dificultad de ◆Dosis: 20mg SbV/Kg de peso/día.
utilizar la vía EV. Tiempo: 30 días por vía IM ó EV.
►La vía intralesional también puede ser utilizada,
sin embargo, algunos pacientes refieren mucho dolor. Aplicar diariamente en una sola dosis

Tratamiento de la Leishmaniasis mucocutánea


(Espundía)
Tratamiento de Primera línea:
Antimoniales Pentavalentes:
√ Antimoniato de N-metil-glucamina.
√ Estibogluconato de sodio.
Lesión Cutánea Primaria
◆Dosis: 20mg /Kg de peso /día.

◆Tiempo: 20 días por vía IM ó EV.


Aplicada diariamente en una sola dosis.

Si hay falta de respuesta a 2 cursos de tratamiento con


antimonial pentavalente, se debe iniciar anfotericina B.
Lesión Mucosa
◆ Indicación: casos de leishmaniasis mucosa con
compromiso leve y moderado de mucosas.

◆Dosis: 20mg SbV/Kg de peso/día. Guia de Leishmaniose MINSA


Perú
◆Tiempo: 30 días por vía IM ó EV. Aplicar diariamente
en una sola dosis.

En los casos severos se utilizará tratamiento de


segunda línea con anfotericina B.

Tratamiento de Segunda Línea: Anfotericina B

◆Indicación: Falla terapéutica a 2 series completas y


dosis adecuada de tratamiento con antimoniales
pentavalentes en los casos de lesión mucosa leve a
moderada y de inicio en los casos de lesión mucosa
severa.
◆Dosis: 0,5mg/kg/día a 1mg/kg/día hasta un máximo
de 50mg/día.
◆ Tiempo de tratamiento: En el caso de
lesiones mucosas hasta alcanzar una dosis
acumulada total entre 2,5 a 3gr. .

122
Manual de INFECTOLOGÍA PLUS MEDIC A

INFECCIÓN POR VIH PLUS MEDIC A


√ Adicción a drogas parenterales
Definición Θ Riesgo 1 : 150

Es una enfermedad causada por el virus de la √ Transfusión de sangre contaminada


inmunodeficiencia humana (VIH). Θ Riesgo 95%

La enfermedad destruye el sistema


inmunológico, lo que hace más difícil para el
organismo combatir las infecciones

Debido a las pruebas de detección rápida de


la infección por VIH el riesgo de contagio por
transfusión con sangre contaminada en USA es 1: 1
OOO OOO

Epidemiología √ Recién nacidos de madres con in -


1
Riesgo de trasmisión del VIH : fección por VIH que no han recibido
tratamiento o niños que no han recibido
√ Relaciones sexuales no protegidas
(principal forma de trasmisión) profilaxis perinatal
Θ 13-40% de los recién nacidos se infectan
Θ Homosexual Pasivo 1 : 1 00 and 1 :30
Θ Homosexual Activo 1 : 1 000
Θ Mujer heterosexual 1 : 1 000
Θ Varón heterosexual 1 : 1 0,000
Θ Felatio con eyaculación 1 : 1 000.

√ Uso de jeringas con sangre infectada


Θ Riesgo 1 :300
PREVALENCIA
Aumentan el riesgo: agujas huecas, con sangre
En estados UNIDOS
visible y estadío avanzado de la enfermedad

El riesgo de contagio por salpicadura Θ 1 ,2 1 8 , 400 personas ≥ 13 años con


de sangre infectada a la conjuntiva es infección por VIH
mucho más bajo
Θ 156,300 no saben que están infectados con
1.Centers for Disease Control and Prevention. VIH
HIV Surveillance Report. 20 1 4;26. http :/
/www.cdc.gov/hiv/library/reports/ surveillance Θ 50,000 nuevas infecciones cada año

123
Manual de INFECTOLOGÍA PLUS MEDIC A

2. NOTI-VIH.RENACE. Grupo Temático


TB, VIH-SIDA. Dirección General de
Epidemiología. Ministerio de Salud 2017

Θ En gestantes es de 0.23%
2
PREVALENCIA Θ En población de hombres que tienen sexo
con otros hombres (HSH) y mujeres
En PERÚ
transgénero es de 12.4%
Θ Principal vía de transmisión : sexual 97%
Θ De madre a hijo 2% y parenteral 1%
Θ El 77 % casos notificados de SIDA son
varones
Θ El 23% de casos notificados de SIDA son
mujeres
Θ La razón hombre /mujer es de 3:1
Θ La mediana de la edad de casos de SIDA es
de 31 años

124
Manual de INFECTOLOGÍA PLUS MEDIC A

FIg 1 Estructura del VIH

Virus VIH
Human T lymphotropic virus type III (virus
Nueva York y San Francisco linfotrópico T humano tipo III, HTLV-III).
Se detectaron casos de Sarcoma Adjudicándose la realización del descubrimiento.
de Kaposi , de infección por
Pneu- mocystis jiroveci y A partir de ese momento se denominó
citome- galovirus en varones al virus human inmunodeficiency virus
homo- sexuales (HIV)
Se sospechaba en un agente infeccioso !Preséntate y canta.........!
relacionado con las prácticas
homosexuales masculinas

Se postuló la existencia de un nuevo


retrovirus humano como agente causal.

En 1971 se había descubierto la transcriptasa


Yo soy ......el VIH
inversa y 2 virus linfotrópicos humanos, uno
de ellos producía un tipo de leucemia, el HTLV. Familia: Retroviridae
Género/ Grupo: Lentivirus
Louis Montagnier, del Instituto
Pasteur de Paris encontró un virus Subgénero:
al que llamó lymphadenopathy Virus de Inmunodeficiencia de primates.
associated virus (virus asociado Especies tipos:
a la linfoadenopatía, LAV). Virus de la Inmunodeficiencia Humana-
HIV
Gallo y su equipo en Estados Unidos, que había Variedades: HIV-1 y 2 HIV
descubierto el HTLV, recibió las muestras de Tamaño: 80- 130 nm
Montagnie y se encargó de caracterizar al
virus llamándolo :

125
Manual de INFECTOLOGÍA PLUS MEDIC A

Partes: Envoltura proveniente de la célula Virulencia: alta


infectada Capacidad antigénica: buena.
Enfermedad: sida.
√ Cápside externa de simetría isocaédrica Patogenicidad: variación en la presenta -
√ Cápside interna conoide ción de la enfermedad y en relación al ini
√ Core que contiene ARN y tres proteínas de -cio de la enfermedad
función enzimática. Fortalezas:

Genoma: √ El ADN del provirus se integra al ADN


cromosómico de la célula infectada.
linear con dos cadenas ARN + no √ Diversidad genética.
complementarias.
Codifica proteínas estructurales, enzimáticas y El virus es inactivado por:
regulatorias. Alto grado de mutación.
√ Hiploclorito de sodio al 5%; alcohol
√Proteínas enzimáticas: transcriptasa etílico 70%; yodo povidona 2,5%;
inversa, proteasa, integrasa. peróxido de hidrógeno 6%.
√ Sensible a solventes lipídicos al po -
√ Transmisión: horizontal (sanguínea, sexual) seer envoltura, se inactivan fácilmente
y vertical. con eter, cloroformo, sales biliares o
detergentes aniónicos)
√ Puerta de entrada: inoculación directa
√ Relativamente termosensible (calor
al torrente sanguíneo, por lesiones en piel,
a 56ºC, ebullición por 30 min., o
mucosas intactas o lesionadas.
esterilización.
√ Tropismo: √ Relativamente resistentes a las radia
Hacia la molécula CD4+ que se encuentra ciones ionizantes y a la luz ultravioleta
en la superficie de linfocitos CD4+ al poseer ácidos nucléicos con dos
cadenas ARN + no complementarias.
√ Hacia células presentadoras de antí -
geno (monocitos, macrófagos y células dendrí
ticas)
VIH-1 y VIH-2
√ Incluso tiene la capacidad de infectar VIH-1 y VIH-2 son virus ARN, retrovirus
células que no se dividen. de
◎Familia
Carácter infectivo: medio. Retroviridae
◎Género
Patogenia: evolución lenta, una vez producida
Lentivirus.
la infección el virus persiste durante toda la
vida del hospedador.
En general presenta al menos tres etapas: Han sido identificados
√ Inicial o aguda cada una está compuesta de múltiples
√ De latencia subtipos.
√ Clínica (sin latencia viral, el virus continúa
Todos los subtipos del VIH-1 tienden a causar
replicando) una enfermedad similar, pero difieren en la
√ Tardía, donde se manifiesta la enfermedad. distribución mundial.

126
Manual de INFECTOLOGÍA PLUS MEDIC A

Esto tiene implicancias futuras con respecto a las Núcleocápside


vacunas, ya que el subtipo B, que es
predominante en los países desarrollados, rara vez √ Dentro de la cápside se
se encuentra en los países en desarrollo. encuentra el core formado
por una segunda cápside
Genéticamente, el VIH-1 y VIH-2 son proteica, la
superficialmente similares. nucleocápside, en forma
de cono (proteína p24) que
protege al nucleoide formado
por dos cadenas de ARN
monocatenario.

Θ Nucleocápside : La parte central


contiene :
√ 2 copias del genoma viral
(ARNss)
!Parecidos pero no iguales...!
√ Proteínas del núcleo
Sin embargo cada uno (VIH-1 y VIH.2) √ Transcriptasa inversa
contiene genes únicos y su propio proceso
de replicación ◆ Junto al ácido nucleico se encuentran
íntimamente relacionadas dos proteínas,
VIH-2 la p7 y la p9.

◆ Le corresponde un riesgo ligeramente ◆ El virión codifica también para-proteínas


menor de la transmisión. con función enzimática:

◆La infección por VIH-2 tiende a progresar √ Integrasa


más lentamente a síndrome de √ Proteasa
inmunodeficiencia adquirida (SIDA). √ Transcriptasa inversa.
Las personas infectadas con el VIH-2 tienden
◆Integrasa: cataliza la inserción de la
a tener una carga viral más baja que las
doble cadena lineal del ADN viral al
personas con VIH-1
cromosoma celular.

Estructura (Fig 1 ) ◆ Proteasa: divide específicamente la


pro -teína gag-pol del VIH-1.
Cápside:
◆Transcriptasa inversa que tiene
√ Estructura proteica formada
funciones de ribonucleasa y polimerasa:
por una serie de monómeros
√ Sintetiza ADN a partir del ARN
llamados capsómeros.
viral √ Como ribonucleasa degrada los
√ Simetría icosaédrica
híbridos ARN- ADN durante la
(poliedro de 60 caras triangulares
con algunas hexámeras transcripción inversa.
y pentámeras) La transcriptasa inversa no tiene actividad de
corrección de lectura por lo que no puede
corregir errores durante la síntesis de ADN ,
debido a esto el VIH_1 tiene una alta
variabilidad en la secuencia nucleótida

127
Manual de INFECTOLOGÍA PLUS MEDIC A

Genoma
Fig 2 Genoma del VIH del VIH-1

Genes estructurales
√ Gag: núcleo viral
Glicoproteinas gp120 y gp41 Cápside (CA), matrix (MA),
nucleocápside(NC)
Θ El virión tiene envoltura compuesta :
√ Lípidos derivados de la célula huésped √ CA: se une a las
√ Proteinas propias del virión ciclofilinas
Θ Las proteínas forman prolongaciones denomi √ NC: se une al ARN
-nadas espículas que están constituidas √ MA: posee una señal de localización
por 2 proteínas virales: nuclear (NLS).
√ gp120, más externa, que se une con altísi √ Pol: enzimas:
-ma afinidad al receptor de membrana de la Transcriptasa inversa (RT)
célula CD4+ y es capaz de inducir la res- ARNsa H; proteasa (PR)
puesta inmune Integrasa(IN).
√ gp41, más interna y pequeña que atra √ RT y PR son los objetivos principales
-viesa la membrana , que es responsable de de los fármacos antivirales.
la fusión con la membrana celular en √ IN contiene NLS
el momento en que el virión ingresa a la célula. √ RT es responsable de la mutación viral.
En la envoltura pueden existir diversas proteínas √ Env: envoltura viral (glicoproteínas de
celulares, la más importante es la del : superficie (gp120/SU), transmembrana
(gp41/TM)). El gp120/SU se une al receptor
√ Complejo mayor de histocompatibilidad CD4 y el TM se funde con la membrana
humana de clase II (MHC de clase II). celular

Genes de transactivación: genes esencia-


Matriz
Entre la envoltura y la cápside se encuentra la les para la replicación in vitro:
matriz formada por la proteína p17 √ Tat: transactivador de la transcripción viral,
se une a la región afín Tar del ARN, acelera la
trans -cripción viral.
√ Rev: regula la expresión viral, se une a una
región afín RRE del ARN, regula el transporte y
unión de las secuencias del ARN viral.

128
Manual de INFECTOLOGÍA PLUS MEDIC A

Genes accesorios: genes no esenciales


para la replicación viral in vitro. I-Etapa de iniciación:
√ Nef: factor negativo (nombre erróneo),
regula a la baja el CD4; MHC-I . Se une a las Adsorción
quinasas celulares, esencial para la inducción de y
la enfermedad in vivo. penetració
√ Vpu: proteína viral desconocida: regula a la n
baja el CD4, MHC-I; promueve la liberación
viral, excepto en el VIH-2. Decapsi-
dación o
√ Vif: factor de infectividad viral: facilita la
maduración del virión. desnuda-
miento
√ Vpr: proteína viral regulatoria: detiene la
proliferación celular, contiene un NLS.
√ Vpx: proteína viral X (solo en los VIH-2):
permite la inefcción de los macrófagos y la
diseminación viral
Adsorción y PENETRACIÓN
Mecanismos de replicación del VIH
En esta etapa participan:
1.Etapa de iniciación
√ Proteínas gp120 y gp 41 de la en
2.Etapa de expresión y replicación del -voltura viral
genoma.
√ Receptores y correceptores
3.Etapa de ensamble, maduración y de las células hospedadoras.
egreso de los virus progenie.

129
Manual de INFECTOLOGÍA PLUS MEDIC A

◆La infección se inicia con la unión de la Decapsidación o desnudamiento


proteína gp 120 de la partícula viral con Para que comience el proceso de
células que tienen la molécula CD4 en su replicación del VIH
membrana. √ El VIH debe desprenderse de su
cubierta proteica. Se libera el cápside
que contiene el material genético y 3
proteinas
Esto sucede gracias a la actividad de enzimas celu
-lares que degradan la cápside viral.

Fig 3 Unión del gp120 con los receptores CD4


del infocito
!Vaya, vaya! No sabía
Θ Esta unión modifica la estructura de la que el VIH para
gp120 lo que facilita la unión de esta proteína a replicarse tiene que
un correceptor de quimiocina (hay más de 7) hacer antes un streap
tease
para que se de la infección se
requiere las moléculas CD4 y la II.- Etapa de expresión y
CCR5 CXCR4.
o REPLICACIÓN del genoma.

Θ Esta unión induce una modificación confor - 2.TRANSCRIPCIÓN inversa en el


macional de la proteína gp 41, quedando ex - citoplasma
puesta una región, llamada péptido de fusión, que se
◆ Transcripción inversa en el citoplasma
inserta en la membrana celular iniciando la fusión
Luego de la liberación del material genético y de
de las dos membranas, ingresando el core al
las enzimas , la transcriptasa inversa usando como
citoplasma.
molde el ARN del virus sintetiza una hebra de ADN
Existen dos cepas de VIH con (transcripción inversa) que posteriormente se convierte
distinto tropismo: en un ADN lineal bicatenario
√ Cepa R5 del VIH antes llamada M-
tropic se detectan en estadios
tempranos de la infección e infectan a
macrófagos (tienen un correceptor CCR5)
√ Cepa R4 del VIH antes denominada
tropic, se encuentra en las etapas tardías
de la in -fección y tienen tropismo por los
 La transcriptasa inversa
linfocitos T, cuyos co -receptores son las sintetiza una cadena de
moléculas CXCR4. ADN utilizando como molde
el ARN viral

130
Manual de INFECTOLOGÍA PLUS MEDIC A

Θ El ADN viral bicatenario migra al


núcleo de la célula. del huésped

Θ los virus pueden de esta manera ingresar


Luego separa la cadena al núcleo celular en células que no se
de ARN viral del ADN viral dividen, como los macrófagos y monocitos.

Después genera la
cadena completa de ADN
viral

III.- Etapa de
INTEGRACIÓN,TRANSCRIPCIÓN y
El ADN recién formado se
TRADUCCIÓN
incorpora al genoma del huésped
3.INTEGRACIÓN mediante la acción de la enzima
integrasa
◆ Acción de la INTEGRASA

La integrasa actúa rompiendo los


fragmentos de ADN en los extremos 3´ de
cada cadena de ADN

 La integrasa rompe los


fragmentos de ADN
la integrasa viral cataliza la integración
√ Genera asi extremos compatibles del ADN al genoma celular, formándose
para que se pueda unir al ADN humano (se un provirus, que puede permanecer inactivo
forma el ADN lineal bicatenario ) durante meses o años.

◎ Este provirus tiene en sus extremos dos


secuencias reguladoras llamadas LTR que
actúan en la transcripción de los genes del
ADN viral para sintetizar el ARN viral.

131
Manual de INFECTOLOGÍA PLUS MEDIC A

◎ El provirus es transcripto a un ARN pre 5.TRADUCCIÓN


cursor que es luego procesado para
generar, en el cito plasma, el genoma El ARNm pasa al citoplasma y se dirihe a los
ribosomas que “leen” las secuencias
viral completo, quedando una parte
nucleosídcas y las traducen a estructuras
como ARN sub -genómico. proteicas mediante el ARN. También se
◎ Luego de ser transportado al forma la progenie del ARN vírico utilizando
citoplasma: la polimerasa celular

√ Una parte del ARN completo será el


genoma de los viriones
√ Otra parte funcionará como ARNm para la
expresión de los genes virales.

Las proteínas virales sintetizadas son


procesadas por la actividad de proteasas.
Las proteínas de la envoltura del virus se Las proteinas después son procesadas por la
sintetizan y se transportan hasta la superficie de la tercera enzima viral la proteinasa la cual
célula. acorta las líneas proteicas para formar
distintos tipos de proteinas
4.TRANSCRIPCIÓN

Durante la reproducción celular la


información contenida en el ADN viral se
transcribe a ARNm para la construcción de
nuevos virus

IV- Etapa de
ENSAMBLAJE, MADURACIÓN
y egreso de los virus progenie

6.ENSAMBLAJE

Después de formadas las proteinas se forma


la cápside , la cual contiene los elementos
para la invasión viral , esta se dirige a la
membrana celular donde sale en una vesícula
que luego conformará su cápsula

132
Manual de INFECTOLOGÍA PLUS MEDIC A

7.LIBERACIÓN En este punto, la carga viral suele ser muy


alta, y el recuento de células T CD4 cae
◆Luego al momento de la activación del precipitadamente.
linfo cito el material viral se multiplica
◆Nuevas generaciones de partículas Con la aparición de anticuerpos anti-VIH y
infectivas por gemación respuestas de células T CD8, las caídas de
◆Se acumulan partículas virales en el la carga viral llegan a un estado de
inte -rior de la célula infectada equilibrio y se produce el retorno de células T
produciéndose la destrucción de esta, CD4 a niveles ligeramente más bajos del
liberándose el exterior esas partículas rango de referencia.
virales , la mayoría de las cuales son
defectivas ya que carecen de la cubierta La seroconversión puede tardar un par de
necesaria semanas, hasta varios meses.

Infección asintomática por VIH


En esta etapa de la infección, las personas infec
-tadas con el VIH presentan pocos signos o
síntomas durante aprox. una década o más.

La replicación viral está en curso durante


este tiempo, y la respuesta inmune contra el
virus es eficaz y vigorosa.

En algunos pacientes, la linfadenopatía


generalizada persistente es un signo
externo de la infección.
FASES DE LA INFECCIÓN POR Durante este tiempo, la carga viral, no se
HIV trata, tiende a persistir en un estado
relativamente estable, pero el recuento de
Se describen 3 fases distintas: células T D4 disminuye constantemente.

Seroconversión aguda No hay evidencia firme que la iniciación de la


terapia temprana en el período
Durante esta fase, la infección se ha
asintomático sea eficaz, aunque el inicio muy
estable -cido, y es creado un reservorio tardío es sabido que da lugar a una respuesta menos
proviral. eficaz a la terapia
Este depósito está formado por células SIDA
persistentemente infectadas, típicamente
macrófagos, y liberación constante del virus. Cuando el sistema inmunológico tiene un
daño bastante significativo, las infecciones
El reservorio proviral, parece ser muy estable. No oportu -nistas comienzan a desarrollar,
disminuye con la terapia antiviral agresiva y la vida entonces la persona se considera que tiene SIDA.
media es tal que la erradicación no es una expectativa
viable. Un recuento de células T CD4 inferior a 200/μL
también se utilizapara diagnosticar el SIDA,
El tamaño del depósito proviral se
correlaciona con la carga viral en estado Algunas infecciones oportunistas se desarrollan
estacionario y es inversamente correlacionado cuando los recuentos de células T CD4 son más
con el anti-VIH CD8 linfocitos T. altos que 200/μL

133
Manual de INFECTOLOGÍA PLUS MEDIC A

INFECCIONES OPORTUNISTAS Categoria C :


Tambiem existen otro grupo de enfermedades
Incluso después de comenzar la terapia y que indican que le paciente con infección por VIH
supresión efectiva de la carga viral, los pacientes se encuentra en categoría C.
con recuentos de CD4 persistentemente bajos
permanecen con alto riesgo de infecciones Encefalo patia VIH
oportunistas, sobre todo durante los primeros 6
meses de tratamiento antirretroviral. Isos poriasis,
INFECCIONES OPORTUNISTAS Kapo si , sarcoma
Nemotecnia
Histo plasmosis diseminada
Categoria C : “Las 6 C “
Leuco encefalopatia multifocal
Candi diasis esofá
esofágica ,traqueal o bronquial .
Cán cer cervical invasivo SI ndrome consuntivo por VIH.
Cito megalovirus (retinitis) TO xo plasmosis cerebral,
Crip tosporidiosis
Linfoma primario SNC
Crip tococosis
Esta nemotecnia tenés que pronunciarla
Cocci dioidomicosis como argentino , sino no funciona ¿viste?
¡Macanudo!
¿Se acuerdan de Candy ? ¿Y de su cancito
Crip-crip?

134
Manual de INFECTOLOGÍA PLUS MEDIC A

Tiempo de aparición de marcadores específicos de infección

Manifestaciones Clínicas En el curso de la infección se pueden


utilizar varios marcadores víricos para
No hay hallazgos físicos específicos de
infección por el VIH: identificar la infección ymonitorizar su
tratamiento.
√ La linfadenopatía generalizada
√ Pérdida de peso. ARN-VIH
√ Evidencia de factores de riesgo o
Θ Es el primer marcador que
infecciones oportunistas concurrentes (por
ejemplo, lesiones, herpética en la ingle, candidiasis oral) aparece tras la infección
pueden ser indicios de infección por el VIH.
Θ A las 2 semanas de la
Muchos pacientes con SIDA desarrollan retinitis infección, generalmente a
por citomegalovirus con pérdida severa de la los 10-12 días.
visión. Θ Se puede detectar por técnicas de
amplificación
Diagnóstico
ADN de VIH
Θ Se puede detectar al mismo tiempo que el ARN-VIH, el
ADN de VIH integrado en el genoma celular (ADN
proviral).

135
Manual de INFECTOLOGÍA PLUS MEDIC A

Antígeno p24 ENAM 2014-A (73): Varón de 29 años acude con


múltiples ganglios cervicales, fiebre y dolor de garganta.
Aparece en suero a los 11-13 días Por factores de riesgo que mencionó el paciente se
sospecha de infección por el VIH. ¿Qué examen es útil si
Θ Se puede detectar, con las técnicas de máxima pensamos que presenta un síndrome retrovirai agudo?
sensibilidad, aproximadamente durante 1 mes y
a.Elisa-VIH
medio b.Western Blot
c.Recuento de CD4
Anticuerpos
d.IFI-VIH
e.Carga Viral
Θ Se detectan en el suero a las 3
ó 4 semanas de la infección, con Rpta E
una media de 22 días, y
Θ Prueba de VIH de alta sensibilidad
alcanzan su concentración mediante PCR
máxima a las 10-12 semanas3.
Θ A los tres días del contagio por VIH, la carga
3.Ann Ist Super Sanitá. 2010; 46:5–14. viral en la sangre crece y el virus empieza a ser
detectable a través de técnicas ultrasensibles como
la prueba de VIH por PCR cuantitativa que
puede detectar desde sólo 20 copias por mililitro de
sangre, una cantidad realmente pequeña teniendo en cuenta
que durante los primeros días de la infección
la carga viral puede llegar hasta millones de copias por
mililitro de sangre.

Θ Dado que la carga viral no evoluciona de la


misma manera de unas personas a otras, la
recomendación es esperar al menos 7 días
desde la exposición para realizar un
análisis de carga viral

período ventana

Es el intervalo de tiempo que existe entre


la infección y la aparición de anticuerpos o
seroconversión

se caracteriza por :
La sensibilidad de estas pruebas para
√ Presencia de ARN-VIH y ADN proviral cargas virales bajas está por encima del
99%
√ Antígeno p24
√ Ausencia de anticuerpos específicos.

136
Manual de INFECTOLOGÍA PLUS MEDIC A

ΘPrueba de VIH rápida en 20 min ELISA para VIH


Θ Esta prueba de VIH rápida detecta de
ELISA : Ensayo inmunoabsorbente ligado a
forma simultanea los anticuerpos que
enzimas
genera nuestro organismo frente a VIH-1 y VIH-2 y
el antígeno p24 de VIH
Tiene alta sensibilidad 99,9%
Θ Esta detección conjunta de
anticuerpos y antígeno p24 incrementa Un resultado negativo no
la efectividad de esta prueba por encima del requiere confirmación ni
test que solo detectan anticuerpos reduciendo el seguimiento serológico,
periodo ventana. excepto en personas con alto
Θ Hay que destacar que el cuerpo puede tardar riesgo de adquirir la in -
en producir anticuerpos fección3
detectables entre 3 y 6 meses después de
haber estado expuestos a VIH. 3. Ann Ist Super Sanitá. 2010 ;46:34–41.
Θ Las pruebas rápidas de cuarta Falsos negativos
generación por lo general permiten
detectar infecciones a partir del Θ En fases iniciales de la infección hasta que se
día 40.
produce la seroconversión
ΘPrueba de VIH COMBO Θ En estadios finales de la misma
(PCR cuantitativa, anticuerpos y p24)
Θ En pacientes con tratamiento inmunosu -
Θ Combinación de tres pruebas para presor, trasplantados de médula ósea, personas con
descartar infecciones por VIH
recientes y establecidas. alteraciones de linfocitos B, en pacientes con
hipogammaglobulinemia,
Θ Una PCR cuantitativa para reducir el
periodo ventana después de contactos de riesgo Θ Infectados por tipos de VIH no detectados
recientes. Además, una pruebas rápida de por la base antigénica
anticuerpos y antígeno p24 para tener Θ Por un error en la identificación de la muestra
información en poco tiempo sobre posibles
infecciones ya establecidas.
ΘPruebas serológicas

L
Especificidad 99,5% y 99.9%

Ante un resultado
positivo de ELISA se debe
seguir con las prue- bas
de confirmación :
√ Western blot
√ inmunoblot recombinante

137
Manual de INFECTOLOGÍA PLUS MEDIC A

Western Blot para VIH

o imunoensayo en línea (LIA) D. Coprocultivo


E. Test de ELlSA para VIH
Pueden incorporar antígenos de envoltura de Rpta. E
VIH-2 lo que permite también diagnosticar este tipo vírico.
Western Blot para VIH
Falsos positivos
Es una metodología en la cual las distintas proteínas
Θ En multíparas, hemodializados, multitrans - víricas ;
fundidos,
Θ Pacientes con hepatitis alcohólica Θ Se separan en función de su peso mole -

Θ Personas con infecciones agudas por otros virus cular mediante electroforesis en gel de poliacri-
como herpes y VHB, vacunados frente a VHB e lamida
Influenzavirus, Θ Se transfieren a una membrana de nitrocelu-
Θ Pacientes con enfermedades autoinmunes, lupus losa sobre
eritematoso diseminado Θ La lectura se basa en la observación de bandas
Θ Personas con anticuerpos frente a diversos antígenos
coloreadas que corresponden a las distintas proteí-
HLA
nas víricas
ENAM 03-A (81) ; ENAM 07 (16) : Varón de 24
Detecta anticuerpos frente a las
años de edad, soltero, 2 meses antes presenta
glicoproteínas de envoltura :
diarrea acuosa de 3 a 6 veces por día, sin moco,
sin sangre, acompañadas de dolor abdominal difuso √ gp160, gp120 y gp41
y pérdida de peso. Hace 2 días se agrega náuseas,
√ las codificadas por el gen gag p55,
vómitos y cefalea global. Al examen: adelgazado,
p24 y p17
pálido, deshidratado, PA 90160 mmHg, FC 110 por
√ las proteínas enzimáticas p66, p51 y
minuto, FR 26 por minuto, mucosa oral seca,
p311.
candidiasis oral. Abdomen: excavado, blando, dolor
difuso. El examen de laboratorio más importante a RM 11-B (77): ¿ Cuál es la prueba de
solicitar es: confirmación para VIH – SIDA?
A. Dosaje de CD4 y CD8 a. Wester blot
B. BK en heces seriado b. PCR
C. Examen de parásitos en heces c. ELISA

138
Manual de INFECTOLOGÍA PLUS MEDIC A
Categorías de Categorías Clínicas
Laboratorio
Linfocitos CD4 (A) (B) (C)
Asintomáticos Sintomáticos Condiciones
Infección Aguda sin condiciones Indicadoras
LGP AoC de SIDA

(1) >= 500 cel/mm3 A1 B1 C1

(2) 499-200 cel/mm3 A2 B2 C2

(3) < 200 cel/mm3 A3 B3 C3

Sistema de clasificación del CDC para Infección por VIH en


adultos y adolescentes mayores de 13a. (1993)

d. Inmunofluorescencia
e.Hemaglutinación
Θ Un recuento de CD4 por debajo de 200/μL
Rpta. A
se considera que definen el SIDA

Estadiaje Debido al aumento del riesgo de infeccio-


nes oportunistas en este nivel.
Estadificación de la enfermedad del VIH
Cultivo viral
Se basa en parte en la presentación clínica, pero
otras pruebas de laboratorio pueden ayudar a decidir
Es costoso y requiere mucho tiempo y es menos
si iniciar o modificar el tratamiento. sensible en pacientes con cargas virales bajas.

√ El recuento de células T CD4 es un indicador El cultivo viral se puede realizar como parte de
confíable del riesgo para adquirir infecciones
las pruebas fenotípicas ante la resistencia a los
oportunistas. medicamentos.

√ El rango de referencia para el recuento de CD4 es


Biopsia de ganglios linfáticos
500 a 2000 células / mL. VIH ADN, el ARN y las proteínas se pueden
Después de la seroconversión, los recuentos detectar con técnicas moleculares y de
de CD4 tienden a disminuir (en torno a 700/μL microscopía electrónica puede revelar viriones.
en promedio) y continúan disminuyendo con
El CDC clasifica la infección por VIH según
el tiempo.
recuento de CD4 de células T .

Además de la presencia de ciertas infecciones o


enfermedades.

139
Manual de INFECTOLOGÍA PLUS MEDIC A

CATEGORÍA C
Es la infección por el VIH con el SIDA

Enfermedades oportunistas según el CD4

Estas tres categorías se subdividen basado en


el recuento de células T CD4:
CATEGORÍA A
Es la infección asintomática por VIH sin - Categorías A1, B1 y C1
antecedentes, síntomas o condiciones que
SE CARACTERIZAN POR CÉLULAS T
definen SIDA. CD4 + MÁS DE 500/ΜL.

CATEGORÍA B - Categorías A2, B2 y C2


Es la infección con síntomas que son SE CARACTERIZAN POR RECUENTO DE
directamente atribuibles a la infección por CÉLULAS T CD4 + ENTRE 200/ml y 400/ml.
el VIH o que se complican .

√ Angiomatosis bacilar
- A3, B3, o C3.
√ Candidiasis orofaringea (aftas)
- INFECCIONES POR EL VIH CON
√ La candidiasis vulvovaginal, persistente o RECUENTOS DE CD4 + T-CELL EN
resistente 200/ΜL SE DESIGNAN COMO
√ Enfermedad inflamatoria pélvica (EIP)
√ Displasia cervical (moderada o grave) o carci Agentes oportunistas
noma cervical in situ √ Las enfermedades oportunistas que se asocian
Leucoplasia vellosa oral a la infección por VIH están en relación al
√ Púrpura trombocitopénica idiopática nivel de CD4
√ Los síntomas constitucionales, como fiebre (>
38,5 ° C) o diarrea que dura más de 1 mes RM 2014 I – A (19) : ¿Cuál es la Infección
√ Neuropatía periférica oportunista en SIDA que cursa con CD4 mayor
√ El herpes zoster (culebrilla), con la participa - de 300 /uL al momento del diagnóstico?
ción de 2 o más episodios o dermatomas.
A.Neumonía por pneumocystis
B.Retinitis por cltomegalovirus
C.Toxoplasmosis

140
Manual de INFECTOLOGÍA PLUS MEDIC A

Lugar de acción de los antiretrovirales

B.D.Tuberculosis ENAM 03-A (42) : La mayoría de las


E.Criptococosis infecciones oportunistas en pacientes con
VIH se producen cuando las cuenta de CD4/mm3
Rpta D
es inicialmente menor de:
A. 50.
Tratamiento B. 100
C. 200
TARGA D. 300
E. 400
Terapia Rpta. C
Anti
Retroviral ENAM 04-B (57) : En relación con la infección
Gran por el virus de VHI, se considera pareja
Actividad discordante cuanto uno de ellos :
A. Es VIH seronegativo
Es el régimen utilizado en los pacientes con SIDA, B. No tiene tratamiento anti – retroviral
con el que se espera se logre reducir la carga viral a C. Tiene baja cantidad de linfocitos CD4
niveles indetectables por el mayor tiempo posible. D. Tiene tiempo de enfermedad mayor
E. Tiene carga virar diferente
Rpta. A

Este régimen consta de por lo menos 3


drogas antiretrovirales (terapia triple)

141
Manual de INFECTOLOGÍA PLUS MEDIC A

Actualmente TARGA es TARV (Terapia Inhibidores de la transcriptasa


AntiRetroViral)
inversa análogos nucleósidos
Recomendación según recuento CD4: (ITIAN)

√Todo persona VIH (+) con CD4 < 500 cels/uL, Θ Zidovudina (AZT)
independientemente de la presencia de sínto - Θ Lamivudina (3TC)
mas Θ Estavudina (d4T),
Θ Didanosido (ddI) Θ Emtricitabina
Recomendación sin considerar recuento CD4:
√ Síntomas de inmunosupresión Acctúan de dos maneras:
√ Embarazo  Inhibiendo competitivamente la incorpo-
√ Pareja serodiscordante ración del nucleótido
√ Nefropatía asociada al VIH (HIVAN)
√ Coinfección VHB (crónica)  Finalizando la formación de ADN viral
√ Coinfección VHC (crónica)
¿Porqué a estos fármacos se le llaman
√ Infección aguda por VIH “análogos de los nucleósidos”?
√Deterioro neurocognitivo asociado al VIH
Porque los ITIAN presentan similitud estructural con los
TARV debe iniciarse en todo adulto 2’- desoxinucleósidos naturales, Compiten tanto con las
infectado por VIH independiente del bases :
recuento de CD4
√ Purínicas (adenosina, guanosina, inosina)
Antiretrovirales √ Pirimidínicas (citosina y timidina).

◆Inhibidores de la transcriptasa ¿Son fármacos o profármacos?


inversa análogos nucleósidos Son profármacos que requieren
(ITIAN): activación intracelular previa
√ Zidovudina, estavudina, lamivudina, Θ Se fosforilan en el interior del citoplasma
didanosina, zalcitabina, abacavir por la acción de enzimas celulares, convirtiéndose en
◆Inhibidores de la transcriptasa nucleótidos (forma trifosfato).
inversa análogos nucleótidos Θ Debido a la similitud con los nucleótidos natu-
(ITIANt): rales, compiten en la incorporación a la cadena de
ADN provírico naciente, proceso mediado por la trans-
√ Tenofovir
criptasa inversa.
◆Inhibidores de la transcritasa
Θ Bloquean la replicación viral en fases preinte -
inversa no análogos nucleósidos grativas, cuando la TI del VIH sintetiza ADN com-
(ITINN): plementario (ADNc) a partir del ARN viral para inte -
√ Efavirenz ,Nevirapina. grarse posteriormente en el genoma de la célula.
◆Inhibidores de la proteasa (IP): Θ Al integrar un análogo de nucleósido
√ Indinavir, saquinavir, nelfinavir, trifosforilado se bloquea el proceso de elongación de
ritonavir, amprenavir, lopinavir/ritonavir la hebra del ADNc, porque no disponen del grupo oxi -
drilo en posición 3’ para la formación de puentes fos -
fodiéster y por lo tanto, actúan como terminadores de ca -
dena.

142
Manual de INFECTOLOGÍA PLUS MEDIC A

Características La combinación de ZDV con estavudina está


contraindicada, porque comparten la misma fosfoqui-
√ Se pueden tomar antes o después de los alimentos nasa celular necesaria para la activación (interacción
(excepto el ddI, que debe ser tomado en ayunas) farmacodinámica).
√ En general, no interactúan con otras drogas
√ Todos los INTR pueden producir una condición clíni- Uso en determinadas situaciones:
ca rara pero fatal: acidosis láctica y esteatosis Θ Trombocitopenia: ZDV recuperación rápida de
hepática la cifra de plaquetas en pacientes con trombopenia.
Θ Complejo demencia- SIDA: El ZDV tiene bue-
ZIDOVUDINA (AZT) na difusión a través de la barrera hematoencefálica, por
lo que se usa en pacientes con afectación neurológica
Θ 1964: fue sistetizada en
para el tratamiento de
compatible con encefalitis por VIH.
tumores. • Prevención de la transmisión vertical y en
la profilaxis post-exposición accidental
Θ 1987 (“Luz verde”) :
Fue aprobado por la FDA Los efectos adversos mas comunes
para el tratamiento de la incluyen:
infección por VIH.
anemia, neutropenia, náuseas, vómitos,
Θ Estructura química : 3’-azido-2’3’-didesoxi- cefalea, fatiga, confusión, malestar, miopatia y
timidina, análogo de timidina. hepatitis.

√ Sufre trifosforilación en el citoplasma por la timi - EsSalud 09 (44): La zidovudina es un fárma -


dinquinasa celular √ co antirretroviral caracterizado por:
La forma ZDV trifosfato actúa como inhibidor com- a.Aumentar sus efectos tóxicos a mayor número de
petitivo de timidina trifosfato. linfocitos CD4
√ Su actividad antivírica es fundamentalmente a b.Incrementar el ARN plasmático del VIH
nivel de los linfocitos T activados, y en menor c.Incrementar su concentración cuando se asocia
grado, en los monocitos y macrófagos infectados. con rifampicina
d.Ser eficaz en pacientes con linfocitos CD4 < 500
Θ Biodisponibilidad oral : 65-70%
células/mm3
Θ Vida media plasmática : 1 hora e.Tener gran actividad en macrófago – monocitos
Vida media intracelular : 3h humanos
Ello permite una frecuencia de administración de 2 ó 3 Rpta D
veces al día. Θ
Dosis : 250- 300mg cada 12 horas.
LAMIVUDINA (3TC)
Θ Atraviesa bien la placenta y también la barrera he - Θ Estructura química : 2’3’-dideoxi-3’-tiacitidina,
matoencefálica, alcanzándose niveles terapéuticos en el análogo de la citosina
líquido cefalorraquídeo (concentración 55% de la Θ Mecanismo de acción : similar al grupo:
plasmática). √ Difunde pasivamente y necesita ser trifosforilado
Θ Metabolismo hepático por glucuronidación en un para ser activo.
75%; el 25% restante se elimina por el riñón, necesitan- √ La inhibición se produce en linfocitos y en célu
do ajuste de dosis en insuficiencia renal (IR). las del sistema mononuclearfagocítico en fase laten-
te.

143
Manual de INFECTOLOGÍA PLUS MEDIC A

Θ Biodisponibilidad oral : 90%


Θ Vida media plasmática : 1.5 hora
También posee
Vida media intracelular : 3.5h
un potente
Θ Dosis : 40 mg cada 12 horas en pacientes con un
efecto inhibidor
frente al virus
peso superior a 60 kg y 30mg en el resto.
de la hepatitis B. Θ Atraviesa bien la placenta y también la barrera he -
matoencefálica, alcanzándose niveles terapéuticos en el
líquido cefalorraquídeo

Θ Biodisponibilidad oral : 86% Θ El 60% del fármaco se elimina por vía hepática
(mismo metabolismo que nucleótidos naturales) y el
Θ Vida media plasmática : 2,5 horas
40% sin modificar por vía renal, necesitando ajuste
Vida media intracelular : 12h
Θ Dosis: 150 mg cada 12 horas y de 300mg cada 24 efectos adversos
h.
Θ Difunde escasamente a través de la barrera hemato neuropatía periférica, lipodistrofia
y acidosis láctica.
encefálica y se elimina principalmente por la orina
(5-10% por metabolismo hepático), siendo necesaria la
DIDANOSINA (ddI)
reducción de dosis en caso de insuficiencia renal. Θ Estructura química : 2’3’- didesoxiinosina,
análogo nucleósido de la purina: inosina.
Tiene sinergismo con AZT, d4T, IP y no
nucleósidos.

Los efectos adversos


Fue el segundo
Más frecuentes: antirretroviral
Náuseas . aprobado por
efectos graves : acidosis láctica, la FDA
pancreatitis , hepatotoxicidad

ESTAVUDINA (d4T)
Θ Estructura química : 2’3’-didehidro-3’deoxiti- √ Presenta un mecanismo de acción más
midina, análogo de la timidina. complejo que los nucleósidos pirimidínicos
√ Sufre trifosforilación en la célula, la forma activa (AZT, ddC, d4T).
compite con la timidina- trifosfato y bloquea la elon- √ Penetra en la célula por difusión pasiva, ini
gación del ADN. cialmente se transforma en ddImonofosfato, mediante
√ Inhibe la replicación viral fundamentalmente en la 5’-nucleotidasa
células activadas, tanto en linfocitos como en macrófagos. √ Después se convierte en didesoxiadenosinmono -
Posee una estructura similar al AZT y fosfato, por las enzimas adenilsuccinato -sintetasa/
comparte la misma quinasa celular liasa.
√ La forma activa tiene una semivida intracelu -
lar superior a 12 horas y actúa inhibiendo la TI por
inhibición competitiva actuando como termi -
nador de cadena, bloqueando la elongación.

144
Manual de INFECTOLOGÍA PLUS MEDIC A

√ Es activo en todas las fases del ciclo Inhibidores de la transcriptasa


celular, por estar siempre presentes las
inversa análogos NO nucleósidos
enzimas citoplasmáticas necesarias para su
activación. (ITIANN)

Θ Biodisponibilidad oral : 90% Θ Nevirapina(NVP ,Viramune))


Θ Vida media plasmática : <1 hora Θ Efavirenz (EFV, Estocrin, Sustiva)
Vida media intracelular : 25-40 h Θ Delavirdina (Rescriptor),

Θ Dosis : 400 mg al día en pacientes con un peso Θ Mecanismo : Actúan inhibiendo la enzima
superior a 60 kg y 250mg/d si el peso es menor. transcriptasa inversa.
Θ La escasa unión a proteínas plasmáticas (5%) hace √ No necesitan ser fosforilados para su activación y
que el ddI se distribuya adecuadamente en los no se incorporan a la cadena de ADN en formación,
tejidos y pasa pobremente a través de la barrera he - sino que actúan por unión no competitiva en
matoencefálica y placentaria un lugar cercano al centro catalítico de la
Θ Eliminación : renal y hepática (por la vía meta - enzima.
bólica de las purinas, degradándose en hipoxantina y √ Son estructuralmente distintos entre sí.
ácido úrico), de manera que requiere ajuste de dosis en √ Como ventaja presentan un índice terapéutico
IR e IH. amplio, alcanzándose niveles plasmáticos muy superio -
res a la concentración inhibitoria del virus, pero por debajo
Se ha utilizado en asociación con hidroxiurea de niveles tóxicos.
porque mejora su perfil farmacodinámico,
debido a su efecto inhibidor sobre la La rápida aparición de resistencia es
producción de dATP (el nucleótido una de sus limitaciones.
competidor natural del ddI).
Nevirapina (NVP )
Θ Mecanismo : Es derivado de la dipiridodiazepi-
nona.
√ Inhibe la TI del VIH-1, pero no actúa sobre el
VIH-2.
√ Se une a la enzima en los residuos de tirosina en
las posiciones 181 y 188 de la subunidad p66 de la
transcriptasa, alterando el sitio catalítico situado en la
proximidad e impidiendo la actividad ARN/ ADN -
polimerasa ARN- dependiente, enlentece la reacción
Los efectos adversos : catalítica necesaria para la función de la enzima.
Esto explica la acción sinérgica con los
Más frecuentes son diarrea, dolor abdomi- análogos de nucleósidos.
nal, cefalea, náuseas, y vómitos
√ Presenta una alta biodisponibilidad oral (>
Menos frecuentes: 90%), que no se ve alterada por comidas.
Pancreatitis aguda , neuropatia √ Una vida media de 24 horas
periférica , despigmentación √ Dosis: 200 mg cada 12 horas.
retiniana ,acidosis láctica √ Su carácter lipofílico favorece una amplia distribu-
ción.

145
Manual de INFECTOLOGÍA PLUS MEDIC A

Atravieza la barrera placentaria y hematoencefálica efectos adversos


(45% de los niveles plasmáticos).
Ha demostrado eficacia en la transmisión Mareos, cefalea, insomnio,
materno- fetal. pesadillas, dificultad para
concentrarse , rash
√ Metabolismo : hepático, a través del citocromo P450.
Sufre un proceso de autoinducción metabólica, proceso ENAM 2013-A (57): Antiretroviral que está
que se estabiliza en dos semanas contraindicado en gestantes:
efectos adversos a. Nevirapina
exantema y la hipertransaminasemia b. Epivir
generalmente leves c. Efavirenz
d. Zidovudina
La incidencia de exantema con la dosis de 400 mg e. Lamivudina/Zidovudina
es cercana al 50 %, y alrededor de 10-30 % con la dosis Rpta A
inicial de 200 mg/día durante los primeros 14 días, por ello
se recomienda escalonar la dosis

DELAVIRDINA
EFAVIRENZ Aprobado por la FDA para el tratamiento
Previamente conocido como DMP-266. del VIH
√ Inhibe de forma no competitiva la trans - √ Biodisponibilidad : 85% por vía oral, presenta
criptasa inversa al unirse al bolsillo cercano al sitio una vida media de 6 horas y se une mucho a las pro -
catalítico de la enzima. teínas plasmáticas (98 %), con lo que los niveles alcanza-
una comida rica en grasas aumenta su dos en LCR son muy bajos (0,8 %).
biodisponibilidad. Se metaboliza por el citocromo P450, es
inhibidor del isoenzima CYP3A4.
√ Alcanza su concentración máxima a las 5 ho-
√ Dosis: 600 mg cada 12 horas
ras de la toma.
√ Se une a proteínas plasmáticas en un porcentaje
alto (>99%), limitando su distribución a distintos com-
partimentos (en LCR se alcanzan niveles del 1%).
√ Es metabolizado por el sistema citocromo
P450 (isoenzimas CYP3A4 y CYP2B6) y es inductor en-
zimático.
√ Los metabolitos se eliminan por orina y heces. Inhibidores de la transcriptasa
inversa análogos nucleótidos (ITIANt)

Θ Tenorfovir

TENOFOVIR
146
Manual de INFECTOLOGÍA PLUS MEDIC A

Se administra por vía oral como profármaco √ Para que los viriones maduren es necesario
en forma de “disoproxil fumarato (DF)” que la proteasa del VIH divida la poliproteí -
√ Con alimentos, para mejorar su biodisponibilidad na en proteínas más pequeñas yfuncionales, son proteí-
(biodisponibilidad de 25% en ayunas y 40% con alimentos). nas estructurales y enzimas virales.
√Las estearasas séricas y tisulares lo con-
vierten en tenofovir que es un análogo nu- La inhibición de la proteasa lleva consigo la
cleotídico de la adenosina monofosfato. formación de partículas virales desorga-
√Necesita ser bifosforilado dentro de la célula nizadas estructural y funcionalmente, y por
para ser activo y tras la activación bloquea la actividad tanto, sin capacidad infectante.
de TI y la elongación del ADNc viral, igual que los
√Los inhibidores de la proteasa se unen reversiblemen-
ITIAN. te al lugar activo de la enzima y la inhiben de for -
√ Vida media intracelular : es mayor de 30 horas ma competitiva, impidiendo la escisión de los precur
√ Dosis: 245 mg de tenofovir (igual a 300 mg de sores proteicos gag y gagpol
fumarato), con alimentos. √ Abortan la maduración y bloquean la infec -
tividad de los nuevos viriones generados
√ Tenofovir es eliminado principalmente por el riñón (70-
80%) de forma inalterada. impiden oleadas sucesivas de infección Son
activos frente a VIH-1 y VIH-2.
efectos adversos
√ No tienen efecto sobre células con ADN pro -
GI: (diarrea, náuseas, vómitos) e viral integrado.
hipofosfatemia.
Tienen un amplio índice terapéutico.

√La biodisponibilidad oral es escasa y varia en


función del metabolismo de primer paso hepático
(saquinavir el más susceptible); y de los alimentos
comida con elevado contenido graso
aumenta la biodisponibilidad
√ Excepto con indinavir que la disminuye y ha de
tomarse con el estómago vacío o con una comida ligera).
Inhibidores de la Proteasa (IP) √ La unión a proteínas plasmáticas es alta
(98%), excepto el indinavir (60%), y dificulta que se
Θ Indinavir, saquinavir, nelfinavir, alcancen altos niveles intracelulares.
ritonavir, amprenavir, lopinavir/ritonavir
Todos los IP son inhibidores enzimáticos,
Θ Estructura : La proteasa es un homodímero si - siendo el ritonavir el más potente, seguido
métrico, formado por dos cadenas idénticas de indinavir, nelfinavir y saquinavir.
de 99 aminoácidos, esencial en la etapa de madura -
efectos adversos
ción viral.
Cuando el ADN viral se transcribe se Intolerancia gastrointestinal, la hiperli -
forman unas poliproteínas no funcionales pidemia, resistencia a la insulina y
(gap y gag-pol) alteraciones en la redistribución de grasa
corporal o lipodistrofia.

147
Manual de INFECTOLOGÍA PLUS MEDIC A

INDINAVIR Norma Tecnica de Salud de Atención Integral


del adulto con Infección por VIH. 11
Tienen mayor selectividad por VIH-1 que Diciembre 2014
VIH-2.
RM 2014 I-A (23): ¿Cuál es la caracteristlca
√ La biodisponibilidad oral es superior al 30 %
los alimentos disminuyen su absorción. que corresponde al sarcomá de Kaposi?:
√ Las desventajas son vida media corta (3 veces al día)
A.Las lesiones son dolorosas y pruriginosas
y sus restricciones dietéticas (ayunas o con comida ligera) y
B.Es una neoplasia maligna
la necesidad de beber abundantes líquidos para evitar la
nefrolitiasis.
C. No invaden la cavidad oral
√ Dosis: 800 mg cada 8 horas con el estómago vacío D.Es infrecuente en el tracto digestivo
El indinavir disminuye el pH gástrico, lo que E.La etiología es el VIH
puede alterar la absorción del ddI (separar Rpta B
los fármacos 1 hora).
Sarcoma de Kaposi
Esquemas de Tratamiento
Es un tumor mesenquimatoso con involucro
De elección de los vasos sanguíneos y linfáticos, originado
por el herpesvirus humano 8, también conocido como
ΘTenofovir/Emtricitabina/Efavirenz en DFC sarcoma de Kaposi asociado a herpesvirus.
Alternativas

ΘTenofovir/Emtricitabina + Efavirenz

ΘTenofovir + Lamivudina + Efavirenz

ΘAbacavir + Lamivudina + Efavirenz

ΘZidovudina/ Lamivudina+ Efavirenz

148
Manual de INFECTOLOGÍA PLUS MEDIC A

√Es hasta 20 veces más frecuente en varones ho - RM 2010-A (55): Tumor cerebral más frecuente.
mosexuales con VIH. a.Linfoma
√Tiene predilección por la región cefálica (párpa - b. Meningioma
dos, nariz y orejas).
c. Astrocitoma
√La afectación de mucosas es frecuente, d. Meduloblastoma
sobre todo a nivel oral, y puede ser la manifestación
e. Glioblastoma ente en el VIH
inicial de la enfermedad en un 15% de estos casos.
Rpta A
Estos pacientes pueden presentar afectación extra-
cutáneas: gastrointestinal, pulmonar y ganglionar.
Linfoma cerebral primario (PCNSL, Primary
RM 2014 I – A (19) : ¿Cuál es la Infección CNS Lymphoma)
oportunista en SIDA que cursa con CD4 mayor de ΘEs un linfoma no Hodgkin agresivo que se origina
300 /uL al momento del diagnóstico? en el sistema nervioso central sin evidencia de linfoma en
otra localización en el momento del diagnóstico.
A.Neumonía por pneumocystis
B.Retinitis por cltomegalovirus C.Toxoplasmosis ΘEl pico de incidencia se sitúa en la sexta década
D.Tuberculosis de la vida, siendo de aparición más precoz en
pacientes inmunodeprimidos (primera década en in -
E.Criptococosis
Rpta D munodeficiencias congénitas y cuarta década en pa -
cientes transplantados y con SIDA).
ENAM 2013-B (42): Trabajadora sexual VIH
Θ La forma clínica de debut más frecuente es
positivo, actualmente asintomática . ¿Cuál es la la focalidad neurológica (50-80%).
recomendación más adecuada? Otras formas de presentación clínica pueden ser:
a.Profilaxis para TBC
√ Síntomas neuropsiquiátricos (20-30%)
b. Profilaxis para infección por Pneumocystis
√ Hipertensión intracraneal (10-20%) y crisis
epilépticas (5-20%).
Jirovecci
c. Profilaxis para criptococosis Essalud 04 (5): ¿Cuál de las siguientes
d. Profilaxis para toxoplasmosis complicaciones neurológicas del SIDA es
e. Profilaxis para Candidiasis espontáneamente reversible?
Rpta A a) Demencia
b) Leucoencefalopatía multifocal progresiva
ENAM 2013-B (58) : Paciente de se pinchó con
c)Sindrome de Guillaim barré
aguja utilizada en paciente VIH positivo. ¿Cuál es d) Mielopalia vacuolar
la conducta inmediata? e) Toxoplasmosis cerebral
a. Antibióticos Rpta. C
b. Corticoides
c. Lamivudina
d. Zidovudina
e. Lavado de manos con agua y jabón
yodopovidona
Rpte E

149
Manual de INFECTOLOGÍA PLUS MEDIC A

CRIPTOCOCOSIS
Diagnóstico

OF: Paciente con diagnóstico de VIH cual es el


agente oportunista más frecuente que produce
meningoencefalitis:
Tratamiento
a. Aspergillus
ENAM 2014-A (24): Mujer de 45 años, con
b. Criptococo
esquizofrenia paranoide y HIV positivo, acude al
c. Micobacterium
hospital por convulsiones y trastorno del
d. Candidiasis
sensorio. Al examen: Glasgow 9, signos
e.Toxoplasma
meníngeos (+). LCR: levaduras, positivo a tinta
china. ¿Cuál es el tratamiento a seguir?
a.Fluconazol
b.itraconazol
c.Anfotericina B
d.Ketoconazol
e.Caspofungina

Rpta B y C

150
Manual de INFECTOLOGÍA PLUS MEDIC A

TOXOPLASMOSIS
Tratamiento
EsSalud (89) : Paciente con VIH, acude por
sensación de adormecimiento, así como también
signos de focalización. Usted sospechando en el
agente causal pide una TEM cerebral que muestra
una imagen con anillo captante. Ante esta
situación, usted daría el siguiente tratamiento :
a. Zidovudina
b. Sulfadiazina + Pirimetamina
c. Anfotericina B
d. Ceftriaxona
e. Aciclovi

151
Manual de INFECTOLOGÍA PLUS MEDIC A

152
Manual de INFECTOLOGÍA PLUS MEDIC A

PNEUMOCISTOSIS
Tratamiento ENAM 08-A (47) : Varón de 32 años de
edad, con 60 kg de peso y diagnóstico de
OF: ¿Cuál es el tratamiento de elección para la SIDA y neumonía por Pneumocystis jirovecci.
neumonía por Pneumocystis jirovecci? ¿ Cuál es el tratamiento antimicrobiano más
adecuado?:
a.Ciprofloxacino A. Cotrimoxazol
b.Amoxicilina/ácido clavulánico B. Clindamicina
C. Tetraciclina
c.Cotrimoxazol
D. Ciprofloxacina
d.Vancomicina E. Imipenem
e.Azitromicina Rpta. A

153
Manual de INFECTOLOGÍA PLUS MEDIC A

MALARIA PLUS MEDIC A

Historia Por lo visto el Perú no sólo ha hecho su


contribución al mundo en la gastronomía
Se considera que la malaria es una sino también en Medicina
enfermedad prehistórica.

Epidemiología
Se estima que esta enfermedad probablemente
cause mayor morbilidad y mortalidad que ninguna
otra en el mundo .

Probablemente estos parásitos y en especial Se calcula que se presentan


Plasmodium falciparum, llegaron durante la probablemente 110 millones de
época colonial a través de los esclavos africanos casos de malaria al año y al menos
traídos a América. un millón de personas mueren
anualmente por esta enfermedad,
principalmente niños.

Desde la ejecución del Programa Nacional de


Erradicación de la Malaria, el 99% de los casos de
malaria han sido debidos a Plasmodium vivax , el
cual generalmente produce un cuadro benigno.

Estos 2 personajes son parecidos pero no


iguales. Lo mismo sucede con malaria y
fiebre tifoidea.

La medicina peruana prehispánica hizo su


contribución a la terapéutica, a inicios del siglo
XVII con el descubrimiento de la “corteza peruana”
(quina), cuyo uso se diseminó rápidamente en
Europa.

¿Porqué el parecido con la fiebre tifoidea?


154
Manual de INFECTOLOGÍA PLUS MEDIC A

Porque en el primer ataque por cualquiera de las 1. FASE ESPOROGÓNICA


especies presentan fiebre diaria, no encontrándose (En el mosquito, es sexual y dura 2 semanas)
la terciana o cuartana características y
generalmente se encuentra un recuento leucocitario El mosquito Anópheles hembra se infecta al ingerir
normal. o leucopenia, con desviación a la izquierda. sangre con macro y microgametocitos En el estómago
Con frecuencia se presentan brotes aislados en
de la misma, forma el CIGOTO, que sufre proceso de
Huachipa (río Rímac), Campoy (río Rímac), Pro (río
mitosis dando lugar a OOCISTOS.
Chillón) y otras áreas aledañas
Los oocistos crecen por división nuclear y se
Terciana maligna liberan miles de ESPOROZOITOS móviles que
Es motivo de alarma la reaparición de la migran a la cavidad digestiva y de allí a las
llamada ”terciana maligna”, debida a Plasmodium glándulas salivares convirtiendo a la
falciparum, en nuestro territorio en los últimos años. hembra del mosquito anópheles en vector
El problema es aún mayor debido a los casos de infectante .
resistencia a la cloroquina y a la combinación
sulfadoxina/pirimetamina, lo cual dificulta el
tratamiento y aumenta el riesgo de complicaciones
severas.

Ciclo biológico
Cuatro son las especies de Plasmodio causantes de
Malaria en el ser humano:

ΘFALCIPARUM
ΘVIVAX
ΘOVALE
ΘMALARIAE

La forma Vivax es la más frecuente en


nuestro país.
El ciclo del parásito incluye:
El vector es el mosquito anópheles.
Sólo la hembra es la que pica a las personas en
busca de sangre que es necesaria para su
reproducción

Los anópheles machos son “vegetarianos”. 2. FASE ESQUIZOGÓNICA exoeritrocítica


(En el hígado de la persona, es asexual )
Cuando el anópheles pica a una persona infectada
e ingiere sangre , está ingiriendo también al Cuando el anópheles pica a una persona sana
plasmodio en forma de gametocitos ( macro y
inocula ESPOROZOITOS que ingresan al
micro gametocitos).
torrente sanguíneo .

Muchos son destruidos por el sistema inmune del


individuo y otros ingresan a las células hepáticas.

155
Manual de INFECTOLOGÍA PLUS MEDIC A

Dentro de los hepatocitos se replican adquiriendo 3. FASE ERITROCÍTICA ESQUIZOGÓNI-


una nueva forma llamada MEROZOITO. CA
(En los GR de la personas infectadas)

La liberación de los merozoitos hepáticos al


torrente circulatorio se produce en un lapso de
tiempo entre 6 a 16 días luego de la primoin-
fección.

. Algunos plasmodios
deciden quedarse dur-
miendo en el hígado ,
son los HIPNOZOITOS
(Vivax y Oval)

Plasmodio Falciparum y Malariae tienen


¡Ovvv que sueño! Sólo el plasmodio Vivax solo una forma exoeritrocítica en contraste con
y Oval se convierten en hipnozoitos. Plasmodio Vivax y Ovale que tienen dos
formas una igual a la de los merozoitos que
pasan al torrente sanguíneo y la segunda forma
quiescente en las células hepáticas.

La fase eritrocítica comienza con la invasión


de los glóbulos rojos por los MEROZOITOS,
que cambian de forma y estructura molecular con
diferentes proteínas de superficie y que adquieren
el nombre de TROFOZOITOS.

Los trofozoitos se multiplican dentro del glóbulo


Es de destacar que esta multiplicación en las rojo dando lugar a 8 a 24 trofozoitos en cada
células hepáticas no determina reacción eritrocito afectado.
inflamatoria del parénquima.
Los trofozoitos se convierten en
ESQUIZONTES

156
Manual de INFECTOLOGÍA PLUS MEDIC A

Dentro del cual se forman nuevos


√Terciana benigna
MEROZOITOS
( paroxismo febril c/ 48h
Al completarse este proceso se produce : por Vivax)
la ruptura del glóbulo rojo, hemólisis, liberando √Terciana maligna
MEROZOITOS MADUROS y ( paroxismo febril c/ 48h
GAMETOCITOS. por Falciparum)
√ Cuartana
En este momento aparecen los síntomas
característicos de la Malaria. ( paroxismo febril c/ 72h
por Malariae)
Estos merozoitos invaden otros glóbulos rojos y
se desarrolla otra generación de parásitos; este
proocurre repetidamente y se llama ESQUI - Algunos merozoitos se diferencian en formas
ZOGONIA ERITROCITICA. sexuadas:
El tiempo en que ocurre este ciclo, depende de ◎MACROGAMETOCITOS
cada tipo de plasmodio, 48 horas para femeninos
Plasmodio Falciparum Vivax y Ovale y 72 horas ◎MICROGAMETOCITOS
para el Malariae . masculinos.
Los gametocitos son ingeridos por el anopheles
Estos ciclos de 48 horas clínicamente se expresan hembra al picar a una persona infectada, y se torna
en picos febriles (paroxismos febriles) al infectante , cerrándose el ciclo vital.
ocurrir la parasitemia por ruptura eritrocítica .

157
Manual de INFECTOLOGÍA PLUS MEDIC A

OF: Cada una de las siguientes afirmaciones es Además presenta:


correcta con respecto a la Malaria, marque lo √Anemia.
falso: √Mononucleosis.- Alto número de leucocitos
a. La hembra del mosquito Anopheles es el vector mononuclerares.
b. Al comienzo de la infección, los esporozoítos
√Hepatoesplenomegalia por liberación de las
ingresan a los hepatocitos
c. La salida de los merozoitos de los eritrocitos es toxinas.
la causa del paroxismo malárico √eutrofilia; por elevado número de leucocitos
d. Los gametocitos se forman principalmente en el neutrófilos en sangre.
tracto gastrointestinal
e. Ninguna de las anteriores A estos síntomas en Plasmodium falciparum hay
Rpta D que añadir:
√Trastornos del aparato circulatorio. (hipotensión
arterial)
Cuadro clínico √Complicaciones en el sistema nervioso central:
sopor
√Alteraciones intestinales: vómitos, diarrea.
1º Periodo de incubación
Corresponde a las esquizogonias hepáticas o
exoeritrocíticas y no presenta síntomas clínicos. Criterio epidemiológico
2º Periodo agudo Procedencia de la selva.
Se da un primer acceso febril, que aparece La muerte suele sobrevenir por disfunción
cuando existen 2000 parásitos por mm cúbico de cardiaca y por la anemia progresiva. Algunos
sangre. individuos gracias a la puesta en marcha a tiempo de su
sistema inmunológico, pueden llegar a detener la
Además, se tiene cefalea, náuseas y en el fase aguda, bajando la frecuencia de los accesos febriles
examen físico se encuentra hepatoespleno- hasta hacerlos desaparecer. El individuo entonces parece
megalia. curado.
Corresponde a las esquizogonias hepá- Son muy típicas del paludismo las
ticas o exoeritrocíticas y no presenta recaídas
síntomas clínicos
El individuo presenta fiebre durante 8-10 días, y OF : En un paciente con MALARIA ¿Cuál de los
luego ésta baja, apareciendo una fuerte sudoración. siguientes signos es el más frecuente?
Cuando desaparece la fiebre el enfermo se A.- Eritema palmar.
recupera rápidamente, pero vuelven a aparecer los B.- Rubicundez.
síntomas a las 48 o 72 horas según la especie. C.- Cianosis.
D.- Palidez.
En el caso de falciparum, la fiebre no E.- Conjuntivas rosadas.
suele comenzar con un escalofrío inicial. Rpta. D

Esquizogonias OF : La denominada terciana benigna, con periodo


En este género son menos sincrónicas y por esto los de incubación de 14 días transmisibilidad entre uno
paroxismos febriles son menos a tres años y que se acompaña de fiebre irregular,
regulares y de más larga duración escalofríos, diaforesis, cefalea, astenia, ictericia,
es producida por el PLASMODIUM :
A.- Falciparum.

158
Manual de INFECTOLOGÍA PLUS MEDIC A

B.- Malarie. En el resto de las especies, el enfermo crónico presenta


C.- Ovale. la caquexia palúdica: donde el individuo tiene una
D.- Vivax anemia progresiva, infarto del bazo y del hígado, y
E.- Tropicalis. la piel se le colorea de amarillo-gris-pálido.
Rpta. B

Fase crónica. Diagnóstico


(Recaídas) El diagnóstico clínico es muy inexacto . tiene
Para vivax y ovale se ha detectado una fase del una especificidad (20 a 60%) en comparación
ciclo nueva llamada hipnozoito. con la microscopía.
Es un zoito que por motivos desconocidos entra en una
fase de latencia en el hígado en la esquizogonia
hepática
Se reactiva este hipnozoito y da nuevas
esquizogonias hepáticas llamadas esquizogonias
eritrocíticas secundarias

P. Vivax y P. Ovale presentan esquizogonias


eritrocíticas secundarias, mientras que P.
Falciparum y P. Malariae, no presentan esqui-
zogonia eritrocítica secundaria ni hipnozoitos

Las recaídas son de dos tipos:

a) Recaída verdadera o recidiva.

Se da en vivax y ovale. Son debidas a


esquizogonias eritrocíticas tardías, que son
consecuencia del aporte de merozoitos
procedentes del hígado que a su vez es procedentes
de hipnozoitos activados.
b)Recrudescencia o recrudecimiento.

EN FALCIPARUM Y MALARIAE. Se debe a un aumento de


Desde 1880, el diagnóstico de esta enfermedad se
una población superviviente de plasmódidos
ha realizado mediante la observación del
eritrocíticos, pasando de una parasitemia baja y
parásito , de sus distintas formas en el examen
muy escasa o nula sintomatología, a una
parasitemia elevada. microscópico de sangre periférica utilizando
El recrudecimiento es corto (en el tiempo) en diversos colorantes.
falciparum y largo en malariae.
¿Será verdad tanta
En la infección por Plasmodium falciparum,
además, existe una complicación llamada fiebre belleza?
hemoglobinúrica, que consiste en un aumento de
la temperatura, anemia severa, ictericia,
insuficiencia renal y hemoglobinuria.

159
Manual de INFECTOLOGÍA PLUS MEDIC A

!No es verdad tanta belleza!

Desventajas

Θ Se requiere un microscopista bien


entrenado
Θ Es dificil observar parasitemias bajas Antígenos palúdicos
Se detectan mediante inmunocromatografía en una
SE PUEDEN REALIZAR EXTENSIONES DE SANGRE FINA Y tira de nitrocelulosa o tarjeta que incorpora bandas de anti -
GRUESA; de las cuales la última tiene la ventaja de cuerpos monoclonales específicos .
que concentra los parásitos (20 a 40 veces más que
las extensiones finas). Los TRD se dividen en 3 grupos según el
Para realizar la microscopía las antígeno principal detectado:
extensiones se tiñen con Giemsa, la
cual permite observar las granulaciones a) Los que detectan la proteína 2 rica en histidina
de Schüffner típicas de la infección en (HRP-2) producida por P. falciparum exclusivamente
especial por P. vivax , b) Los que detectan HRP-2 y una enzima producida por
todos los plasmodios (antígeno panmalárico), bien la
La microscopía permite el diagnóstico de aldolasa o bien la LDH
infecciones mixtas y cuantificación del parásito,
c) Los que detectan una LDH específica
por lo que permite monitorizar el tratamiento .
√de falciparum (PfLDH)
√de P. vivax
Otros métodos diagnósticos
TRD: Test rápidos de detección de antígenos √producida por todas las especies de Plasmodium (PLDH).
Detectan antígenos específicos (proteínas) World Health Organization. Rapid Diagnostic Tests.
que producen los plasmodios presentes en: Results of round 3: 2012 [consultado 1-10-2014]. Dis
ponible en http: //www.who.int /tdr/publications / td-
◎sujetos infectados o
rresearch- publications/rdt_round3/en/index.html
◎de los infectados que recibieron trata -
miento recientemente (en las 4s previas). ◆ El TRD es una prueba inicial
◆El TRD no sustituye a la gota gruesa porque su espe-
Detección de antígenos parasitarios
cificidad es baja y tiene un número elevado de de falsos ne -
TRD: Test rápidos de detección de antígenos
gativos en los casos de malaria no falciparum.
√ Son pruebas muy fáciles de realizar, rápidas,
sensibles y no precisan microscopio.
√ Los sistemas comerciales (dipstick, "jabonera") son
estables a temperatura ambiente, lo que permite el trans
porte al trópico, y constituyen una importante ayuda para
el diagnóstico de malaria en los laboratorios con poca
experiencia en la microscopía.

160
Manual de INFECTOLOGÍA PLUS MEDIC A

Test inmunocromatográficos
◆Detección del HRP-2.
√ La proteína-2 rica en histidina (HRP-2) se secreta
por P. falciparum a la sangre
√ Permite su detección mediante la captura antigénica
con anticuerpos específicos y técnicas de
inmunocromatrografía.

◆Detección antígeno HRP-2 de P. falciparum y


antígeno panmalárico
El antígeno panmalárico se expresa en las fases
sanguíneas de P. falciparum y P. vivax y, probablemente, Pruebas de PCR
tam bién de P. ovale y P. malariae.
√ Sensibilidad 90-92% y una especificidad del 96- pruebas de PCR
98%.
Para P. vivax son inferiores, del 75% y 95%
ΘMétodos de diagnóstico y caracterización basados en
Desventajas
la amplificación genómica
√ No detectan parasitemias bajas (<0,1%),
√ Presentan falsos negativos Θ La sensibilidad, en torno a 0,01 parásitos/l de
√ Presentan falsos positivos, especialmente en pre - sangre, es mayor que en los métodos
convencionales
sencia del factor reumatoideo
√ Basados en microscopia (5-20 parásitos/l) o
◆ Detección de la lactato deshidrogenasa (LDH) √ Métodos basados en detección de antígeno (100
parasitaria.
parásitos/l) √ La especificidad es alta y permite una
Se basa en la detección de esta enzima parasitaria
-LDH-, común a las cuatro especies de Plasmodium. mejor caracte -rización de especie y la detección de
La especificidad es similar a las técnicas que detectan HRP- infecciones mix
2, pero la sensibilidad es un poco inferior (88-90%),
disminuyendo ésta a medida que la parasitemia baja Trans R Soc Trop Med Hyg. 2002; 96 Suppl
1: S199 - 204.
PCR (Reacción en cadena de polimerasa)
Métodos de PCR convencional para la de -
tección de las diferentes especies de Plasmodium:
Θ Ventaja:
√ Capacidad que posee para detectar Plasmodium a) Nested PCR
en pacientes con bajos niveles de parasitemia
Una primera amplificación seguida de 4 ampli-
√ Capacidad para identificar la malaria a nivel de
ficaciones
especies y la detección de infecciones mixtas
una para cada una de las 4 especies de
√ Es efectiva empleando pocos ugL de sangre y con la Plasmodium
presencia de 5 ó menos parásitos por uL y en función de
b) Nested Multiplex
los cebadores empleados se puede llegar al 100% de
sensibilidad y especificidad 2 amplificaciones encadenadas
en la segunda de las cuales el tamaño de los
fragmentos determina la especie o especies
de Plasmodium .

161
Manual de INFECTOLOGÍA PLUS MEDIC A

PCR a tiempo real El método de IFI es un método que permite


Tiene menor sensibilidad que los métodos cuantificar la respuesta inmunológica frente
al parásito.
basados en PCR convencional

Nested PCR

Son menos recomendables, ya que no permiten la


cuantificación de los microorganismos y presentan un
elevado riesgo de contaminación cruzada entre las muestras;
√ Por la persistencia de anticuerpos en el
Kamau E, Alemayehu S, Feghali KC Multiplex tiempo, no diferencia infecciones pasadas de actuales
qPCR for detection and absolute quantification √ En primoinfecciones, debido al tiempo reque-
of malaria. PLoS One. 2013;8:e71539
rido para el desarrollo de anticuerpos, no
aparecen anticuerpos (seroconversión) hasta después
de varios días tras la aparición de los síntomas

La detección de anticuerpos puede ser útil para:


◆ Cribado de donantes de sangre nativos o viajeros de
zonas endémicas
◆ Diagnóstico retrospectivo cuando se desea confirmar el
diagnóstico previo en pacientes no inmunes que sufrie -ron
un proceso febril durante un viaje
El diagnóstico por PCR debe considerarse
◆Estudios epidemiológicos
como método de segunda línea,
Ariey F, Witkowski B, Amaratunga C, Beghain J,
Razones : Langlois AC, Khim N, et al. molecular marker of
√ El tiempo de diagnóstico, entre 6 y 24 artemisinin-resistant Plasmodium falciparum
h,retarda el tratamiento (microscopia < 1 h o los malaria. Nature. 2014; 505:50-5.
TDR < 30 min ) OF: ¿Cuáles de las siguientes características
√ El costo en equipos puede ser excesivo sugieren forma grave de malaria por P. falciparum?
para muchos laboratorios . A.- Cefalea y mialgia.
B.- Alteraciones mentales, signos neurológicos y
Técnicas serológicas convulsiones.
C.- Esplenomegalia y anemia.
Las pruebas serológicas detectan anticuerpos D.- Escalofríos, fiebre y sudores.
E.- Malestar general, anorexia e ictericia.
en sangre producidos en respuesta a una infección por Rpta. B
plasmodio Con
frecuencia se utilizan métodos basa- dos en OF : Son complicaciones de la Malaria por
inmunofluorescencia indirecta (IFI) o Plasmodium faciparum. Marque lo falso:
enzimoinmunoanálisis (ELISA).
a. Insuficiencia renal aguda
b. Paludismo cerebral

162
Manual de INFECTOLOGÍA PLUS MEDIC A

c. Ruptura tardía de bazo C.Quinina


d. Anemia severa D.Mefloquina
e. Esplenomegalia tropical E.Hidroxicloroquina
Rpta B
Rpta. C
Essalud 08 : Tratamiento de elección para la forma
Tratamiento intracelular hepática del Plasmodium vivax :

a.Quinina
Régimen de tratamiento para el p. vivax: b.Primaquina
c.Pirimetamina
◆Cloroquina d.Cloroquina
Dosis : e.Mefloquina
1er y 2do días: 10mg/kg/d Rpta. B
3er día: 5mg/kg/d
◆Primaquina Régimen de tratamiento para el
Dosis : 0.5mg /Kg/d 2tab c/d x 7 días P. vivax en gestantes
Cloroquina actúa mejor en la fase
eritrocítica esquizogónica (en el GR) ◆Cloroquina
Primaquina actúa mejor en la fase Dosis :
esquizogónica hepática (en el GR) ↓la 1er y 2do días: 10mg/kg/d (4 Tb de 250mg)
recaida 3er día: 5mg/kg/d (2 Tb de 250 mg)

Luego :
2tab VO c/ semana hasta la semana del parto

Posterior al parto:
◆Primaquina: 2 tab de 15mg VO c/día x 7
días

√ Está contraindicada la primaquina


Riesgo de hemólisis y metahemoglobinemia en el
feto

Régimen de tratamiento para malaria


no complicada por P. falciparum
Por 3 días

◆Sulfadoxina 500mg-Pirimetamina 25mg


RM 10-B (54): 54-B Único fármaco disponible 3 Tb Vo c/día x 3 días
para erradicar la fase hepática de los plasmodios:
◆Artesunato(derivado de Artemisinina)
A.Cloroquina 1 Tb 250mg o 5 tb de 50mg x 3 días
B.Primaquina

163
Manual de INFECTOLOGÍA PLUS MEDIC A

Régimen de tratamiento para malaria por p.


falciparum resistente a sulfadoxina-
pirimetamina

◆ Artesunato TB 250mg
4mg/kg/día 1 Tb VO c/d
x 3 días
◆ Mefloquina Tb 250mg
12.5mg/kg/día 3 Tb VO c/d
x 2 días (2do y tercer días)

ENAM 05-A (3): En gestantes, el fármaco


antipalúdico indicado es:
A. Primaquina
B. Dapsona
C. Sulfadoxina
Régimen de tratamiento para malaria no
D. Cloroqulna
complicada por p. falciparum en gestantes E. Pirimetamina
Rpta. D
http://www.plus-
◆ Quinina TB 300mg
medica.com/portal/index.php?p=pm_tabla&t=17&fx=
10mg/kg/día 2tab VO c/8h (M-T-N)
8&acc=4&id=22&idr=426
x 7 días
RM 11-A (9): Síntomas de mal pronóstico
◆ Clindamicina caps 300mg
10mg/kg/día 2caps VO c/12h (M-N)
en la enfermedad por Plasmodium
x 5 días (Del 3er al 7mo día) Falciparum (paludismo):
◆ Primaquina caps 300mg
0.75mg/kg/día 3 Tb VO c/d a. Dolor y anuria
x 1 día (Al 3er día) b. Diarrea y fiebre
c. Anuria y convulsiones
√ Está contraindicada la sulfadoxina -
d. Fiebre y escalofríos
pirimetamina y artesunato-mefloquina
e. Dolor abdominal
ENAM 09-B (72): Paciente de 54 años de edad, Rpta.C
procedente de Pucallpa. Acude a Hospital de la
capital porque tiene fiebre intermitente y palidez. ENAM 11 (7): Complicación más
¿Cuál es el diagnóstico más probable?: peligrosa de malaria por P
A. Malaria falciparum:
B. Hepatitis C -Insuficiencia renal
C. Fiebre tifoidea http://www.plus-
D. Brucelosis medica.com/portal/index.php?p=pm_tabla&t=17&fx=
E. Bartonelosis 8&acc=4&id=22&idr=426
Rpta. A

164
Manual de INFECTOLOGÍA PLUS MEDIC A

165
Manual de INFECTOLOGÍA PLUS MEDIC A

FACIOLASIS PLUS MEDIC A


INTRODUCCIÓN Ciclo vital dentro del caracol(PRIMER
La fasciolosis es una zoonosis de distribución HUÉSPED INTERMEDIARIO)
mundial, causada por Fasciola hepatica
(Trematoda, Digenea), un platelminto de las
zonas templadas.

CICLO VITAL
Indirectos
REQUIERE EL PASO POR UNO O MÁS HOSPEDADORES
INTERMEDIOS(generalmente 2).
En estos los estadios inmaduros sufren cambios
morfológicos considerables.

Fig 3 Ciclo vital del tremátode dentro del caracol


Miracidio
PENETRA ACTIVAMENTE EN SU HOSPEDERO
INTERMEDIARIO, a menudo un peque-
ño caracol anfibio o acuático.
Fig 1 Los tremátodos “saltan” de un Dentro del caracol, el miracidio se desarrolla al
hospederto a otro estadio siguiente, el ESPOROCISTO
Los trematodos inmaduros pueden reproducirse
asexualmente (poliembrionía), (un único huevo
puede originar varios adultos).

Huevos fertilizado
Salen del hospedador de ordinario a
través de las heces, orina o esputo

LOS HUEVOS EMBRIONAN EN MEDIO ACUÁTICO. CADA


HUEVO PRODUCE UNA LARVA LIBRE CAPAZ DE NADAR Fig 4 Esporocisto
Y DENOMINADA MIRACIDIO (larva ciliada
acuática) Esporocisto
Puede dividirse y producir varios
esporocistos.
CADA ESPOROCISTO SE REPRODUCE ASEXUALMENTE Y DA
ORIGEN AL SIGUIENTE ESTADIO DE DESARROLLO,
LAS REDIAS.

Fig 9 Miracidio

166
Manual de INFECTOLOGÍA PLUS MEDIC A

Fig 14 Ciclo vital de la Fasciola hepática: huevo, miracidio. Ciclo vital en el primer hospedero que es
un caracol (el miracidio se convierte en esporoquiste,éste en redia y la redia en cercaría).Las
cercarías se enquistan como metacercarias y son ingeridas en plantas acuáticas por
humanos ,ganado,ovejas (hospederos definitivos) .Se desenquistan en el duodeno y migran al hígado. Allí
se ubican en los conducto biliares en donde finalmente se desarrolla el adulto que elimina huevos con la
bilis y las materias fecales. El ganado vacuno, lanar y ocasionalmente el ser humano actúan como
huéspedes definitivos

Redia
Cada redia también
puede a su vez
dividirse y DAR LUGAR
A CERCARÍAS.
Fig 5 Redia Las cercarías habitual-
mente con cola natatoria;
salen del caracol y PUEDEN
SER INFECTANTES

Fig 6 Cercarias

167
Manual de INFECTOLOGÍA PLUS MEDIC A

Metacercarias Al analizar la sintomatología, se observa que el


DOLOR ABDOMINAL es el más frecuente (91.8%).
Solo en algunas especies las cercarias de vida
libre pueden enquistarse en el segundo
Los síntomas que siguieron en frecuencia fueron:
hospedero intermediario produciendo
►Náuseas y vómitos (41.2%)
METACERCARIAS .
►Diarrea (38.9%)
TANTO LAS CERCARIAS COMO LAS ►Pérdida de peso (37.7%)
METACERCARIAS PUEDEN INFECTAR AL hiporexia (32.3%).
GANADO O LAS MASCOTAS. Se ha divido en 2 etapas, relacionadas con el
estadio de desarrollo y número de los
UNA VEZ INGERIDAS POR EL HOSPEDERO DEFINITIVO SE parásitos.
DESARROLLAN A ADULTOS Y EMIGRAN HACIA SUS
ÓRGANOS PREDILECTOS En su inicio
Una variante : género Schistosoma, cuyas Corresponde a la migración de los tremátodos
cercarias nadan en el agua e infectan al inmaduros (período de invasión, migratorio o
ganado de modo activo y directo penetrando a agudo) desde intestino hasta vías biliares.
través de la piel.
Período de estado (o fase crónica)
PATOgENIA Se presenta después de unos 3 - 4 meses
postinfección, dichas manifestaciones están
Hepatitis traumática hemorrágica y asociadas a la presencia de los trematodos
necrosante adultos en vías biliares.
Las formas juveniles en su fase Durante el período invasivo, el cuadro clínico
migratoria causan el engrosamiento de la incluye: dolor localizado en epigastrio y/o
cápsula de Glisson y una hepatitis cuadrante superior derecho con irradiación a
traumática hemorrágica y necrosante. escápula del mismo lado, hepatomegalia.
Con destrucción de hepatocitos que pueden Además brotes febriles irregulares, naúseas,
llevar a la muerte en casos graves. vómito, diarrea, hiporexia, mialgias, artralgias,
urticaria fugaz con dermografismo ocasional.
Fibroesclerosis ductal
Una vez instalados en los canales biliares En la biometría hemática puede apreciarse
producen irritación y lesiones, fibrosis, leucocitosis con desviación a la izquierda,
dilataciones, obstrucción de ellos por anemia y eosinofilia (30 - 70%).
fibroesclerosis ductal y/o por acumulación de
cálculos y abscesos. Período de estado
Cuando esto sucede a nivel de conducto
cístico, vesícula biliar o colédoco genera Se caracteriza por signos y síntomas
retención de bilis e incremento de la fosfatasa relacionados con la obstrucción biliar
alcalina, transaminasemia, leucocitosis y intermitente y el grado de inflamación: dolor
anemia con desnutrición,.
abdominal, náuseas, vómitos, ano-
CUADRO CLíNICO rexia, ictericia de tipo obstructi- vo),
hepatomegalia blanda y fiebre.
En el hombre, la migración del parásito y su Se ha reportado un mayor riesgo de colelitiasis
ulterior localización en las vías biliares en zonas endémicas, asociado con el número
determina un cuadro clínico caracterizado de parásitos en conducto biliar, colangitis,
por un ESTADO DE HIPERSENSIBILIDAD Y SÍNTOMAS cronicidad y los niveles de lipoproteínas de
HEPATOBILIARES. alta densidad y triglicéridos.

168
Manual de INFECTOLOGÍA PLUS MEDIC A

También se han reportado cirrosis periportal y Fase de estado (crónica)


pancreatitis
Asimismo, existen publicaciones de casos con Los exámenes parasitoscópicos son positivos
carga parasitaria importante y ausencia de transcurridos 3 - 4 meses postinfección,
manifestaciones clínicas y de migraciones cuando los parásitos adultos eliminan huevos y
erráticas (fasciolosis ectópica) en diferentes éstos pueden identificarse en:
tejidos y síndromes con componente alérgico.
Exámenes coproparasitoscópicos de con-
centración por sedimentación
Entonces el cuadro clínico de la fasciolosis
humana, es muy variado, y abarca desde
La eliminación de huevos es irregular y puede
formas abortivas y asintomáticas, hasta ser baja o inexistente en infecciones con uno o
cuadros severos de hemorragia intra- pocos parásitos, en infecciones crónicas,
abdominal y abdomen agudo quirúrgico. ectópicas.
Dan "falsos positivos" en sujetos que han
Antes se le clasificaba en dos etapas como se ingerido hígado infectado de ganado.
mencionó en líneas anteriores:
-Aguda o invasiva (tres meses) Detección de coproantígenos
-Crónica (de localización biliar, asintomática).
Métodos invasivos
Actualmente se le clasifica en tres grupos: Estudio de contenido duodenal y biopsia de
-Forma aguda, con 3 subtipos diferentes (típica, tejidos.
atípica y ectópica) La colangiopancreatografía retrógrada
-Forma crónica endoscópica es necesaria en ocasiones para
-Asintomática. identificar defectos de llenado en conductos
biliares o vesícula, y permite recolectar las
DIAgNÓSTICO formas juveniles o adultas del parásito
(procedimiento diagnóstico y terapéutico).
Existen cuatro métodos de diagnóstico: También puede recurrirse a la colangiografía
percutánea.
-EL MÉTODO DE DIAGNÓSTICO BASADO EN LOS
SÍNTOMAS y que debe ser confirmado por los Otros estudios:
métodos de laboratorio.
-EL COPROLÓGICO, por el hallazgo de huevos en las -Pruebas de función hepática.
heces, por sedimentación. -Tomografía computarizada hepática
-EL SEROLÓGICO, o inmunológico por detección de Después de la administración de medio de
anticuerpos en el suero por ELISA; contraste, es frecuente observar lesiones
-EL POST MORTEM, por hallazgo del parásito a la
hipodensas pequeñas de 2–10mm de
necropsia
diámetro, microabscesos distribuidos con
Fase inicial (migración, aguda) patrón radial o ramificado, lesiones
subcapsulares y hemorragias, engrosamiento de
•Serología: hemaglutinación indirecta, ELISA e la cápsula hepática.
inmunofluorescencia indirecta.
•Leucocitosis con eosinofilia. -Ecografía abdominal
•Hipergammaglobulinemia En conductos biliares o vesícula biliar se
•Pruebas funcionales hepáticas. busca la forma adulta) y ultrasonografía
(permite visualizar parásitos adultos en
movimiento, en conductos biliares o vesícula).

169
Manual de INFECTOLOGÍA PLUS MEDIC A

-PCR escleras. Examen de laboratorio: hemograma


Existen estudios en los que se ha utilizado PCR con 30% de eosinófilos. ¿Cuál es el diagnóstico
para la identificación de especie: Fasciola más probable?:
hepatica y Fasciola gigantica. A. Fasciolosis
EL DIAGNÓSTICO DEPENDE ENTONCES DE LA FASE B. Hidatidosis
EN QUE SE ENCUENTRE EL PACIENTE: C. Ascariosis
D. Oxiurosis
Fase aguda o invasiva
Sólo puede ser diagnosticada mediante el uso E. Uncinar
de métodos serológicos. Rpta. A

Fase crónica ENAM 04-B (32) : ¿En cuál de las siguientes


Por el hallazgo de huevos en heces mediante la parasitosis se observa con mayor frecuencia
técnica de Sedimentación Rápida de Lumbreras. eosinofilia de más del 40%?
Otras técnicas coprológicas tienen menos A. Giardiosis
sensibilidad. Pueden existir falsos positivos en B. Fasciolosis
personas que han ingerido hígado crudo o mal C. Cisticercosis
cocido con los parásitos adultos. D. Hidatidosis
E. Oxiurosis
Antes se utilizaron pruebas como la Rpta. B
intradermorreacción, las reacciones de fijación
de complemento, inmunofluorescencia TRATAMIENTO
indirecta, y de hemaglutinación.
Finalmente, el tratamiento de elección es el
Triclabendazol, usado desde 1998 en humanos.
LOS MÉTODOS SEROLÓGICOS SON: ARCO2, ELISA, Y
En dosis única de 10-12 mg/Kg, postprandial,
WESTERN BLOT. COPROANTÍGENOS, están siendo
demostrando ser inocuo para humanos.
utilizados en humanos, pero las formas
extrahepatobiliares no serían detectadas
Actualmente la OMS ha recomendado incluir al
PATOgENIA triclabendazol en el tratamiento para fasciola en
humanos por su eficacia y seguridad.
La patología en el período agudo comprende Otros fármacos utilizados son el bitionol,
abscesos eosinofílicos y trayectos de parásitos praziquantel o dehidroemetina, y en ocasiones se
marcados por zonas de necrosis rodeadas de requiere cirugía.
infiltrado inflamatorio y cristales de Charcot-
Leyden, fibrosis y tejido granulomatoso. Su espectro clínico es muy amplio, y a que las
Período crónico técnicas diagnósticas dependen de la fase en
Se observan hiperplasia e hipertrofia de que se encuentre el paciente, es una
los conductos biliares con fibrosis y enfermedad que no se diagnostican
adecuadamente.
esclerosis periductal - con o sin datos de
obstrucción; áreas de necrosis, cambios
ENAM 09-B (60) : Paciente mujer de 35 años de
adenomatosos, hepatomegalia, atrofia parén-
edad, vendedor ambulante, con antecedente de
quimatosa,cirrosis periportal.
consumo de berros. Presenta dolor abdominal en
ENAM 08-B (100) : Varón de 50 años de edad, hipocondrio derecho. Al examen se palpa
agricultor y ganadero, que presenta dolor tumoración en dicha zona. ¿Cuál es el
abdominal continuo en hipocondrio derecho. Al tratamiento en dicho caso?:
examen se detecta ictericia de piel y A. Quinidina

170
Manual de INFECTOLOGÍA PLUS MEDIC A

B. Ceftriaxona
C. Tetraciclina
D. Triclabendazol
E. Mebendazol
Rpta. D

171
Manual de INFECTOLOGÍA PLUS MEDIC A

RABIA PLUS MEDIC A


TRASMISIÓN

INTRODUCCIÓN
El ser humano se infecta a través de la piel
Dentro de las zoonosis, la rabia constituye uno tras mordeduras o arañazos por animales
de los más graves problemas de salud pública infectados.
debido a su trascendencia, su gravedad e
impacto social porque conduce a la persona El perro
expuesta al virus rábico a una muerte segura, Es el principal huésped y transmisor de
si no recibe vacunación oportuna la rabia, es la fuente de la infección en la
totalidad de los 55 000 casos mortales de rabia
La rabia es una zoonosis humana que se producen anualmente en Asia y
La rabia es una enfermedad transmitida al ser África.
humano por los animales , causada por un
virus que infecta a animales domésticos y
salvajes, y se propaga a las personas a través
del contacto con la saliva infectada a través de
mordeduras o arañazos.
La rabia está presente en todos los
continentes, excepto en la Antártida, pero
más del 95% de las muertes humanas se
registran en Asia y África.
Una vez que aparecen los síntomas, la
enfermedad es casi siempre mortal.

DATOS IMPORTANTES

 Hay rabia en más de 150 países y


territorios. Los perros son fuente de infección de
 Cada año mueren de rabia más de 55 000 los 55 000 casos de rabia en Asia y
personas en todo el mundo. Africa.

 El 40% de las personas mordidas por
animales presuntamente rabiosos son
menores de 15 años. El murciégalo ¡horrible oye!
 Los perros están en el origen del 99% de
las muertes humanas por rabia.
 La limpieza de la herida y la inmunización
en las horas siguientes al contacto con un
animal presuntamente rabioso pueden En los Estados Unidos
evitar la aparición de la enfermedad y la de América y el
muerte. Canadá, la fuente de la
 Cada año más de 15 millones de personas mayoría de los casos
reciben profilaxis postexposición para humanos mortales es
evitar la enfermedad, y se calcula que esto el murciélago.
ahorra 327 000 muertes anuales.

172
Manual de INFECTOLOGÍA PLUS MEDIC A

La rabia del murciélago también se ha


convertido recientemente en una amenaza La transmisión de
para la salud pública en Australia, América persona a persona por
Latina y Europa Occidental. mordeduras es teórica-
Las muertes humanas tras la exposición a zorros, mente posible, pero
mapaches, mofetas, chacales, mangostas y nunca se ha confirma –
otros huéspedes carnívoros salvajes son muy do.
raras.

Contacto directo Mecanismo de infección


También puede haber transmisión al ser El hombre recibe el virus de la rabia a través del
humano en caso de contacto directo de material contado con la saliva del animal enfermo.
infeccioso (generalmente saliva) con mucosas o Esto quiere decir, que para ser inoculado, no
heridas cutáneas recientes. necesita necesariamente ser mordido: basta que un
tajo, herida, rasguño profundo o quemadura en su
Aunque es raro, también puede contraerse piel entren en contacto con la saliva del animal
la rabia por trasplante de órganos rabioso.
infectados o inhalación de aerosoles que
El virus
contengan el virus.
El virus de la rabia es un Lyssavirus, perteneciente a
La ingestión de carne cruda o de otros
la familia Rhabdoviridae.
tejidos de animales infectados no es fuente
de infección humana.

173
Manual de INFECTOLOGÍA PLUS MEDIC A

-Forma : de bala
-Tamaño: mide 75 nm de diámetro por 180 nm Si el mordisco del animal rabioso
de longitud. fue efectuado a través de la ropa
de la víctima, solamente una pe-
La membrana está recubierta por una serie de queña cantidad de saliva contami-
prolongaciones glucoproteícas, la proteína G, nada llegará hasta la herida, y es-
que es la proteína de adhesión vírica y la ta puede retardar el proceso de la
responsable de la aparición de anticuerpos afección.
neutralizantes. Su ácido nucleico es ARN
monocatenario de sentido negativo.
Período de incubación
CUADRO CLíNICO
-Período de incubación : es variable,
El virus de la rabia infecta a animales usualmente entre tres y ocho semanas.
domésticos y silvestres, y como se mencionó
anteriormente se transmite a las personas a
través del contacto estrecho con la saliva que Síntomas generales
resulta de una mordedura o arañazo. Las primeras manifestaciones de la rabia son
Cerca del 90% de los casos de en el hombre parecidas a las de la gripe: fiebre, dolor de cabeza
son debidos a la transmisión por el perro; el y fatiga.
gato es el responsable de solo un 5%. La fiebre se acompaña de dolor o parestesias
(sensación inusual o inexplicada de hormigueo,
Pero prácticamente todos los animales picor o quemazón) en el lugar de la herida.
domésticos son susceptibles a la rabia,
incluyendo las vacas y cerdos y también los
animales salvajes como murciélagos, mapaches,
zorros.

Tiempo de inoculación
Varía con la naturaleza del virus, el lugar de
inoculación y la cantidad inoculada.

Si el punto de contacto ha sido la cabeza, el


Mordedura canina
cuello o los miembros superiores, el
período de incubación será más breve, Manifestaciones clínicas respiratorias y en el
porque el virus alcanzará la región predilecta SNC
Posteriormente, afecta el aparato respiratorio y
con mayor rapidez (llega al sistema nervioso
el sistema nervioso central, para luego progresar
central principalmente a través de los
hacia la parálisis completa, seguida de coma y
troncos nerviosos, propagándose a lo largo
muerte en todos los casos.
de los nervios sensoriales). Las células que lo
acogen son destruidas. A medida que el virus se propaga por el sistema
nervioso central se produce una inflamación
A PARTIR DE AHÍ EL VIRUS EMIGRA HACIA LOS TEJIDOS, progresiva del cerebro y la médula espinal que
PERO SOBRE TODO HACIA LAS GLÁNDULAS SALIVALES,
acaba produciendo la muerte.
de donde es excretado juntamente con la saliva.

174
Manual de INFECTOLOGÍA PLUS MEDIC A

FORMAS CLÍNICAS c. Período de excitación:


1. Rabia furiosa ►Continúa la fiebre, presenta hiperestesia y sialorrea
Con signos de : (salivación abundante).
►Posteriormente, hay espasmos de la laringe y
►Hiperactividad
contracciones musculares dolorosas ante la presencia
►Excitación de agua (hidrofobia), a corrientes de aire (aerofobia) y a
►Hidrofobia y, a veces la luz (fotofobia). Las personas pueden manifestar
Aerofobia respuestas agresivas.
la muerte se produce a los pocos días por ►Además puede presentar dificultad para la deglución,
paro cardiorrespiratorio. delirio, alucinaciones y convulsiones.

2. Rabia paralítica (30%)


Tiene un curso menos dramático y generalmente d.Período paralítico:
más prolongado que la forma furiosa. ►Suele observarse hemiparesias, parálisis flácida y
Los músculos se van paralizando gradualmente, coma.
empezando por los más ►El paciente puede permanecer lúcido o con períodos
cercanos a la mordedura o de lucidez.
arañazo. El paciente va En los casos de rabia silvestre es la fase más manifiesta
entrando en coma y como regla general comienza por las extremidades
lentamente, y acaba por inferiores y en forma ascendente.
fallecer. La muerte sobreviene a consecuencia de parálisis
A menudo la forma paralítica no se respiratoria.
diagnóstica correc- tamente lo cual contribuye a La duración de la enfermedad generalmente es de dos
la subnotificación de la enfermedad. a seis días, prolongándose algunas veces por unos días
más.

PERÍODOS DESCRIPCIÓN CLÍNICA DE RABIA


HUMANA
Entonces en el ser humano presenta los siguientes El paciente con síndrome neurológico agudo
períodos: (encefalitis) dominado por formas de hiperactividad
seguido de síndromes paralíticos que progresan hacia
a. Período de incubación: Es un periodo variable el coma y muerte, por insuficiencia respiratoria,
que está comprendido desde el momento de la entre siete y diez días después de la aparición del
exposición hasta que se inicia la primer síntoma, teniendo el antecedente de
sintomatología. mordedura o arañazo de un animal sospechoso.
En el Perú se ha notificado rangos de 10 a 312 días,  Caso sospechoso:
con un promedio de 50 días. Este período está Compatible con la descripción clínica.
relacionado con la localización de la mordedura,  Caso probable:
carga viral e inervación del área afectada. Caso sospechoso con antecedente de mordedura,
arañazo o contacto con un animal sospechoso de
rabia.
b. Período prodrómico:
Caracterizado porque la persona presenta  Caso confirmado:
Caso probable que es confirmado por laboratorio ya
alteraciones sensoriales imprecisas
sea por serología IF o por cultivo en ratones.
(«hormigueos» o parestesias) o dolor relacionado
con el lugar de la mordedura del animal, malestar  Caso de complicación postvacunal:
Paciente con vacunación antirrábica en curso, con
general, insomnio, dolor de cabeza, fiebre,
cuadro sugestivo de encefalomielitis de no más de
sensación de angustia e intranquilidad, irritabilidad.
20 días de evolución.

175
Manual de INFECTOLOGÍA PLUS MEDIC A

Diagnóstico diferencial Informes de resultados


Paciente sin antecedente de mordedura
que presenta cuadro de encefalitis o Una prueba positiva es concluyente de
meningoencefalitis (forma paralítica): rabia y determina la necesidad de realizar
síndrome de Guillain Barré, polio, tétanos y las acciones correspondientes de control de
cuadros psiquiátricos. foco.

DIAgNÓSTICO ◆ Cuando la prueba de inmunofluorescencia directa


es negativa: Se realiza la prueba biológica.
No hay pruebas para diagnosticar la rabia en La prueba biológica es la única que
el ser humano antes de la aparición de las determina la negatividad de la muestra.
manifestaciones clínicas, y el diagnóstico
clínico puede resultar difícil, a no ser que haya CRITERIOS DE DIAGNÓSTICO DE
signos específicos, como la hidrofobia o la LABORATORIO PARA LA CONFIR -
aerofobia. MACIÓN DE CASOS DE RABIA HUMANA
Tras la muerte, la técnica diagnóstica
convencional consiste en la detección de Uno o más de los siguientes:
antígenos del virus de la rabia en el tejido ◆Detección del antígeno rábico
cerebral con anticuerpos fluorescentes.
por la prueba de anticuerpos fluorescentes directa (AF) en
tejido cerebral (obtenido post mórtem).
Prueba de inmunofluorescencia directa
Es una prueba que se realiza en muestras ◆Detección del antígeno rábico por AF
humanas o animales y se basa en una reacción en impronta de córnea o folículo piloso (obtenido ante
mórtem).
antígeno (Ag) – anticuerpo (Ac) que se hace visible
◆Detección del antígeno rábico por AF
a través del microscopio de fluorescencia.
en cerebros de ratones adultos o lactantes y en cultivo
celular, después de la inoculación de tejido nervioso
Esta prueba es altamente sensible y específica.
cerebral, saliva o líquido cefalorraquídeo (LCR).
◆ Detección de títulos de anticuerpos
Prueba biológica
Esta prueba consiste en inocular ratones albinos de neutralizantes del virus rábico
en el LCR de una persona no vacunada.
laboratorio con una muestra de cerebro, humano o
◆ Identificación de antígenos víricos por
animal sospechoso de rabia.
reacción en cadena de polimerasa (PCR)
en tejido fijo obtenido post mórtem o en un espécimen
Prueba negativa: clínico (tejido nervioso cerebral o cutáneo, córnea o saliva.
Si entre el quinto y el vigésimo primer día
de la inoculación, los ratones no mueren
ni presentan signos de rabia.
MANEjO
Si al contrario alguno muere, se realiza la ATENCIÓN DE PERSONAS
prueba de inmunofluorescencia directa en EXPUESTAS AL VIRUS RÁBICO
una muestra de cerebro del ratón albino
muerto. Exposiciones leves
Son mordeduras ocasionadas por perros o gatos
En caso de muestras de murciélagos se que no presentan signos o sospecha de rabia y cuya
recomienda esperar hasta 30 días de lesión sea única y superficial localizada en cualquier
inoculación para determinar su negatividad. parte del cuerpo que no sea cara, cabeza, cuello o
en el pulpejo de los dedos de la mano.

176
Manual de INFECTOLOGÍA PLUS MEDIC A

La exposición a la sangre, Si la situación epidemiológica lo requiere se


leche, orina y heces no recomienda infiltrar suero antirrábico alrededor y por
constituyen riesgo de debajo de la herida.
transmisión de rabia. Se aplicará inmunización antitetánica de acuerdo a
los antecedentes de vacunación del paciente. Además
se indicará antibióticos y otros que sean necesarios.

Exposiciones graves Tratamiento Antirrábico


Cuando se presenta por lo menos una de las Es la aplicación de la vacuna antirrábica o suero
siguientes condiciones: y vacuna, la cual debe ser hecha bajo vigilancia
-►Mordeduras localizadas en cara, cabeza, cuello o médica a fin de determinar la presentación de
pulpejo de dedos de las manos, por animales reacciones posvacunales.
domésticos con o sin sospecha de rabia. No existen contraindicaciones para el tratamiento
-►Mordeduras profundas o desgarradas. antirrábico.
-►Mordeduras múltiples. El uso de fármacos inmunosupresores
-►Mordeduras ocasionadas por animales (corticoides, antimaláricos, antineoplásicos)
desconocidos. deben ser suspendidos durante el período de
-►Mordeduras por perros y gatos que mueren administración del tratamiento antirrábico.
durante los siguientes diez días de la exposición.
-►Mordeduras por animales con diagnóstico Suero antirrábico
laboratorial de rabia. Es la aplicación de suero antirrábico
-►Mordeduras por animales silvestres susceptibles hiperinmune (transferencia pasiva de anticuerpos)
de rabia (murciélagos, monos, zorros, etc). con la finalidad de dar una protección inmediata
-►Contacto de heridas recientes o mucosas con saliva pero temporal.
de animal diagnosticado con rabia.
Indicaciones
Tratamiento local de las heridas por -►Mordeduras localizadas en cara, cabeza, cuello y
mordeduras pulpejo de dedos de la mano producidas por un perro,
gato u otro animal doméstico sospechoso de rabia.
El lavado inmediato y concienzudo de -►Mordeduras localizadas en cara, cabeza, cuello o
la herida durante un mínimo de 15 minutos con
pulpejo de los dedos de la mano. ocasionadas por
abundante agua y jabón, povidona yodada, cualquier
animales silvestres.
detergente de uso doméstico u otras sustancias que
-►Mordeduras localizadas en cara, cabeza, cuello o
maten al virus de la rabia, es el paso más importante
pulpejo de dedos de manos, ocasionadas por animales
para reducir la carga viral y la contaminación
huidos o desconocidos.
bacteriana en el lugar de la mordedura.
Estas indicaciones pueden cambiar de acuerdo con la
situación epidemiológica del área.
La herida no debe ser suturada porque
podría provocar la infiltración de mayor cantidad de
partículas virales, a excepción de mordeduras En las exposiciones graves provocadas por
extensas, desgarradas o localizadas en cara cuya perros o gatos conocidos y sin signos de rabia
lesión requiera sutura. requieren sólo vacunación antirrábica hasta
el quinto día de la mordedura. La
De ser necesaria la sutura, ésta debe observación clínica del animal mordedor se
quedar floja y con puntos de afrontamiento, sin realizará hasta el décimo día después de
suturar el tejido celular subcutáneo (TCSC), para ocurrido el accidente.
facilitar el drenaje.

177
Manual de INFECTOLOGÍA PLUS MEDIC A

Si no se dispone del suero al


¿Cuándo se debe aplicar el suero?
inicio del esquema del
-El suero debe ser aplicado, sea cual fuere el tratamiento, éste aún
intervalo transcurrido entre el momento de la puede ser administrado
exposición y el comienzo del tratamiento y por hasta antes de la aplicación
única vez en la vida del paciente.
de la séptima dosis de la
-Antes de la administración de suero antirrábico
se debe realizar una prueba de sensibilidad. vacuna tipo Fuenzalida –
En caso que se necesite la aplicación del suero Palacios o de la tercera
antirrábico debe realizarse a nivel hospitalario dosis de las vacunas cultivo
bajo estricta vigilancia médica y con todos los celular
insumos y equipos necesarios (equipo de trauma
shock) para resolver un posible choque
anafiláctico u otra reacción adversa. Suero antirrábico con vacuna CRL, tipo
Fuenzalida – Palacios
Tipos de Suero Antirrábico Cuando se utilice este suero antirábico se
completará un esquema clásico de vacunación
Heterólogo: (debido a que el alto nivel de anticuerpos
(ERIG) Elaborado con suero de equinos circulantes hace que las cuatro o cinco primeras
hiperinmunizados. dosis del esquema no puedan desarrollar
plenamente su capacidad formadora de
Dosificación: La dosis aplicada por vía inmunoglobulinas), cuidando que ambos
intramuscular es de : biológicos no coincidan en el lugar de aplicación.
40 UI por kg de peso corporal,
previa prueba de sensibilidad por vía Una persona que haya recibido suero,
intradérmica u oftálmica.
vacunación o que tenga antecedentes de
vacunación antirrábica debidamente
Homólogo:
registrado e independientemente del tiempo
(HRIG) inmunoglobulina antirrábica de
origen humano. transcurrido y que nuevamente presente
Elaborado con suero de humanos exposición grave, no recibirá suero
hiperinmunizados. antirrábico, sólo vacuna antirrábica.

Dosificación: La dosis aplicada por vía A fin de confirmar los títulos de anticuerpos
intramuscular es de 20 UI por kg de peso neutralizantes, es recomendable su dosaje, el cual
corporal. se realizará en instituciones especializadas
(Instituto Nacional de Salud y Centro Antirrábico
El suero debe infiltrarse alrededor y dentro de de Lima).
la/s herida/s, hasta donde sea anatómicamente
posible, lo restante debe administrarse por vía Reacciones Adversas por Sueros Antirrábicos
intramuscular en la región glútea en dosis
única. El shock anafiláctico, la enfermedad del suero y
el fenómeno de Arthus .Estas sustancias inducen
la producción de anticuerpos, las que en presencia
No excederse de la dosis recomendada porque
de complemento, forman inmunocomplejos
puede interferir con la respuesta inmunológica
circulantes que se depositan en los vasos y dan
a la vacuna. origen a los síntomas de la hipersensibilidad,
produciendo reacciones adversas inmediatas o
tardías.

178
Manual de INFECTOLOGÍA PLUS MEDIC A

Reacción inmediata: Shock Anafiláctico Vacunas derivadas de tejido cerebral – vacuna de


Este tipo de reacción ocurre muy raramente cerebro de ratón lactante (CRL) tipo Fuenzalida -
(1:40,000 tratamientos - Brasil). No se tiene Palacios
referencias en el Perú.
Sin embargo, el suero antirrábico debe ser El Ministerio de Salud utiliza la vacuna de
aplicado a nivel hospitalario con capacidad cerebro de ratón lactante (CRL) a virus
resolutiva para atender eventuales ocurrencias inactivado de comprobado poder antigénico.
de este tipo, debiéndose mantener al paciente Las vacunas son producidas y sometidas a
en observación por un período de dos horas. pruebas de calidad (inocuidad, potencia,
esterilidad) por el Instituto Nacional de Salud.
Reacciones tardías:
Enfermedad del Suero
La vacuna CRL se presenta en forma líquida,
La enfermedad del suero es un síndrome de
cada frasco de vacuna contiene 14 mL (dosis
naturaleza alérgica que tiene cuatro
de 2 mL), antes de usar se debe agitar
componentes: -Febril
-Cutáneo moderadamente el vial para obtener una
-Articular suspensión homogénea.
-Linfoganglionar. La dosis de vacuna CRL es de 2 mL,
independientemente de la edad, peso o sexo
Fenómeno de Arthus
Es otra manifestación alérgica que se puede
presentar luego de la inyección de antígeno en
un individuo previamente sensibilizado con
este.

Prueba de Sensibilidad
Para prevenir estas reacciones se debe
practicar una prueba cutánea destinada a del paciente.
conocer la sensibilidad del paciente al suero.
Antes de la administración del suero, tiene La vacuna se aplica por vía subcutánea en la
valor limitado por el alto número de falsos región periumbilical con aguja descartable
positivos, baja sensibilidad, bajo valor 25x5/8".
predictivo y el mismo riesgo de reacción
anafiláctica. Contraindicaciones
El Comité de Expertos en rabia de la OMS No existen contraindicaciones para el tratamiento
sugiere la aplicación directa del suero con antirrábico humano.
todos los cuidados necesarios para atender
una reacción. Tratamiento antirrábico específico
a. Preexposición:
Tipos de Vacuna Destinado a aquellas personas que tienen
Existen dos tipos de vacuna actualmente en el riesgo ocupacional por estar en contacto con
mercado internacional, las vacunas animales susceptibles a la rabia (médicos
derivadas de tejido cerebral y vacunas veterinarios, trabajadores de centros antirrábicos,
desarrolladas en cultivo celular. Todas personal que trabaja en control de rabia
son consideradas eficaces cuando se emplean silvestre, zoológicos) y que laboran con virus
oportunamente. rábico en el laboratorio o personal vacunador de
canes en situación de riesgo. podrán utilizar los
siguientes esquemas, vía subcutánea y en la
región periumbilical:

179
Manual de INFECTOLOGÍA PLUS MEDIC A

En el caso de situaciones que se requiera Vacunas de cultivo celular


una vacunación de preexposición en un En la actualidad también se encuentran
tiempo más breve utilizar el esquema B. disponibles en el mercado internacional vacunas
antirrábicas que utilizan como sustrato
Después de la inmunización de preexposición, para la replicación del virus, cultivos
determinar el título de anticuerpos 30 días celulares de origen humano (vacuna de células
después de aplicada la última dosis. diploides), también se emplean vacunas
producidas en células VERO, cuyo substrato es
El valor mínimo de anticuerpos requeridos riñón de mono verde africano y vacunas
es de 0,5 U.I. por mL, en caso contrario se preparadas en cultivos celulares de embrión de
administrará dosis de refuerzo hasta que haya pollo.
anticuerpos demostrables
a. Esquema preexposición
Si el paciente sufre un Se administran tres dosis los días 0, 7 y 28 por vía
accidente de mordedura IM en la región deltoidea (brazo). En los niños
y requiere tratamiento menores de dos años en la cara externa del muslo.
antirrábico, entonces se le
prescribirá el esquema b. Esquema postexposición
reducido. Se administran cinco dosis :
Los días 0, 3, 7, 14, 21 ó 28 por vía IM en
la región deltoidea.
b. Postexposición En los niños menores de dos años en la cara
De acuerdo con la clasificación de las externa del muslo.
exposiciones en leves o graves, se indicará la
conducta por seguir con respecto al animal Existe una alternativa, el régimen de
agresor y al tratamiento pasivo con suero postexposición 2-1-1, donde se administra :
antirrábico o activo con vacuna antirrábica. Dos dosis el día 0 (uno en cada región deltoidea)
y una dosis los días 7 y 21.
Esquemas de tratamiento antirrábico Se le utiliza cuando no esté indicado el suero
antirrábico.
Con la vacuna CRL existen dos esquemas de
vacunación, el «reducido» y el «clásico». Las vacunas de cultivo de tejido nunca
deben aplicarse por vía IM en región glútea.
-El esquema reducido: siete dosis + tres La profilaxis postexposición que se
refuerzos recomienda depende del tipo de contacto
Consiste en aplicar una dosis diaria por con el animal presuntamente rabioso.
siete días y los refuerzos a los 10, 20
y 60 días después de la última dosis de la serie. EsSalud 08 : El virus de la rabia tiene un periodo
de incubacion y agresividad variable esto
-El esquema clásico: catorce dosis + dos
depende de:
refuerzos
Consiste en la aplicación de 14 dosis en a.Si la mordida es en la cara
forma diaria ininterrumpida y dosis de refuerzo b.Es más agresiva si la mordedura es en el
a los 10 y 20 días de la última dosis de la serie. miembro inferior
Rpta A
ESTE ESQUEMA SÓLO SE UTILIZARÁ CUANDO SE
INDIQUE SUERO ANTIRRÁBICO.

180
Manual de INFECTOLOGÍA PLUS MEDIC A

Profilaxis Postexposición recomendada Tratamiento: Es sintomático (antihistamínicos y


contra la Rabia analgésicos), debiendo continuar el esquema de
vacunación.
Tipo de exposición al
animal Medidas -Sistémicas
Se puede presentar cefalea, decaimiento, mareos,
presuntamente posexposición: escalofríos, fiebre y exantema.
rabioso:
Tratamiento: Es sintomático (analgésicos,
Tipo I – alimentación Ninguna antipiréticos y antihistamínicos), debiendo continuar
de animales, la vacunación bajo supervisión médica.
contacto con ellos, o
-Neurológicas
lame- duras en zonas Ocurre generalmente en personas altamente
con piel intacta (es alérgicas. Las reacciones pueden ser neuritis
decir, ausencia de periférica, polineuritis, encefalitis. El paciente se
expo- sición) queja de adormecimientos en las extremidades,
disminución o ausencia de reflejos.
Tipo II – mordisqueo Vacunación y Se han reportado casos parecidos al síndrome de
de piel no cubierta, tratamiento local Guillain-Barré, parálisis ascendente de Landry o
encefalitis perivascular.
arañazos o abrasio- de la herida inme-
nes menores sin diatos Tratamiento: En caso de presentarse estas
sangrado reacciones en el paciente, deben ser inmediatamente
notificados, refiriéndosele a un hospital para la
Tipo III – mordeduras Vacunación y atención y manejo.
o arañazos transdér- administración de
micos únicos o inmuno- globulina Si se considera que las dosis aplicadas han sido
suficientes (mínimo siete dosis) lo que se
múltiples, lameduras antirrábica inme-
demuestra mediante dosaje de anticuerpos, la
sobre piel no intacta; diatas; tratamien-
vacunación se suspenderá definitivamente, de
contaminación de to local de la lo contrario se completará la vacunación con
membranas mucosas herida. otro tipo de vacuna.
por lameduras; expo-
sición a murciélagos
El cuadro neurológico se tratará
sintomáticamente y se administrarán corticoides
de acuerdo al criterio médico.
Las Reacciones Posvacunales pueden ser:
Toda persona que va recibir un tratamiento
-Locales antirrábico y que tiene antecedentes de
Son las más frecuentes, el dolor es la primera vacunación previa o antecedentes alérgicos,
sensación, seguido de hiperestesia, eritema y deberá recibir evaluación médica durante
prurito, en la zona de aplicación. Pueden el proceso de tratamiento y hasta quince
presentarse ronchas o adenopatía regional. días después del término de la vacunación por
el riesgo de reacciones tardías.

181
Manual de INFECTOLOGÍA PLUS MEDIC A

BRUCELOSIS PLUS MEDIC A


INTRODUCCIÓN Otro mecanismo
Por inoculación o contacto directo con
SINÓNIMO: es conocida también como animales infectados o sus productos (manejo de
sangre, orina, descargas vaginales, fetos abortados y
Fiebre de Malta, Enfermedad de Bang,
placentas de animales infectados).
Fiebre Ondulante o Fiebre del Por esto la gente que trabaja en mataderos o
Mediterráneo . granjas, como también los veterinarios están en
riesgo de adquirirla y diseminarla entre el
Es una clásica zoonosis (antropozoonosis) ganado sano cuando no se toman las medidas
sanitarias adecuadas.
transmisible al hombre a partir de animales
infectados, los cuales eliminan gran número de RESERVORIO
bacterias a través de los genitales, la leche y las heces.
Algunos de los reser-
EPIDEMIOLOgíA vorios naturales son:
-LOS BOVINOS, CAPRINOS,
En el Perú el agente etiológico más frecuente OVINOS, CERDOS Y
es Brucella melitensis . MAMÍFEROS MARINOS.
Zona endémica: zona central del país, en
los departamentos de Lima e Ica, con un
Por otro lado se han encontrado brucellas
reciente desplazamiento hacia el norte. en una inmensa cantidad de mamíferos tan
En Lima y Callao se registran el 95% de los dispares como pequeños roedores, cánidos,
casos notificados del país y es porque se camélidos y cetáceos.
continúa la costumbre ancestral de consumir
queso fresco de cabra sin pasteurizar.
Temporada: setiembre y febrero.

TRANSMISIÓN
Se transmite por ingestión de leche o
quesos o derivados lácteos sin pasteurizar.

ETIOLOgíA

Brucella es un parásito intracelular


"facultativo", esto es, puede vivir dentro y
fuera de la célula.
A esa categoría pertenecen también los agentes
causantes de la tuberculosis, legionelosis y
salmonelosis.

182
Manual de INFECTOLOGÍA PLUS MEDIC A

Son cocobacilos pequeños no móviles, no (digestiva, conjuntival o respiratoria). SON


encapsulados, gram negativos, y no forman OPSONIZADOS Y LUEGO FAGOCITADOS POR
esporas. POLIMORFONUCLEARES.

Crecen con lentitud en los cultivos, son Vía de


Puerta de Fuente de Población
aerobios, aunque algunas cepas requieren de Infec
Entrada Infección de Riesgo
ción
dióxido de carbono para su desarrollo (B.
Abortus). Leche
No fermentan hidratos de carbono. Mucosa cruda, Población
Oral
digestiva derivados en general
Son catalasa y oxidasa positivos, lácteos
reducen el nitrato, no fermentan los Trabajado
azúcares y tienen actividad ureasa variable. res en
Su membrana citoplasmática está recubierta por Productos contacto
una capa rígida de proteoglicano, y por una Piel animales con
membrana externa que presenta lipopo- erosiona contamina animales
lisacáridos endotóxicos (LPS). Por da, dos: infectados o
El género está compuesto por 6 especies, contac conjunti- placenta, sus
pero sólo 4 de ellas están asociadas con va,
to heces, productos
brucelosis humana.
mucosa secreciones (veterinarios
Especie Vías de nasal vaginales , matarifes,
Hués cuidadores),
de Transmis Patogenia
ped
Brucella ión personal de
Oral, Abortos. Orquitis. laboratorio
B. nasal y Epididimitis. Aerosoles
Bovino Ocasionalmente en
abortus conjuntiv
al. artritis. laboratorios
Personal de
con
Aborto. laboratorio,
Oral y muestras
Cerdos B. suis Esterilidad. Trabajadore
genital Respira Mucosa contamina-
Orquitis. s de la lana,
toria nasal das,
Abortos (poco personal de
vacunas
Ovinos B. ovis Genital. frecuentes). limpieza de
vivas,
Epididimitis. los establos
aerosoles en
B. Abortos. establos,
melitens Esterilidad.
Perros lanas
is, B. Oral y Epididimitis.
y otros Vacunas
abortus, Genital. Dermatitis
canidos
B. canis, escrotal. SON CONDUCIDOS A GANGLIOS LINFÁTICOS REGIONALES,
B. suis SI EL MICROORGANISMO RESISTE EL ATAQUE DEL
Inoculaci Fiebre aguda e SISTEMA INMUNITARIO.
B. ón intermitente. Se establece la infección crónica: pasa a la
melitens Conjuntiv Adenopatías. sangre, originando una bacteremia, y se
Hom is, B. al. Hepatoesplenome LOCALIZA LUEGO EN LOS ÓRGANOS DEL SISTEMA
bre abortus, Inhalació galia. RETÍCULO ENDOTELIAL (bazo, hígado, médula ósea,
4. Patogenia
B. canis, n. Complicaciones ganglios linfáticos y riñones).
B. suis Cutánea. ostearticulares.
Digestiva
La brucella penetra en el interior del organismo a
través de la piel dañada o por las mucosas

183
Manual de INFECTOLOGÍA PLUS MEDIC A

Es un parásito intracelular facultativo de este


Especies Huesped
sistema. Enfermedad Clínica
Brucella Natural
Puede evadir la respuesta inmune humoral B. Cabra, Enfermedad grave aguda y
frente a la infección. melitensis oveja complicaciones comunes
Sobrevive intracelularmente a no ser que se Enfermedad leve y
desarrolle inmunidad celular específica. B.abortus Vaca complicaciones supuradas
Provoca enfermedad grave con incidencia alta de raras
complicaciones serias, por su capacidad de
Enfermedad supurada
sobrevivir en células fagocíticas y multiplicarse
hasta alcanzar concentraciones elevadas. Cerdo, destructiva
B. suis
roedores con manifestaciones
B. MELITENSIS ES LA ESPECIE CON MAYOR crónicas
RESISTENCIA AL EFECTO BACTERICIDA DEL Enfermedad leve y
SUERO Y DE LOS FAGOCITOS, acentuando su B. canis Perro complicaciones supuradas
mayor virulencia (mayor virulencia junto raras
con B.suis). Es la causa más común de Fiebre
brucelosis -Essalud 06 (1)- La fiebre es alta (>=40ºC) con escalofríos , en
la ETAPA AGUDA se presenta todos los días por las
tardes (vespertina) .
CUADRO CLíNICO La sudoración no es generalizada sino
“parcelar” (a nivel de la car o tórax).
Aguda: hasta los 3 meses La fiebre es ondulante en la ETAPA SUBAGUDA (hay
Subaguda: hasta los 9 meses períodos sin fiebre).
Y en la ETAPA CRÓNICA es inusual que se presente
Crónica : > 9 meses
fiebre.
Periodo DE INCUBACIÓN: 2 A 3 SEMANAS, Adenomegalia y hepatomegalia
Cuadro CLÍNICO: Los síntomas en un 90% de los -Adenopatías en un 12 a 20%, de los casos,
casos van a consistir en : principalmente a nivel cervical e inguinal.
FIEBRE, CEFALEA, SUDORACIÓN PARCELAR , ASTENIA, -Hepatoesplenomegalia en un 30 a 50% de los
MIALGIAS Y ARTRALGIAS pacientes.
-ENAM 10-B (56); ENAM 03-B (61)-
ENAM 11 : Paciente con sudoración parcelar y
Las INFECCIONES CAUSADAS POR : sacroileitis ¿Cuál es el diagnóstico más probable?
B. MELITENSIS a.Brucelosis
B. SUIS b.Fiebre tifoidea
Son las más graves. c.Leptospirosis

Manifestaciones articulares
El sistema osteoarticular va a ser el más
frecuentemente afectado y en estos casos la
infección casi siempre obedece a B. melitensis;

Las principales formas de presentación


son SACROILEÍTIS, LA ARTRITIS PERIFÉRICA Y
LA ESPONDILODISCITIS.

184
Manual de INFECTOLOGÍA PLUS MEDIC A

Complicaciones ENAM 03-B (61) : Un paciente consumidor de


queso de cabra que presenta fiebre persistente,
Genitourinarias diaforesis, artralgias, mialgias, palidez y
La orquioepididimitis representa la forma más hepatoesplenomegalia. El diagnóstico etiológico
frecuente. más probable es:
A. Peste bubónica
Neurológicas B. Bartonelosis
La afección del sistema nervioso central es C. Tuberculosis
poco común y se manifiesta como D. Fiebre tifoidea
meningoencefalitis. E. Brucelosis
Rpta. E
Cardiológicas
La endocarditis es la afección cardiovascular
PATOgENIA ASOCIADA
más común y representa la mayor causa de
mortalidad, afectando con mayor frecuencia a La brucelosis puede afectar a distintos órganos y
la válvula aórtica. sistemas:
Osteomielitis: con dorsalgia o lumbalgia,
Gastrointestinal por la afectación frecuente de la columna
Existe compromiso del sistema lumbosacra.
gastrointestinal y hepático en un 30-60% de A veces presenta complicaciones con supuración
los casos. al exterior de una colección de material
La afección hepática usualmente se purulento proveniente de la articulación
manifiesta por elevación de enzimas afectada.
hepáticas y hepatomegalia, aunque puede En ocasiones aparecen abscesos paravertebrales.
presentarse hepatitis granulomatosa y
absceso hepático. Artritis: más frecuente en las rodillas
Abscesos esplénicos: que generalmente se
CLASIfICACIÓN calcifican.
Afectación genitourinaria: en algunas
CLASIFICACIÓN ocasiones puede manifestarse como una
-Aguda: 1 a 3 meses orquiepididimitis unilateral que no suele
-Subaguda: 4 -8 meses dejar secuelas.
-Crónica: > 8 meses Rara vez produce prostatitis brucelar o nefritis
intersticial aguda, produciendo granulomas no
caseosos y calcificaciones.
ENAM 10-B (56) : Mujer de 35 años,
procedente de Cajamarca, presenta desde hace Alteraciones neurológicas: (2-5%),
3 meses dolor lumbar, fiebre y sudoración pueden presentarse mielitis, meningo-
parcelar a predominio nocturno .Al examen encefalítis, radiculitis y neuropatía
físico: FC: 100 lat/min FR: 20 resp/min T:38.5ªC
periférica, presentan un mal pronóstico.
pálida con adenopatías generalizadas, hepato y EN EL LCR : PLEOCITOSIS LINFOCITARIA, AUMENTO DE
esplenomegalia ¿Cuál es el diagnóstico LAS PROTEÍNAS Y GLUCOSA NORMAL.
probable?
A. Fiebre tifoidea Endocarditis: (1-2%) Es la causa más
B. Tuberculosis de columna vertebral frecuente de mortalidad por este cuadro. Afecta
C. Fiebre malta principalmente a la válvula aórtica (81%),
D. Malaria pudiendo presentar dicha válvula vegetaciones
E. Leptospirosis voluminosas y ulceradas. Tiene un comienzo
Rpta. C insidioso y predomina en varones.

185
Manual de INFECTOLOGÍA PLUS MEDIC A

Como complicación pueden aparecer Rpta. A


embolias arteriales; y su curación exige
tratamiento antibiótico y además sustitución
DIAgNÓSTICO
quirúrgica de la válvula afectada.

Alteraciones del sistema respi- Hemograma: leucopenia o leucocitosis con


ratorio: Puede aparecer tos y expectoración, neutropenia.
Habitualmente el conteo leucocitario es normal
en raros casos presenta derrame o empiema
aunque puede estar disminuido
pulmonar.
TGO-TGP: en ocasiones se encuentra ligera
Alteraciones en la piel: Presentan transaminasemia.
erupciones transitorias, e incluso púrpura Imágenes: Los estudios de imagen demuestran
secundaria a trombocitopenia. anormalidades sólo en las formas localizadas de
Los veterinarios que extraen placentas de
brucelosis.
animales infectados pueden presentar una
erupción papulo-pustulosa local, se cree que
Cultivo:
por una reacción de hipersensibilidad.
El diagnóstico definitivo se
basa en el aislamiento de la
Brucella en cultivos de
sangre, médula ósea, hígado
y otros tejidos.

La sensibilidad de los hemocultivos para la


brucelosis aguda es de 80% y de
MIELOCULTIVO de 90% (Prueba de oro)

OF: Paciente de 45 años acude con T.E de 4


semanas presentando fiebre de predominio
nocturno y artralgias. Tiene como antecedente el
cnsumo de queso fresco. Ante la posibilidad del
diagnóstico de Brucelosis ¿Cuál es la mejor
prueba que solicitaría para demostrar el
diagnóstico?
A. Aglutinaciones
B. Urocultivo
C. Hemocultivo
D. Mielocultivo
Complicaciones de la Brucelosis E. Set de Brucella
Rpta. D
ENAM 03-B (55) :¿Cuál es el compromiso
articular más frecuente en la brucelosis ¿Cuántos días hay que esperar el resultado
crónica?: del mielocultivo?
A. Artritis periférica El desarrollo del microorganismo en el medio
B. Osteoartritis doble de Ruiz Castañeda habitualmente ocurre
C. Costocondritis entre los 7 y 21 días, aunque existen casos de
D. Sacroileitis crecimiento tardío que pueden llegar hasta
E. Gonoartritis los 35 días.

186
Manual de INFECTOLOGÍA PLUS MEDIC A

RM 12-B (67): Mujer de 24 años con varios días Las Pruebas Tamiz son:
de fiebre, de mayor intensidad por las tardes.
Recibe tratamiento sintomático sin mejoría; a) Prueba de Rosa de Bengala
hace 3 días se agrega dolor a nivel sacro iliaco, b) Prueba de Aglutinación en Placa
persistiendo el cuadro febril. ¿Cuál es el
diagnóstico más probable?
A.Salmonelosis Aglutinación con Rosa de Bengala
B.Malaria ES LA PRUEBA DE DESPISTAJE
C.Pielonefritis aguda Es una prueba muy económica y con una
D.Leptospirosis sensibilidad muy alta del 99%, pero tiene el
E.Brucelosis inconveniente de una especificidad del 40%.
Rpta. E Esta prueba es de utilidad en áreas rurales, en
donde no es posible llevar a cabo la
aglutinación en tubo y en casos en donde es
muy importante un tratamiento temprano
como en la neurobrucelosis, la artritis y la
¡TE ADVIEEERTO! orquitis.

Todas los mielocultivos si son negativos La enfermedad deberá ser corroborada por medio
son descartadas a los 21 días. Por eso de una prueba confirmatoria.
cuando solicites un mielocultivo encargarte de Esta prueba es rápida y cualitativa, detecta
consignar en la orden que es para hacer el específicamente anticuerpos de tipo IgG1
diagnóstico de Brucelosis. De esa manera esta contra Brucella, permite descartar las reacciones
muestra de ser negativa a los 21 días no será cruzadas o falsos positivos.
descartada , ya que en un porcentaje de pacientes
recién se aislará la brucella después de los
primeros 21 días.

En la actualidad existen medios de cultivo de


aislamiento rápido como los sistemas Bactec
Plus, Vital Aer y el medio difásico que
permiten la identificación del germen entre
60 y 160 horas,.
Prueba de Rosa de Bengala

Serológico Prueba de Aglutinación en Placa


-Aglutinaciones en placa Detecta anticuerpos de tipo IgG e IgM, es una
> = 1/320 prueba que reacciona rápidamente al inicio de una
Tiene una baja especificidad infección, esta puede permanecer en forma residual
-Aglutinaciones en tubo por mucho tiempo .
> = 1/160 PUEDE DAR REACCIONES CRUZADAS CON OTRAS
Tiene una mayor especificidad BACTERIAS.
Otros métodos de diagnóstico serológico son:
aglutinación con Rosa de Bengala, Reacción cruzada
aglutinación en tubo (SAT), fijación del
►(Salmonella grupo N (serogrupo 0:30),
complemento, Coombs anti-brucella y ELISA.
Vibrio cholerae,
La aglutinación con Rosa de Bengala y la ►Escherichia coli 0:157,
aglutinación en tubo son las pruebas ►Yersinia enterocolítica 0:9,
serológicas más frecuentemente utilizadas por ►Francisella tularensis, Stenotrophomonas maltophilia
su rapidez y sencillez. y otras bacterias).

187
Manual de INFECTOLOGÍA PLUS MEDIC A

*Cuando la Prueba Tamiz es positiva se Detecta IgG


realizará la prueba complementaria. El suero de Coombs (inmunoglobulina humana)
se encarga de facilitar la aglutinación de los
Pruebas complementarias : anticuerpos no aglutinantes del suero problema,
fijados a la suspensión antigénica de B. abortus.
El título obtenido es, por ello, como mínimo el de la
a) Prueba de Aglutinación en Tubo. aglutinación y generalmente es mucho más elevado,
b) Prueba del 2 Mercapto-etanol tanto más cuanto mayor es el tiempo de evolución de
la enfermedad.
Son pruebas cuantitativas que permite el
seguimiento, tanto en el estado infeccioso Prueba del 2-Mercaptoetanol
como durante el tratamiento, mediante la Prueba selectiva, detecta la presencia de
evaluación del título inmunológico. anticuerpos IgG, se basa en la degradación de las
Las pruebas complementarias corroboran la
IgM debido a la acción de compuestos que
prueba tamiz y son cuantificables,
permitiendo observar el descenso de las IgG, contienen el radical tiol como el 2-mercaptoetanol.
que indicaría la eficacia del tratamiento.
Nemotecnia
Prueba de Aglutinación en Tubo !Ge Ge
(SAT) pobrecita IgM!
Detecta anticuerpos contra la bacteria tanto (EL 2ME
de tipo IgM como de IgG, mediante la técnica destruye las
de aglutinaciones seriadas de suero en tubos. IgM y detecta
Un título > 1:160 se considera positivo; sin IgG)
embargo en áreas endémicas, se
recomiendan títulos > 1:320 y la prueba
puede permanecer positiva por tiempo ES ÚTIL PARA DETECTAR INFECCIONES CRÓNICAS EN LOS
prolongado. QUE LA PRUEBA DE AGLUTINACIÓN EN TUBO PUEDE
TENER UN TÍTULO BAJO, ya que el suero contendrá
exclusivamente anticuerpos IgG.

Prueba de Inmunoabsorción Indirec-


ta Ligada a Enzimas (ELISA)
También permite detectar y cuantificar los
anticuerpos IgG, IgM e IgA contra el
lipopolisacárido de la bacteria.
ESTE MÉTODO DIAGNÓSTICO ES MÁS SENSIBLE Y
En infección aguda aparecen rápidamente ESPECÍFICO QUE LA AGLUTINACIÓN EN TUBO .
anticuerpos IgM que son seguidos por
anticuerpos IgG e IgA, estos anticuerpos se Se recomienda en áreas endémicas y en
pueden detectar por aglutinación en tubo, en individuos con recidivas de la enfermedad.
placa o por microaglutinación.

Prueba de Coombs
Los anticuerpos detectados por método de
aglutinación son anticuerpos divalentes que
en casos de infección crónica suelen ser muy
bajos.
SE RECOMIENDA REALIZAR LA PRUEBA DE COOMBS,
LA CUAL DETECTA ANTICUERPOS IGG INCOMPLETOS o
también llamados univalentes

188
Manual de INFECTOLOGÍA PLUS MEDIC A

Prueba de Reacción en Cadena de la TRATAMIENTO


Polimerasa (PCR)
Es un método diagnóstico rápido para detectar Hay varios esquemas de tratamiento :
a la Brucella en muestras de sangre.
OTRA VENTAJA DE LA PCR ES QUE PERMITE 1.Sin complicaciones focales
DIFERENCIAR LAS DIFERENTES ESPECIES DE BRUCELLA.
En un estudio se demostró que la PCR mediante 2.Con espondilitis, sacroileitis
ELISA es una técnica que alcanza una sensibilidad 3.Neurobrucelosis
hasta del 94.9% y una especificidad del 96.5%. 4.Endocarditis

PCR se ha recomendado como el método Esquemas


diagnóstico de elección.
Sin complicaciones
focales
PCR tiene implicaciones pronósticas, ya
que puede ser utilizada como evaluación de la
Doxiciclina 100 mg VO
respuesta terapéutica.
Recientemente se demostró por medio de PCR c/12h x 6 semanas más
en tiempo real que el DNA de B. melitensis
persiste a pesar de existir curación clínica, lo Gentamicina 5mg/kg IM c/d
anterior, podría explicar la razón de las x 7 días
recidivas de la enfermedad y plantearía la
posibilidad de que la brucelosis una vez
adquirida, permanece como una infección Con
latente. espondilitis,sacroileitis

OF: Paciente de 35 años de edad, con tiempo de Doxiciclina 100 mg VO


enfermedad de 3 semanas, con manifestaciones c/12h x 6 semanas más
de artralgias, mialgias, cefalea global,
sudoración generalizada, temperatura Gentamicina 5mg/kg IM c/d
ondulante. Al examen: hepatoesplenomegalia. x7d
¿Qué análisis es prioritario para precisar el
diagnóstico?: Más
A. Transaminasas
B. Hemograma Rifampicina 10 mg/Kg c/d x
C. Urocultivo 3 meses
D. Aglutinaciones
E. Velocidad de sedimentación
Rpta. D Neurobrucelosis

Comentario Doxiciclina 100 mg VO


Se trata de un paciente con un enfermedad c/12h
subaguda caracterizada por fiebre ondulante y
artralgias y se encuentra en el examen físico Rifampicina 10 mg/Kg c/d
hepatoesplenomegalia.
El cuadro clínico es compatible con brucelosis y Ceftriaxona 2g EV c/12h
para el diagnóstico serológicose necesita
aglutinaciones en tubo. Hasta que el LCR se normalice.

AAC56: 1523, 2012

189
Manual de INFECTOLOGÍA PLUS MEDIC A

Endocarditis

Cirugía más

Doxiciclina 100 mg VO
c/12h

Rifampicina 10 mg/Kg c/d

TMP + SMx

x 1-6 meses

Más

gentamicina

x 2-4 semanas

CID: 56: 1407, 2007

En
gestantes

Rifampicina 900mg VO c/d x


6 meses

Systematic Review and Meta-Analysis of


Randomized Clinical Trials in the
Treatment of Human Brucellosis

190
Manual de INFECTOLOGÍA PLUS MEDIC A

FIEBRE TIFOIDEA PLUS MEDIC A


EPIDEMIOLOgíA
DEfINICIÓN
Reservorio
La fiebre tifoidea es una enfermedad sistémica, EL RESERVORIO DE ESTE MICROORGANISMO ES EL
febril, aguda, de origen entérico, secundaria a HOMBRE.
la infección por S. typhi Los contactos en el núcleo familiar pueden ser
Aunque ocasionalmente puede ser originada portadores transitorios o permanentes.
por S. paratyphi A, S. schotmuelleri o S. En muchas partes del mundo son más comunes
hirschfeldii (antes S. paratyphi C). los portadores fecales de corta duración que los
urinarios.
AFECTA ÚNICAMENTE AL SER HUMANO, cursa
Portador (5%)
habitualmente con afectación sistémica y, en
Son las personas que se recuperan de la infección,
ocasiones, puede originar complicaciones
pero continúan portando la bacteria en el cuerpo
graves como son la perforación intestinal y la
y la excretan en las heces y la orina..
enterorragia.

ETIOLOgíA

El agente infeccioso que produce la fiebre


tifoidea es la Salmonella typhi (S. typhi) o
bacilo tifoídico.
Serológicamente pertenece al grupo
Salmonella D sobre la base de su antígeno O,
es móvil y su flagelo porta el antígeno
flagelar (H), antígeno D, el cual se encuentra
aproximadamente en 80 otros serotipos de
Salmonella. El enfermo clínico constituye la fuente más
activa de infección y la única susceptible de ser
Polisacárido capsular sometida a un control adecuado.
Además, posee en su superficie un polisacárido Sin embargo también hay que considerar a la
capsular, el antígeno Vi que es un gran masa de infección desconocida en la
homopolímero del ácido N-acetyl galacturónico. comunidad constituida por casos
Observaciones clínicas y epidemiológicas en subclínicos, inaparentes y portado-
estudios con voluntarios han permitido res sanos.
conocer que las cepas de S. typhi que poseen Se puede establecer, sin duda, que la endemia
el antígeno Vi, son más virulentas que las que de fiebre tifoidea está condicionada, en gran
no lo tienen. medida, por el tamaño de ese reservorio.

Mecanismo de la infección
Las bacterias Salmonella Typhi se encuentran en
las heces de las personas infectadas. El modo de
transmisión más común de la fiebre tifoidea es a
través del agua y los alimentos conta-
minados con heces u orina de un enfermo o
portador.

191
Manual de INFECTOLOGÍA PLUS MEDIC A

Mecanismo de infección de la Fiebre tifoidea

Historia de María Mallon o “María Tifoidea”


Mary Mallon (“María Tifoidea”) que fue la
primera persona identificada como
Patogenia de la Fiebre tifoidea
portadora sana en los Estados Unidos.
Puede haber infectado a hasta 53 personas durante Los gérmenes son transportados a los
su carrera de cocinera en la ciudad de Nueva York. FOLÍCULOS LINFOIDES INTESTINALES
El investigador de la fiebre tifoidea George
Soper originalmente publicó hallazgos sobre En los folículos linfoideos intestinales las
Mallon en el número de JAMA del 15 de junio bacterias se multiplican en el interior de las
de 1907. células mononucleares.
Los monocitos transportan estos
¿Cuánto tiempo puede contagiar una microorganismos a los ganglios linfáticos
persona? mesentéricos.
El período de transmisibilidad dura mientras
persistan los bacilos en las excretas, por lo
común desde la primera semana hasta el final
de la convalecencia.
Cerca del 10% de los pacientes de fiebre
tifoidea no tratados dispersarán bacilos
durante los tres meses siguientes al
inicio de los síntomas, y del 2% al 5%
se tornarán portadores permanentes.

PATOgENíA
El tamaño del inóculo por S. Typhi, necesario
para causar la enfermedad es de entre 105 y Fig.Primera exposición de la S. Tiphy en la plcas
109 bacterias. de Peyer
La Salmonella typhi penetra por la boca, se Los gérmenes alcanzan después el torrente
multiplica en el ID durante 3-4 días, tras sanguíneo a través del conducto torácico,
anclarse a las microvellosidades del ribete en causando una bacteriemia transitoria.
cepillo del ileon, las bacterias invaden el
epitelio intestinal, aparentemente a través de Las bacterias circulantes penetran en las
las placas de Peyer. células reticuloendoteliales del hígado, bazo
y médula ósea.

192
Manual de INFECTOLOGÍA PLUS MEDIC A

La constipación es típica en adolescentes y


adultos.

La diarrea es más frecuente en los niños y


en pacientes con compromiso
inmunológico (37- 66%), con frecuencia
se acompañada de dolor abdominal y
vómito.

SIGNOS
Funciones vitales
Fig. 2da. exposición de la S. Tiphy en las placas de -Bradicardia relativa
Peyer: las bacterias a través de la vía biliar (disociación pulso temperatura):
reingresan a tracto GI. la fiebre no produce un aumento de la frecuencia
La vesícula biliar es especialmente susceptible del pulso.
a la infección, tanto a través de torrente -Pulso arterial dícroto:
sanguíneo, como por la vía biliar. pulso de doble onda , una sistólica y la otra
diastólica.
Se cree que la endotoxina circulante, un -Fiebre alta
componente lipopolisacárido de la pared
celular bacteriana, causa la fiebre prolongada y Examen general
los síntomas tóxicos de la fiebre. -Roseola tífica (13-46%) : exantema máculo
eritematoso (2 a 4 mm de diámetro) que se
PATOgENíA localiza en el tórax y abdomen , desaparece a la
digitopresión y se debe a émbolos microsépticos
Período de incubación: de 3 a 21 días cutáneos.
Se presenta en la primera semana de la
enfermedad
SINTOMAS
Se recomienda considerar el diagnóstico clínico
de fiebre tifoidea cuando los pacientes presenten
los siguientes datos clínicos:

• Fiebre >39º C durante más de 72hs


• Cefalea
• Malestar general
• Tos seca Roseola tífica

De acuerdo a numerosas series de casos, otras -Orofaringe: angina de Duwet


síntomas y signos que presentan los pacientes -Hepatomegalia (15-75%) y esplenomegalia (39-
con sospecha de fiebre tifoidea son: 64%),
La presencia de esplenomegalia y
• Constipación (10 – 79 %) hepatomegalia , no corresponde necesa-
• Vómitos (23 – 54 %) riamente a una complicación.
• Lengua Saburral (50 %)
• Exantema (13 a 46 %). DIAgNÓSTICO
• Dolor abdominal (19 – 56 %)
Definitivamente el diagnóstico de fiebre tifoidea
es clínico y de laboratorio.

193
Manual de INFECTOLOGÍA PLUS MEDIC A

ENAM 09-B (94) : Varón de 25 años de edad. La sensibilidad es del 36% al 70% y la
Desde hace 2 semanas presenta fiebre y cefalea especificidad del 76% al 99 % a partir de la
segunda semana
intensa. Al examen: temperatura 39° C,
Tiene un valor predictivo positivo (VPP) de
bradicardia, pulso dícroto, hepatoespleno- 78.7% y un valor predictivo negativo (VPN)
megalia y ruidos hidroaéreos aumentados. El de 84%.
diagnóstico más probable es:
A. Mononucleosis infecciosa La aglutinación de Widal puede ser positiva en
B. Fiebre malta las siguientes situaciones:
C. Fiebre tifoidea a) Paciente con fiebre tifoidea
D. Enfermedad citomegálica b) Inmunización previa con antígeno de
E. Toxoplasmosis adquirida Salmonella
Rpta. C c) Reacción cruzada ocasionada por una
infección por Salmonella no typhi.
Hemograma d) Paludismo, dengue y otras enterobacterias
Podemos encontrar: e) Tratamientos previos con antimicrobianos
-Anemia
-Leucopenia (20 al 25 %) con neutrofilia.
-Eosinopenia (70-80 %). La sensibilidad de la prueba
También se puede encontrar trombocitopenia
en la segunda semana de la enfermedad (10- de aglutinación de Widal en
15%). combinación con el
hemocultivo es hasta del
SEROLOGÍA 79%, con una especificidad
La prueba de aglutinación de Widal detecta de 81%.
anticuerpos contra los antígenos “O” y “H” de S.
typhi En los casos de reacciones febriles con títulos
para antígeno O y antígeno H ≥1:320 la
AGlUTINACIONES positivas sensibilidad disminuye hasta en un 46.6%, pero
Valores para antigeno O y antigeno H ≥ 1:160, la especificidad se incrementa hasta 99%.
con una sensibilidad 79.3% y especificidad
del 89%. La prueba inmunoabsorbente (“Typhidot –
M”) es una prueba rápida que detecta
La reacción de Widal demuestra la presencia de anticuerpos del tipo Inmunoglubulina M
anticuerpos aglutinantes contra: (IgM) contra S. typhi.
-Los antígenos H flagelar u O somático de la Tiene una sensibilidad del 54% y una
Salmonella typhi en el suero de los pacientes con especificidad del 91%.
fiebre tifoidea.
Hemocultivo
Los anticuerpos contra el antígenos O -En la 1era semana: su positividad es
aparecen a los 6 a 8 días de iniciada a mayor ,hasta en un 90% .
la enfermedad y desaparecen posteriormente Esto es debido a que durante ese periodo hay
entre 3 a 6 meses. mayor bacteriemia.
Los anticuerpos contra el antigeno H aparecen a
los 8 a 12 días, alcanzando títulos más elevados -En la 3era semana: la sensibilidad es
con respecto a los anti-0 y pueden persistir por aproximadamente 50%
más de un año. Esto es porque después de la 1era semana
disminuye la bacteriemia.

194
Manual de INFECTOLOGÍA PLUS MEDIC A

En los pacientes que han recibido Mielocultivo


antimicrobianos antes del diagnóstico el El aislamiento de S typhi a partir de tejido de
médula ósea (mielocultivo) tiene una:
desarrollo de S. typhi en cultivo sanguíneo
-Sensibilidad del 80% al 95%
es del 40 al 60%.
-Especificidad del 100%
Indicaciones:
El MIELOCULTIVO ES EL ESTÁNDAR DE ORO
para el diagnóstico de fiebre tifoidea.
-Síndrome febril
de más de 3 días de evolución
-No evidencia clínica de otras causas infecciosas. Se recomienda tomar un mielocultivo en
Particularmente, a finales de la primera semana o aquellos casos en los que existe alta sospecha
clínica de fiebre tifoidea con reportes de
durante la segunda semana de la enfermedad.
hemocultivo negativos.
Es positivo en cualquier momento de la
RM 11-B (61): ¿Cuál hace el diagnóstico de enfermedad, no se afecta con el uso de
certeza de fiebre tifoidea?: antibióticos.
a. Hemocultivo
b. Reacción Widal Otros cultivos:
c. Hematocrito Urocultivo:
d. Serológico tiene una sensibilidad aproximada del 30%.
e. Transaminasa Bilicultivo:
Rpta. A es positivo desde la 1era semana y no se afecta
con el uso de antibióticos.
Coprocultivo
Es el menos utilizado y sólo nos sirve para:

-Diagnóstico del estado de portador


-Diagnóstico de fiebre tifoidea a partir de la 2da
semana (valor diagnóstico dudoso).

Se encuentra
positivo en el
25% a 30% de
los adultos
60% de los
niños con
fiebre tifoidea.

ENAM 08-A (43) : Escolar de 7 años de edad, que


Los pacientes pueden eliminar durante mucho
tiempo la bacteria por las heces presenta desde hace 7 días fiebre en incremento,
decaimiento general, dolor abdominal, cefalea y
UN COPROCULTIVO POSITIVO NO HACE DIAGNÓSTICO diarrea. Examen clínico: palidez y hepatomegalia.
DE ESTA ENFERMEDAD. Exámenes auxiliares: hemograma con leucopenia y
desviación izquierda. Aglutinaciones: H: 1/160, O:
Se realiza con un hisopado rectal y el 1/320. ¿Cuál es el diagnóstico más probable?:
transporte es le Medio Cary-Blair.

195
Manual de INFECTOLOGÍA PLUS MEDIC A

A. Hepatitis B TRATAMIENTO
B. Hepatitis A LAS FLUOROQUINOLONAS SON EL TRATAMIENTO DE
C. Brucelosis PRIMERA ELECCIÓN PARA LA FIEBRE tifoidea en áreas
D. Bartonelosis en donde no existe resistencia a este
E. Fiebre tifoidea medicamento.
Rpta.E Las cefalosporinas de tercera generación) son
drogas también efectivas
COMPLICACIONES
Fluoroquinolonas
-Más frecuentes: sangrado gastrointestinal • Falla clínica (2.1% casos; IC 95%: 1.4 –
(10-20%)y perforación intestinal (1-3%).
3.2%),
-Menos frecuente: hepatitis, meningitis,
• Tiempo de desaparición de la fiebre (3.9
pancreatitis.
días; IC95% 3.8 a 3.9 días)
Sangrado intestinal • Recaídas (1.2 %, IC95%: 0.7 a 2.2%)
La ulceración de las placas de Peyer en el íleon • Estado de portador (1.5%, IC95%:0.5 a
puede producir hemorragia o perforación 0.9%)
intestinales ,especialmente en los cuadros tardíos
no tratados. Ceftriaxona
La tasa de letalidad del 10% puede disminuir al • Falla clínica (8.7 %; IC 95%: 6.1 a 12 %)
1% o menos con la administración inmediata de • Tiempo de desaparición de la fiebre (6.1
antibióticos. días; IC95%: 5.9 a 6.3 días)
• Recaídas (5.3 %, IC95%: 3.7 a 8.2 %)
OF: Paciente con fiebre y cefalea de tres semanas • Estado de portador (1.2 %, IC 95 % 0.4 –
de evolución, que presenta abruptamente dolor 3.2 %)
abdominal con abdomen en tabla. El diagnóstico
más probable es perforación intestinal secundaria Amoxicilina: 500mg c/8hrs por 14 días
a: Ceftriaxona: 2gr c/24hrs por 10-14 días
A. Tuberculosis digestiva
B. Fiebre tifoidea Las cefalosporinas de tercera generación
C. Linfoma no Hodgkin
se consideran antibióticos de elección para
D. Apendicitis aguda
el tratamiento de la fiebre tifoidea en las
E. Enfermedad de Crohn
siguientes situaciones:
Rpta. B
• Intolerancia a la vía oral
• Fiebre tifoidea severa,
Hepatitis granulomatosa
• Cepas resistentes de S. typhi.
Es la complicación más frecuente, se presenta en
la segunda semana de enfermedad. . La S. typhi Ciprofloxacina: 500mg c/12hrs por 10-14 días
producen afectación del sistema hepato-biliar en Azitromicina: 1gr el 1er día y 500mg los 6 días
grado variable (1-26%). siguientes
El cuadro clínico es de una hepatitis aguda
caracterizado por un síndrome ictérico febril. AZITROMICINA
Se recomienda utilizar azitromicina en los casos
Biopsia hepática de brote epidémico de fiebre tifoidea o cuando se
Muestra una hepatitis reactiva inespecífica, trate de resistencia probada de S typhi a fármacos
aunque en algunos pacientes pueden observarse de primera línea, siempre y cuando exista la
los característicos nódulos tifoideos que son posibilidad de ofrecer el Mtratamiento de forma
granulomas con una importante proliferación de ambulatoria:
las células de Kupffer.

196
Manual de INFECTOLOGÍA PLUS MEDIC A

Tratamiento del portador


Bibliografía
Para el tratamiento de erradicación del estado
de portador se recomiendan los siguientes 1.Centro Nacional de Epidemiología. Resultados de la
antimicrobianos de primera elección: vigilancia epidemiológica de las enfermedades
transmisibles. Informe anual 2013. Madrid, 2015.
1. Ciprofloxacina:
Disponibleen: http ://www.isciii.es/ISCIII/es/
Niños: 15 a 20 mg/Kg/día por vía oral
cada 12 horas por 28 días contenidos/fd-servicios-cientifico-tecnic...
Adultos: 750 mg por via oral cada12 horas
por 28 días 2.Jackson BR, Iqbal S, Mahon B; Centers for
2. Amoxicilina: Disease Control and Prevention (CDC). Updated
recommendations for the use of typhoid vaccine--
Niños: 100 mg/Kg/día por vía oral cada 6
Advisory Committee on Immunization Practices,
horas por 6 semanas
United States, 2015. MMWR Morb Mortal Wkly
Rep. 2015;64:305-8.
Tratamiento en gestantes
En mujeres embarazadas con fiebre tifoidea el
tratamiento de elección es ampicilina, 3.The Green Book, chapter 33: Typhoid (last
amoxicilina o cefalosporinas de tercera updated, 28 August 2015)
generación como cefotaxima o ceftriaxona.
v2_0.https://www.gov.uk/ government/uploads/
Perforación intestinal system/ uploads/ attachment_data/fil...
Si se confirma la perforación, la reparación
quirúrgica no debe retardarse por más de 6
horas. La intervención temprana es crucial, y los 4.WHO. Guidelines on the quality, safety and
rangos de mortalidad incrementan cuando se efficacy of typhoid conjugate vaccines. 2013.
prolonga el tiempo entre a perforación y la
intervención quirúrgica. Disponible en:http://www.who.int/biologicals/
areas/vaccines/TYPHOID_BS2215_doc_v1.14
Vacunas _W...
Las vacunas principales contra la fiebre
tifoidea son la oral Ty21a y la parenteral del
polisacárido Vi.
Ambas proveen una protección hasta del 65-
70%, con una inmunidad de 3 a 7 años

EsSalud 01 (20): En un paciente con fiebre


TIFOIDEA lo correcto es:
a) La complicación más severa es enterorragia
b) El antimicrobiano de primera elección es
cotrimoxazol
c) El bilicultivo es el método de mayor
sensibilidad para el diagnóstico
d) La perforación intestinal sólo requiere
tratamiento medico
e) NA
Rpta. E

197
Manual de INFECTOLOGÍA PLUS MEDIC A

198
Manual de INFECTOLOGÍA PLUS MEDIC A

LEPTOSPIROSIS PLUS MEDIC A

INTRODUCCIÓN
Leptospiras saprofitas
La leptospirosis es una enfermedad zoonótica que es
Entre sus especies se han identificado leptospiras
producida por leptospiras patógenas, que se
saprofitas, libres, no patógenas pertenecientes a la
caracteriza por un amplio espectro de especie L. Biflexa, L. wolbachi y la L. parva
manifestaciones clínicas que oscilan desde una Entre sus especies se han identificado leptospiras
infección inaparente hasta una enfermedad saprofitas, libres, no patógenas pertenecientes a la
fulminante y fatal. especie L. Biflexa, L. wolbachi y la L. parva.

Sinónimos: Enfermedad de Weil, Leptospiras patógenas


enfermedad de los porqueros, fiebre de los L. INTERROGANS, L BORGPETERSENII, L NOGUCHII, L
arrozales o fiebre de los cañaverales. SANTAROSAI, L KIRSCHNERI, L WEILII son
consideradas patógena tanto para animales como
para humanos, pudiendo persistir por largos
periodos de tiempo en animales infectados y en el
AgENTE ETIOLÓgICO ambiente.
L. Interrogans es la que se aísla con mayor
frecuencia de gran número de especies animales y
Orden: también de personas.

EPIDEMIOLOgIA
La leptospirosis tiene alta prevalencia en los
países tropicales ya que el clima y las condiciones
desfavorables de trabajo e higiene favorecen la
supervivencia del agente patógeno.
Spirochaetales En las diferentes regiones los serotipos varían y
Familia:Leptospiraceae dependen de la ecología del medio que los alberga.
Género: Leptospira Sin embargo hay serovares que son universales
Forma: helicoidal, flexibles, móviles en espiral (L. Interrogans con sus serovares
con extremos curvos como ganchos, son icterohaemorrhagiae y canícola) así como hay
aerobias. serovares que solo se presentan en algunas
La Leptospira sobrevive hasta 180 días en regiones.
suelos húmedos, por varios meses en
superficies acuosas y sobreviven aún mejor En Perú los serovares con mayor prevalencia
en agua estancada que en movimiento. son icterohemorragiae, grippothyphosa,
pomona, cynopteri, georgia, canícola,
Es altamente susceptible a la desecación y a djasiman y autumnalis.
los cambios de pH (pH<6 y pH>8) .
Los rangos extremos de temperaturas (<7-10 La distribución de los casos de leptospirosis en el
ºC o >34-36ºC) son nocivas.
país incluye los departamentos de:
Se han identificado 16 especies genómicas de LAMBAYEQUE, JUNÍN, PIURA, CUZCO, LIMA, SAN
leptospiras patógenas. Las leptospiras MARTIN, LORETO, UCAYALI, HUÁNUCO, MADRE DE DIOS,
patógenas se dividen en más de 200 TUMBES, LA LIBERTAD.
serovariedades integran 23 serogrupos.

199
Manual de INFECTOLOGÍA PLUS MEDIC A

FACTORES DE RIESGO Los reservorios domésticos más importantes


incluyen a los bovinos, porcinos, equinos, caninos,
El contacto directo entre roedores es uno de los ovinos y caprinos.
principales factores de riesgo.
La leptospirosis se asocia a diferentes ocupaciones En los reservorios la infección suele ser oligo o
por lo que en algunos casos es una enfermedad de asintomática pero mantienen leptospiruria por
tipo ocupacional.
largo tiempo y en algunas especies por toda la vida.
La transmisión depende de muchos factores como
el clima, la densidad y el grado de contacto entre el
-Agricultores (cortadores de
caña de azúcar, arroceros) reservorio y los hospederos accidentales.
-Médicos veterinarios.
-Técnicos agropecuarios Algunos serotipos se asocian con determinados
animales, por ejemplo:
-Trabajadores de limpieza de alcantarillas. ►El serotipo icterohemorrhagiae y copenhageni con las
-Mineros. ratas, el hardjo con el ganado bovino, el canicola con los
-Carniceros. perros y el Pomona con los cerdos, pero también pueden
-Militares. presentarse en otros animales.
Las variedades que infectan a los reptiles y
En el área urbana hay grupos poblacionales más anfibios (ranas) al parecer no infectan al ser
expuestos y son aquellos que viven en áreas humano.
sujetas a inundaciones o sin saneamiento
adecuado y que puedan estar en contacto con
orina del roedor.

Las aglomeraciones de animales también pueden


considerarse factores de riesgo para la ocurrencia
de leptospirosis.
Así como las inundaciones y las copiosas lluvias
principalmente de la selva tropical.

RESERVORIO

Los reservorios de las leptospirosis son una serie


de animales salvajes y domésticos.

La serovariedad de la leptospira infectante varía


de acuerdo con el animal afectado.

El reservorio más importante Los canes


son roedores, ante todo las Han sido considerados una fuente habitual de
ratas, pero también hay otros infección para el hombre, constituyéndose en
mamíferos silvestres como reservorio de las serovariedades canicola e
zorros, mapaches , marsupia- icterohaemorragiae.
les, entre otros.
Las ratas

200
Manual de INFECTOLOGÍA PLUS MEDIC A

Especialmente “rattus norvegicus” es considerado MODO DE TRANSMISIÓN


uno de los mayores responsables de la transmisión
de la leptospira al hombre. El agua tiene un papel La leptospira es transmitida entre animales y
primordial en la transmisión de la enfermedad. del animal al hombre.
FUENTES DE INFECCIÓN
La transmisión interhumana es muy rara y no tiene
Las fuentes de infección importancia práctica.
de la leptospirosis son
bastante variadas: Forma Indirecta
Es la más frecuente y ocurre por contacto de las
mucosas nasal, oral, conjuntival, genital y/o piel con
lesiones contaminadas con agua, lodo o terrenos
- -Agua contaminada. con orina de animales infectados.
-Piel y orina de animales infectados.
Forma Directa
Los alimentos también pueden constituirse en Por contacto con sangre, tejidos, órganos u orina de
fuentes de infección si son contaminados con animales infectados.
orina de animales infectados
201
Manual de INFECTOLOGÍA PLUS MEDIC A

Es de 7 a 26 días, con un promedio de 12 días.


Y en raras ocasiones por ingesta de agua y
alimentos contaminados. Fase leptospirémica de la enfermedad

La transmisión accidental en el laboratorio El microorganismo penetra a través de la piel


también ha sido descrita pero es rara. integra reblandecida por el agua, por piel
lesionada o mucosas integras (incluyendo la
Infección transplacentaria pulmonar por inhalación de aerosoles que
Es común entre animales contienen gérmenes y alcanza rápidamente
Cuando eso ocurre puede resultar en aborto, parto el torrente sanguíneo, diseminándose a todos los
prematuro o nacimiento de animales debilitados. órganos

Leptospiras patógenas La leptospira se multiplica en la sangre


(L. interrogans) solo se multiplican dentro de los y en los tejidos, y puede aislarse tanto en
organismos animales
la sangre como en el LCR en los primeros 4
►Los focos de leptospirosis precisan de a 10 días de la enfermedad; su movimiento
condiciones ambientales favorables para la en tirabuzón y la producción de
supervivencia del agente causal en el medio hialuranidasa, pueden explicar la
exterior. penetración a estos sitios.
Así el agua salina no favorece su
supervivencia al contrario del agua dulce
(arroyos, embalses naturales, etc).
El estudio del LCR
►Necesitan un alto grado de humedad, un pH neutro En ese lapso corrobora la presencia de
o ligeramente alcalino y temperaturas adecuadas. pleocitosis en la mayor parte de los casos, pero
►La composición del suelo también influye en su sólo una pequeña proporción de enfermos
supervivencia. termina por mostrar síntomas y signos de
En resumen la fuente de infección y el modo meningitis en ese período.
de transmisión son variados y tienen Leptospira es resistente a la actividad
relación con actividades de riesgo bactericida del suero normal y en ausencia
de anticuerpos específicos no es fagocitada
ni destruida por los polimorfonucleares o
4. Patogenia y Fisiopatología macrófagos.
Periodo de incubación
La lesión pulmonar MANIfESTACIONES CLíNICAS
Es consecuencia de la hemorragia y no de la
inflamación. La expresión clínica de la infección por
En el músculo estriado Leptospira varía ampliamente en el ser humano:
Provocan tumefacción, degeneración vacuolar -►Procesos totalmente asintomáticos, que son los más
de las miofibrillas y necrosis focal. En los casos frecuentes.
graves, la vasculitis altera finalmente la -►Formas de evolución generalmente benignas.
microcirculación y aumenta la permeabilidad
-►Cuadros graves icterohemorrágicos con colapso
capilar con la consiguiente pérdida de líquidos e
vascular y serio compromiso de funcionamiento
hipovolemia.
hepático-renal, que pude ser de evolución fatal
(enfermedad de Weil).
Uveitis crónica
De las formas clínicas sintomáticas de la enfermedad:
La persistencia de las leptospiras en el humor
Θ El 80-90% evoluciona en una forma anictérica
acuoso da lugar ocasionalmente a uveítis crónica
o recidivante. benigna
Θ 10-20% como leptospirosis grave con ictericia e
insuficiencia renal.

202
Manual de INFECTOLOGÍA PLUS MEDIC A

Leptospirosis Sintomática En general, los síntomas duran sólo unos


días pero a veces persisten durante semanas
La leptospirosis es una enfermedad bifásica: En esta fase, la fiebre es menos acusada y las
-Fase inicial o de leptospiremia con una mialgias menos intensas que en la fase
duración de tres a siete días, caracterizada por la leptospirémica.
presencia de las Leptospira en sangre. En este periodo aparece una complicación
importante:
-Segunda fase inmune o leptospiruria con la meningitis aséptica; la cual se observa en
menos del 15% de los enfermos y siendo más
una duración de 8 a 30 días donde se puede común en niños. Los síntomas meníngeos
detectar anticuerpos específicos en circulación. desaparecen por lo general al cabo de unos
días, pero a veces persisten varias semanas. De
Ambas fases son comunes a las dos formas modo similar, la pleocitosis suele desaparecer
clínicas de presentación: anictérica e ictérica. en dos semanas, pero puede persistir algunos
meses.
Leptospirosis anictérica. La iritis, la iridociclitis, la uveítis y la
-Primera fase : es comúnmente referida como coriorretinitis, son complicaciones tardías
fase septicémica. que en ocasiones se prolongan durante años,
La leptospirosis puede presentarse como un pueden ocurrir ya en la tercera semana
proceso seudogripal agudo con :
aunque casi siempre se manifiestan varios
►Cefalea intensa y persistente, mialgias en la meses después del proceso inicial.
región lumbar y gemelar, inyección
conjuntival, escalofríos y dolor abdominal. Leptospirosis ictérica (Síndrome de Weil).
►Se presentan nauseas, vómitos y malestar Es la forma más grave de la enfermedad,
general con postración. caracterizada por el curso fulminante de rápida
►La fiebre es de carácter remitente instauración de fallo hepático y renal, desarrollo de
alcanzando 40 ºC o más. hemorragias, colapso vascular, alteraciones graves
►Con cierta frecuencia se observa un exantema de la conciencia y una mortalidad
macular de pocas horas de duración, en el tronco. aproximadamente de 5 - 40%.
Se puede presentar confusión mental, tos,
dolor torácico o hemoptisis y exantema petequial Inicio:
en el paladar. Es similar a la forma anictérica, pero al cabo de
tres a siete días de evolución, los síntomas
EL DATO MÁS COMÚN EN LA EXPLORACIÓN FÍSICA ES alcanzan su máxima intensidad.
LA FIEBRE, junto con la conjuntivitis. La ictericia es una manifestación constante y está
Los síntomas remiten en la mayoría de los asociada con daño hepatocelular, con
enfermos en una semana. La enfermedad predominancia de la bilirrubina directa.
recidiva en algunas ocasiones tras un plazo de Después de la primera semana disminuyen ligeramente los
uno a tres días.
síntomas, pero no se aprecia el patrón bifásico de la
La evolución de estos casos es usualmente
leptospirosis anictérica.
satisfactoria en un periodo de cuatro a diez días.
Insuficiencia renal
-Periodo afebril : de duración variable que ►Se presenta casi siempre en la segunda semana de la
luego es seguida de una fase inmunitaria. enfermedad.
►La necrosis tubular aguda, con oliguria o anuria,
Inicio: coincide con la aparición de los
contribuye a la hipovolemia y al descenso de la perfusión
anticuerpos. renal.

203
Manual de INFECTOLOGÍA PLUS MEDIC A

Lesión pulmonar Citoquímico de LCR:


Es frecuente y concurre con tos, disnea, dolor de
pecho y expectoración teñida de sangre o incluso En casos de meningitis muestra elementos
hemoptisis e insuficiencia respiratoria.
inflamatorios: opalescencia, xantocromía,
Manifestaciones hemorrágicas aumento de proteína (hasta 1g) y células con
El síndrome de Weil conlleva manifestaciones recuento, habitualmente, entre 100 y 800/ mm3
hemorrágicas como : con predominio linfocitario.
Epistaxis, petequias, púrpura y equimosis, si bien
la hemorragia digestiva grave y la hemorragia Exámenes específicos
suprarrenal o subaracnoidea son raras.
La clínica con los hallazgos de laboratorio
Puede desarrollarse delirio y convulsiones, mencionados pueden sugerir un caso probable
junto con la aparición de manifestaciones de leptospirosis pero no es específico, por lo que
hemorrágicas diversas y acentuación de la el diagnóstico probado de leptospirosis se basa
ictericia.
en la identificación del microorganismo en
En la leptospirosis grave se han descrito:
cultivo o en la seroconversión, o el incremento en
Rabdomiólisis, hemólisis, miocarditis,
pericarditis, insuficiencia cardíaca el título de anticuerpos en la prueba de
congestiva, choque cardiógeno, síndrome aglutinación microscópica (microscopic
apneico del adulto y fracaso multiorgánico. agglutination test, MAT)

DIAgNÓSTICO Cultivo: Puede recuperarse leptospiras


durante los primeros 7 a 10 días de
Datos analíticos enfermedad, en sangre, tejidos o LCR y de la
orina a partir de la segunda a cuarta semana de
Hemograma: Puede observarse leucocitosis enfermedad.
moderada con desviación a la izquierda. Se ha La bacteria puede permanecer en la orina
descrito trombocitopenia, en pacientes con daño hasta 11 meses después de iniciada la
renal. enfermedad. Para ello, puede efectuarse
cultivo en medios especiales, semisólidos
Velocidad de eritrosedimentación: Se eleva en como el Medio de Fletcher enriquecido con
suero de conejo, durante 5 a 6 semanas a
forma también moderada.
28-30 ºC, en ambiente oscuro.

Análisis de orina: Puede evidenciar proteinuria, Detección de antígenos:


hematuria microscópica o cilindruria, en Dentro de las técnicas que se pueden utilizar se
pacientes con compromiso renal. encuentran el radio-inmunoanálisis (RIA), prueba
de inmunoabsorbente ligado a enzimas (ELISA); de
las cuales la última mencionada es más popular.
Creatininemia y nitrógeno ureico: Se elevan en
los casos con daño renal. Detección de anticuerpos:
El test de aglutinación microscópica (MAT) es uno
Pruebas de función hepática: En enfermos con de los métodos serológicos por excelencia debido a
compromiso hepático se puede observar su alta sensibilidad y especificidad, además
hiperbilirrubinemia, aumento de GPT y GOT y puede identificar el serovar o serogrupo de
aumento de fosfatasa alcalina. Leptospira comprometida en la infección.

204
Manual de INFECTOLOGÍA PLUS MEDIC A

El diagnóstico molecular se puede realizar a través de Dengue


reacción de polimerasa en cadena (PCR) y la La diferenciación básicamente será por la
tipificación por medio de digestión de ADN por procedencia del paciente y los datos
enzimas de restricción (REA), restricción del epidemiológicos en general además por las pruebas
largo del fragmento polimórfico (RFLP) entre laboratoriales específicas de cada enfermedad.
otras pruebas rápidas de utilidad diagnóstica; Clínicamente las características son similares
estas pruebas que más bien utilizan la aunque la intensidad de los dolores musculares
metodología de flujo lateral, aglutinación podría ser mayor en el dengue.

DIAGNOSTICO DIFERENCIAL Hepatitis


Las hepatitis virales son de inicio menos brusco
El espectro de diagnóstico depende de la forma con fiebre de menor intensidad y que desaparece al
clínica de leptospirosis iniciarse la ictericia. Frecuentemente no hay
El diagnóstico de leptospirosis debe ser mialgias y compromiso renal.
investigado en todo paciente con fiebre y
mialgias, toda vez que las manifestaciones Meningoencefalitis
El diagnóstico diferencial de las formas meníngeas
clínicas son superponibles con otras
de la leptospirosis es generalmente establecido
enfermedades.
por el examen del LCR y la evolución del cuadro.
Enfermedades que en ocasiones tienen una
El LCR de las meningitis bacterianas presentará
presentación clínica indistinguible de la
leucocitosis con hipoglucorraquia lo que no
leptospirosis pueden ser sospechadas por la ocurre en la leptospirosis.
historia epidemiológica como la fiebre
amarilla y la malaria. Malaria por P. Vivax o P. falciparum (en caso
de ictericia o hemorragia)
En nuestro país la necesidad de exámenes de La curva térmica característica ayudará al
laboratorio se hace imperante para el diagnóstico, además existe hepatoes-
diagnóstico diferencial de malaria, dengue y plenomegalia, anemia de rápida progresión y no
fiebre amarilla por el síndrome es frecuente la leucocitosis con neutrofilia así
icterohemorrágico. como la presencia de congestión cutáneo mucosa
Otras enfermedades pueden eventualmente
Fiebre Amarilla ser confundidas con la leptospirosis como
El período virémico de la fiebre amarilla puede colecistitis, infecciones respiratorias, sarampión,
confundirse con el mismo período de la rubéola, pielonefritis, sepsis, brucelosis con
ictericia, Sepsis bacterianas graves con ictericia,
leptospirosis, los fenómenos son esencialmente endocarditis.
los mismos pero en la FA la duración de esta fase
es mucho más corta y la intensidad mucho mayor.
TRATAMIENTO
En la fase toxémica de la FA podría haber
también confusión con el período en el cual la
El tratamiento se basa principalmente en la
leptospirosis se localiza en los tejidos (2da fase
terapia de :
- inmune) sin embargo la intensidad y la ►Soporte
presencia del vómito negro ayudara a la
►Corrección del desequilibrio electrolítico y ácido
diferenciación con la FA.
básico
Algunas veces sólo pruebas diagnósticas
a fin de evitar la descompensación
específicas de ambas enfermedades serán
capaces de hacer el diagnóstico diferencial. ocasionada por la fiebre, anorexia y vómitos.
Esta es una de las razones por lo que ambas han En casos graves con oliguria, se debe tener
sido incluidas dentro de la vigilancia del cuidado con la reposición hídrica excesiva, que
síndrome ictero hemorrágico. puede empeorar la insuficiencia respiratoria,
pudiendo llegar hasta insuficiencia cardiaca.

205
Manual de INFECTOLOGÍA PLUS MEDIC A

Si a pesar de las medidas adoptadas, no mejora la El manejo y el tratamiento de leptospirosis


insuficiencia renal se debe indicar precozmente de moderado a grave deben ser en forma
la diálisis peritoneal o derivación a un hospitalaria.
establecimiento de salud que cuente con unidad
Leptospiras son sensitivas a una variedad de
de cuidados intensivos (UCI).
agentes antimicrobianos incluyendo las penicilinas,
Todo paciente con diagnóstico presuntivo de
cefalosporinas, aminoglucósidos, tetraciclinas y
leptospirosis debe ser hospitalizado si es que se
presenta los siguientes signos de alarma: macrólidos.

-►Fiebre elevada que no cede a antipiréticos (39 PRONÓSTICO


La mayoría de los enfermos con leptospirosis se
ºC)
recupera.
-►Vómitos persistentes. ►La mortalidad es mayor entre los enfermos ancianos
-►Dolor abdominal intenso que puede llegar al o con síndrome de Weil.
abdomen agudo Manifestaciones hemorrágicas ►La leptospirosis contraída durante el embarazo se
vincula con alta mortalidad fetal.
(gingivorragia, hemoptisis, melena, petequias El seguimiento prolongado de los enfermos
generalizadas). con insuficiencia renal y alteraciones de la
-►Ictericia. función hepática revela una buena
recuperación de ambos órganos.
-►Dificultad respiratoria.
Cuando se inicia el tratamiento
-►Trastornos hemodinámicos (shock). antimicrobiano de la leptospirosis puede
-►Oliguria. ocurrir una REACCIÓN DE JARISCH-
HERXHEIMER
-►Signos meníngeos. ►Es parecida a la que se observa en otras
enfermedades por espiroquetas.
◆ Para grupos de personas que ingresen a zonas
►Si bien esta reacción se describía a menudo en las
endémicas en forma temporal : publicaciones antiguas, se produce muy rara vez en la
leptospirosis
Personal militar, practicantes de deportes de aventura,
brigadistas y otrosse recomienda aplicar: Es menos común en esta infección que en
otras enfermedades producidas por
►En adultos: espiroquetas.

Doxiciclina 200 mg VO una vez por semana o PREVENCION


Amoxicilina 500 mg VO una vez por semana Medidas sanitarias mejoran las condiciones
socioeconómicas, el autocuidado y la
►En niños : autoprotección utilizando métodos de barrera,
que protejan piel y mucosas, cuando se realizan
Amoxicilina 250 mg VO una vez por semana. El actividades con riesgo de contaminación:
tratamiento quimioprofiláctico está recomendado ►Control de roedores cerca de los hogares.
mientras dure la estadía. ►Drenaje de aguas estancadas.
La antibioticoterapia se debe iniciar lo más ►Uso de guantes y botas de goma en industrias
temprano posible para evitar las lesiones en los de riesgo
tejidos. ►Manejo adecuado de los residuos sólidos.

206
Manual de INFECTOLOGÍA PLUS MEDIC A

ANEXO PLUS MEDIC A


Osteomielitis
Hematógena Pie diabético infectado
Tto empírico previo hemocultivo Úlcera sin inflamación
y cultivo de la cura ósea Flora colonizadora de la piel
Adultos > 21 años No requiere antibióticos
Etiología Infección leve
S aureus Etiología
Esquemas -S aureus
SAMR posible: -S pyogenes
Vancomicina 15-30 mg/kg IV c/12 h Tratamiento oral
+ Ceftriaxona 2g IV c/d o -Dicloxacilina 500 mg c/6h o
Cefepime 2g IV c/8h Cefalexina 500mg VO c/6h o
Levofloxacina 750mg c/d Amoxicilina/Ac clavulánico 875/125 mg c/12h o
SAMR improbable: Clindamicina 300mg VO c/8h
Oxacilina 2g IV c/6h Infección moderada
Etiología
+ Ceftriaxona 2g IV c/d o -S aureus
Cefepime 2g IV c/8h -S pyogenes
Levofloxacina 750mg c/d -Coliformes
NEJM 362: 11, 2010 Tratamiento
Oral: igual que en infección leve
Meningitis Parenteral:
De 1 mes a 50 años -Ampicilina/sulbactam 3g IV c/6h o
Etiología Ticarcilina/clavulanato o
S pnumoniae Piperacilina/Tazobactam 3.375 IV c/6h
Meningococo Ertapenem 1g IV c/d u otro carbapenems
Esquemas
+ Vancomicina 1g IV c/12h o
Cefotaxima 2g IV c/4-6 h o
Daptomicina 6mg/kg IV c/d o
Ceftriaxona 2g IV c/12h Linelozid 600mg IV c/12h
+ Vancomicina 2g IV c/d Infección local extensa más toxicidad
+ Dexamentasona 0.15 mg/Kg c/6h por sistémica
2-3 d Etiología
NEJM 357:2431 – 2441,2007 -S aureus
-S pyogenes
> 50 años o alcoholismo o enferme- dades -Coliformes
debilitantes -Anaerobios
Etiología Tratamiento
S pneumoniae Parenteral:
Listeria monocitógenes -Vancomicina 1g IV c/12h
BGN + Blactámicos /Inhibidor de betalactama- sas o
Esquemas -Vancomicina 1g IV c/12h
Ampicilina 2g IV c/4 + Doripenem 500mg (infusión 1h) o
+ Ceftriaxona 2g IV c/12h o Imipenem 500 mg IV c/6h o
Cefotaxima 2g IV c/4-6h Meropenem 1g IV c/8h o
+ Vancomicina Ertapenem 1g IV c/24h
+ Dexamentasona 0.15 mg/Kg c/6h por Sanford Guide To Antimicrobial Therapy 2017
2-3 d
Sanford Guideline To Antimicrobial Therapy 2017

207
Manual de INFECTOLOGÍA PLUS MEDIC A

208

Vous aimerez peut-être aussi